Chapter 1 Foundations for medical-surgical nursing, Ch 1, Chapter 1: Introduction to Medical-Surgical Nursing, Chapter 2 Interprofessional collaboration and care coordination, Medical Surgical Chapter 2, Chapter 3: Cultural Considerations, ch 4 Ethic...

¡Supera tus tareas y exámenes ahora con Quizwiz!

Culture and Illness: Mennonite women

Wear head coverings during hospitalization

Sodium content of common foods

-frozen pizza, plain cheese -tomato soup/juice -pretzels -potato chips -salsa -salad dressing -tortilla chips -breads -frozen vegetable

Magnesium Foods

-halibut -almonds, cashews, NUTS -spinach -cereal, bran, shredded wheat, oatmeal -potato -PB, -BEANS, rice, lentils -avocado -yogurt -chocolate milk, milk -banana -bread -raisins -chocolate pudding

A patient with *acute renal failure* has *jugular vein distention*, *LE edema*, & *elevated blood pressure*. Based on this data, *which nursing diagnosis is the most appropriate*?

*Excess Fluid Volume* Jugular vein distention, edema, and elevated blood pressure are indications of excessive fluid. The diagnosis Excess Fluid Volume should be selected to guide this patient's care.

Calcium foods

-milk -orange/grapfruit -canned salmon -american cheese -broccoli -tofu -kale -ice cream -egg

The nurse is planning care for an older adult patient with *respiratory acidosis*. Which intervention should the nurse include in this patient's plan of care?

*Maintain adequate hydration*. in respiratory acidosis= drop in blood pH, reduced lvl of oxygen, & retaining of CO2. the body needs to be well-hydrated

The nurse is providing care to an older adult patient who is receiving intravenous (IV) fluids at 150 mL/hr. The patient is currently exhibiting *crackles in the lungs, shortness of breath, and jugular vein distention*. Which complication of IV fluid therapy does the nurse suspect the patient is experiencing?

*Fluid volume excess* Fluid volume excess may occur when older adult patients receive intravenous fluids rapidly.

A patient is admitted to the emergency department (ED) for *fluid volume deficit*. Which *body system should the nurse focus* to determine the cause of this imbalance when assessing this patient?

*Gastrointestinal* The most common cause of fluid volume deficit is excessive loss of gastrointestinal fluids, which can result from vomiting, diarrhea, suctioning, intestinal fistulas, or intestinal drainage. Other causes of fluid losses include chronic abuse of laxatives and/or enemas.

The nurse is reviewing lab values for a female patient suspected of having a *fluid imbalance*. Which laboratory value evaluated by the nurse supports the *diagnosis of dehydration*?

*Hematocrit 53%* The hematocrit measures the volume of whole blood that is composed of RBCs. Because the hematocrit is a measure of the volume of cells in relation to plasma, it is affected by changes in plasma volume. The hematocrit increases with severe dehydration.

(CC 8.6) The nurse prioritizes the patient with which electrolyte abnormality at greatest risk for *ventricular fibrillation*?

*Hypokalemia*

The nurse is providing care to a patient who has been vomiting for several days. The nurse knows that the patient is at *risk for metabolic alkalosis because gastric secretions have which characteristic*?

*Gastric secretions are acidic*. -metabolic alkalosis due to loss of hydrogen ions usually occurs because of vomiting or gastric suction. gastric secretions are highly acidic (ph 1-3). increased alkalinity results from the loss of acid from selective retention of bicarbonate by the kidneys as chloride is depleted

The nurse is providing care to a patient who seeks emergency treatment for *headache and nausea*. The patient works in a mill without air conditioning. The patient states, "I drink water several times each day but I seem to *sweat more than I am able to replace*." Which suggestions should the nurse provide to this patient?

*Eat something salty when drinking water*. Both salt and water are lost through sweating. When only water is replaced, the individual is at risk for salt depletion. Symptoms include fatigue, weakness, headache, and gastrointestinal symptoms such as loss of appetite and nausea. The client should be instructed to eat something salty when drinking water to help replace the loss of sodium.

The nurse is instructing a patient with heart failure about a prescribed sodium-restricted diet. Which patient statement indicates that additional teaching is required?

"*I can use as much salt substitute as I want*." Low-sodium salt substitutes are not really sodium-free. They may contain half as much sodium as regular salt. The patient should be instructed to use salt substitutes sparingly because larger amounts often taste bitter instead of salty.

The nurse is caring for a patient with a *potassium level of 5.9 mEq/L*. The health-care provider prescribes both glucose and insulin for the patient. The patient's spouse asks, "Why is insulin needed?" Which response by the nurse is the most appropriate?

"*The insulin will cause his extra potassium to move into his cells*, which will lower potassium in the blood." Serum potassium levels may be temporarily lowered by administering glucose and insulin, which cause potassium to leave the extracellular fluid and enter cells.

The nurse is caring for a patient who is receiving IV fluids post-op following *cardiac surgery*. The nurse is aware that *this patient is at risk for fluid volume excess*. The family asks why the patient is at risk for this condition. Which response by the nurse is the most appropriate?

"Fluid volume excess is common due to *increased levels of ADH in response to the stress* of surgery." Antidiuretic hormone (ADH) and aldosterone levels are commonly increased following the stress response before, during, and immediately after surgery. This increase leads to sodium and water retention. Adding more fluids intravenously can cause a fluid volume excess and stress upon the heart and circulatory system.

The nurse completes discharge teaching for a patient with an anxiety disorder. Which patient statement indicates correct understanding of information related to respiratory alkalosis?

"I will see my counselor on a regular basis."

The nurse is providing care to an adult patient admitted with *dehydration* and *hyponatremia*. *Which* medical condition *supports the current nursing diagnosis of Electrolyte Imbalance*?

*Hypotonic dehydration* Hypotonic dehydration occurs when fluid loss is characterized by a proportionately greater loss of sodium than water, causing serum sodium to fall below normal levels

HYPOvolemia complications

*Hypovolemic shock* can develop as evidenced by hypotension, tachycardia, and signs of organ hypoperfusion such as cool, clammy skin, oliguria progressing to anuria (lack of urine output), decreased level of consciousness, and tachypnea.

What are alternative healers for African American?

"Root doctor" "voodoo priest" "spiritualist"

An older adult patient, who lives in a long-term care facility, presents in the emergency department (ED) due to *fever, nausea, and vomiting over the past two days*. The patient *denies thirst*. The urine dipstick indicates a *decreased urine specific gravity*. Which medical diagnosis should the nurse anticipate when planning care for this patient?

*Dehydration* Older adult patients are less able to concentrate their urine, making them susceptible to dehydration. In addition, there is a deficit of the thirst response. However, fever, nausea, and vomiting resulting from these changes are not considered normal. The patient's symptoms of nausea and vomiting suggest decreased intake and increased output through vomiting, placing the client at risk for dehydration.

The nurse is caring for a *comatose patient with respiratory acidosis*. For which intervention will the nurse need to collaborate when caring for this patient?

*Determining recent eating behaviors*

Steps of Evidence-based practice

*Develop the question *Search and collate the best evidence *Evaluate the quality of the evidence *Integrate evidence into practice *Evaluate outcomes of practice change *Disseminate the evidence

Which patient statement indicates the need for *additional education* regarding the use of sodium *bicarbonate to treat acidosis*?

*"I should use the antacid for at least 2 months."* use no longer than 2 weeks

The nurse is teaching a group of children and their parents about the prevention of *heat-related illness during exercise*. Which statement by a parent indicates an appropriate *understanding* of the preventive techniques taught during the teaching session?

*"I will have my child stop every 15-20 minutes during the activity for fluids."* During activity, stopping for fluids every 15-20 minutes is recommended.

isotonic solutions

*250-375 mOsm/L* administered to dehydrated patients with deficits in intravascular volume because they increase the amount of fluid circulating in the vascular system without causing movement of fluid in and out of cells examples: 0.9% sodium chloride (0.9% NaCl, NS) 5% dextrose in water (D5W) lactated Ringer's solution (LR) indications: fluid deficits dehydration fluid challenges

A patient is admitted to the emergency department (ED) for *dehydration*. The patient is *154 lbs*. Which *urine output* indicate the rehydration efforts for this patient have been effective?

*35 mL/hr* Expected urine output for an adult patient is 0.5 mL/kg/hr. The patient currently weighs 70 kg; therefore, adequate urine output would be at least 35 mL/hr.

Which diagnostic test should the nurse anticipate when providing care to a patient diagnosed with chronic obstructive pulmonary disease (*COPD*) to monitor acid-base balance?

*ABG*

The nurse is providing care for a patient admitted to the unit with *respiratory failure and respiratory acidosis*. Which data from the nursing history is the probable cause for the patient's current diagnoses?

*Aspiration pneumonia*

The nurse is providing care to a patient who is prescribed *furosemide* as part of the treatment for congestive heart failure (CHF). The patient's serum *potassium* level is *3.4 mEq/L*. Which food should the nurse encourage the patient to eat based on this data?

*Bananas* A potassium level of 3.4 is low, so the client should be encouraged to consume potassium-rich foods. Of the foods listed, the highest in potassium is banana.

The results of a patient's arterial blood gas sample indicate an oxygen level of 72 mmHg. Which should the nurse closely assess when providing care to this patient?

*Cognition* an oxygen level less than 74 can be due to HYPOventilation. this will change the pt's lvl of responsiveness

The nurse is caring for a patient with congestive heart failure who is admitted to the medical-surgical unit with *acute hypokalemia*. Which prescribed *medication may have contributed* to the patient's current hypokalemic state?

*Cortisol* Excess potassium loss through the kidneys is often caused by such medications as corticosteroids, potassium-wasting (loop) diuretics, amphotericin B, and large doses of some antibiotics. Cortisol is a type of corticosteroid and can cause hypokalemia.

Which *chronic lung condition noted in the patient's health history supports* the current diagnosis of *respiratory acidosis*?

*Cystic fibrosis*

Which clinical manifestation supports the nurse's plan of *care* focusing on *chronic respiratory acidosis*?

*Daytime sleepiness*

The nurse is analyzing the patient's ABG report, which reveals a pH of 7.15. The patient has just suffered a *cardiac arrest*. Which *consequences of this pH value* does the nurse consider for this patient?

*Decreased cardiac output* nurse knows that *severe acidosis depresses Myocardial contractility*, which leads to *decreased cardiac output*.

The nurse is caring for a patient admitted with *renal failure & metabolic acidosis*. Which clinical manifestation would indicate to the nurse that planned interventions to *relieve the metabolic acidosis have been effective*?

*Decreased depth of respirations* pt with metabolic acidosis will have increased respiratory rate and depth. effective care will decrease this

An older adult patient, who appears *intermittently confused*, is admitted to the hospital after a fall. Based on the current data, which is the *patient at an increased risk for developing*?

*Dehydration* During the aging process, the thirst mechanism declines. In a patient with an altered level of consciousness, this can increase the risk of dehydration and high serum osmolality.

The nurse is providing care to a patient whose *serum calcium levels have increased* since a surgical procedure performed three days prior. Which *intervention* should the nurse implement to *decrease the risk for the development of hypercalcemia*?

*Encourage ambulation three times a day* *Hypercalcemia can occur from immobility*. Ambulation of the client helps to prevent leaching of calcium from the bones into the serum.

The nurse is providing care for an adult patient who is admitted to the emergency department (ED) after passing out. The patient has been *fasting and currently has ketones in the urine*. Which acid-based imbalance should the nurse monitor the patient for based on the current data?

*Metabolic acidosis* -pt who are fasting = risk for metabolic acidosis. The body recognizes fasting as starvation & begins to metabolize its own proteins into ketones, which are metabolic acid

The nurse is providing care to a patient who is diagnosed with multisystem *fluid volume deficit*. The patient is currently experiencing *tachycardia* and *decreased urine output* along with *skin that is pale and cool to the touch*. The patient has a *decreased urine output*. Which probable cause to the patient's symptoms should the nurse include when educating the family?

*Natural compensatory mechanisms* The internal vasoconstrictive compensatory reactions within the body are responsible for the symptoms exhibited. The body naturally attempts to conserve fluid internally specifically for the brain and heart. ❄️

A patient is admitted to the emergency department for the treatment of a *drug overdose causing acute respiratory acidosis*. Which substance noted on the toxicology report is the most likely cause for the current diagnosis?

*Oxycodone* -excessive use or overdose of narcotic substances can lead to respiratory depression and respiratory alkalosis

The nurse is planning care for the patient with *acute renal failure*. The nurse plans the patient's care based on the nursing diagnosis of Excess Fluid Volume. Which assessment data supports this nursing diagnosis?

*Pitting edema in the lower extremities* The patient in acute renal failure will likely be edematous, as the kidneys are not producing urine.

A patient with metabolic acidosis has been admitted to the unit from the emergency department (ED). The patient is experiencing *confusion and weakness*. Which independent nursing intervention is the *priority*?

*Protecting the patient from injury* the pt w/ metabolic acidosis may have symptoms of drowsiness, lethargy, confusion and weakness

The nurse is providing care to a patient who is admitted after a *morphine overdose*. Which acid-base imbalance should the nurse plan this patient's care to reflect?

*Respiratory acidosis* -Morphine is a narcotic and generally acts tso decrease or suppress respirations; therefore, this patient is probably hypoventilating. The expected acid-base imbalance would be respiratory alkalosis

The client is admitted to the emergency department (ED) with symptoms of a *panic attack*. Based on this data, the nurse plans care for which health problem?

*Respiratory alkalosis*

The nurse is caring for a patient admitted with *HTN* and *chronic renal failure* who receives hemodialysis three times per week. The nurse is assessing the patient's diet and notes the *use of salt substitutes*. When teaching the patient to avoid salt substitute, which rationale supports this teaching point?

*They can potentiate hyperkalemia*. Many salt substitutes use potassium chloride. Potassium intake is carefully regulated in patients with renal failure, and the use of salt substitutes will worsen hyperkalemia.

The nurse is analyzing the I&O record for a patient being treated for *dehydration*. The patient weighs *176 lbs. and had a 24-hour intake of 2,000 mL and urine output of 1,200 mL*. Based on this data, which conclusion by the nurse is the most appropriate?

*Treatment is effective and should continue*. Urinary output is normally equivalent to the amount of fluids ingested; the usual range is 1,500-2,000 mL in 24 hours, or *40-80 mL in 1 hour* (0.5 mL/kg per hour). Patients whose intake substantially exceeds output are at risk for fluid volume excess; however, the patient is dehydrated. The extra fluid intake is being used to improve body fluid balance. The patient's output is 40 mL/hr, which is within the normal range.

Which data collected by the nurse during the assessment process *places the older adult patient at risk for dehydration*?

*Water intake of 2 glasses per day* A poor intake of water could indicate a loss of the thirst response, which occurs as a normal age-related change. Since the patient only ingests two glasses of water each day, this could indicate a reduction in the normal thirst response.

hypertonic solutions

*greater than 375 mOsm/L* have concentrations higher than plasma and cause fluid to move from the cells into the intravascular space because of the danger of circulatory overload, these solutions are given only in critical situations examples: 3% sodium chloride 5% dextrose in lactated Ringer's 20% dextrose in water dextrose 5% in 1/2NSS dextrose 5% NSS albumin 25% indications: severe dehydration severe electrolyte imbalance

hypotonic solutions

*less than 250 mOsm/L* have a lower solution concentration than plasma and cause fluid to move from the intravascular space into both the intracellular and interstitial spaces hydrates cells but results in depletion of intravascular fluid volume examples: 0.45% sodium chloride (0.45% NaCl, 1/2NS) 2.5% dextrose in water 0.33% sodium chloride indications: diabetic ketoacidosis hyperosmolar hyperglycemia

Euvolemic causes

- hypothyroidism -SIADH -Meds: carbamazepine, opioids, barbs, vincritine, phenytoin, pantoprazole

Hyperchloremia treatment

-0.45% NS

Hypernatremia treatment

-Fluid replacement 1/2 NS or D5W

Hyponatremia treatment

-Fluid restriction -In severe neurological symptoms, 3% NS -Oral sodium supplements -Loop diuretics

Hypocalcemia treatment

-IV calcium -Oral supplementation (calcium tablets, Tums)

Hypercalcemia treatment

-IV fluids -Loop diuretics -Bisphosphates

Hypomagnesemia treatment

-IV replacement Magnesium 2 g in 100 mL *NS* or *D5W* over 1 hr -Mg tablets -*IVP during Code Blue* situation

Hypokalemia treatment

-Increase dietary intake or supplementation -Slow-K, KCl liquid, powder or tablets -IV replacements (KCL runs, KCL aliquots): Peripheral IV: KCL 10 mEq in 100 mL NS or D5W over 1 hour Central line (PICC, triple-lumen catheter, or portacath): KCL 10 or 20 mEq in 100 mL NS or D5W to infuse over 1 hour *Never administer as IVP*

Hyperkalemia treatment

-Kayexalate, 50% dextrose & IV regular insulin IVP bolus -Sodium bicarbonate 50 mEq IVP -Calcium gluconate -Albuterol nebulization for 1 hour -Loop diuretics

Hyperphosphatemia treatment

-PhosLo capsules -Oral phosphate-binding drugs -If normal renal function, IV NS and loop diuretics

Hypophosphatemia treatment

-Phosphorous supplementation -Neutra-phos powder -KPhos IV replacement

lumens of the central venous access device

-multiple-lumen catheters provide separate fluid pathways that make it possible to deliver two or more solutions at the same time; each lumen, or fluid pathway, is totally separate from the other lumens (because the lumens are separate, incompatible solutions can be infused using the different ports attached to each of the lumens) -these lumens are referred to as proximal, medial, or distal lumens depending on the location of the end of the fluid path on the catheter -include nontunneled percutaneous central catheters, tunneled catheters, implanted ports (Mediports), and peripherally inserted central catheters (PICCs) -placed by physicians or advance practice registered nurses

Potassium foods

-spinach -baked potato -tomato juice -mushrooms -sweet potato -vegetable juice cocktail -banana -cantaloupe -low-fat milk -kidney beans -avocado -tomato -kiwi -strawberries -orange

The nurse is caring for the patient experiencing hypovolemic shock and metabolic acidosis. Which nursing actions are appropriate for this patient? (SATA)

1) Administer sodium bicarbonate. 2) Monitor ECG for conduction problems. -these are appropriate for a pt with shock 3)Keep the bed in the locked and low position. -pt recovering from shock is at risk for injury, so the bed should be kept in the locked and low position.

The nurse is providing care to a patient who is admitted with manifestations of metabolic alkalosis. Which diagnostic test findings support the admitting diagnosis? (SATA)

1) Blood pH 7.47 and bicarbonate 34 mEq/L 2)Electrocardiogram changes consistent with hypokalemia

A patient recently diagnosed with diabetes mellitus (DM) is hospitalized in diabetic ketoacidosis (DKA) after a religious fast. The patient tells the nurse, "I have fasted during this season every year since I became an adult. I am not going to stop now." The nurse is not knowledgeable about this particular religion. Which nursing actions would be appropriate? (SATA)

1) Request a consult from a diabetes educator. 2) Assess the meaning and context of fasting for this religion. 3) Encourage the patient to seek medical care if signs of ketoacidosis occur in the future.

Which nursing actions are appropriate when conducting an Allen test? (SATA)

1) Rest the patient's arm on the mattress. 2) Support the patient's wrist with a rolled towel. 3) Press the patient's *radial* and *ulnar* arteries using the index and middle fingers.

Which risk factors exhibited by the patient presenting in the emergency department (ED) would place the patient at risk for metabolic acidosis? (SATA)

1)Abdominal fistulas 2)Acute renal failure 3)Hypovolemic shock -metabolic acidosis i s a primary disorder. It usually develops during the course of another disease presence of abdominal fistulas; which cause bicarbonate loss, acute renal failure, and hypovolemic shock

1. A patient presents in the emergency department (ED) with fever, nausea, and vomiting over the past 2 days. The nurse monitors for which laboratory result in this patient? 1. Urine specific gravity of 1.040 2. Serum potassium of 4.8 mEq/L 3. Serum sodium of 135 mEq/L 4. Urine positive for glucose and ketones

1. ANS: 1 Chapter number and title: 8, Fluid and Electrolyte Management Chapter learning objective: 3. Explaining the significance of osmolality, osmolarity, blood urea nitrogen (BUN), creatinine, and urine specific gravity related to fluid and electrolyte status Chapter page reference: 115 - 116 Heading: Fluid and Electrolyte Regulation/Indicators of Fluid Status Integrated Processes: Nursing Process: Assessment Client Need: Physiological Integrity: Physiological Adaptation Cognitive Level: Analysis [Analyzing] Concept: Assessment Difficulty: Moderate Feedback 1 The normal range for specific gravity is 1.005 to 1.030. High specific gravity values indicate concentrated urine and can be seen in patients with decreased renal perfusion or dehydration. 2 Serum potassium is not related to fluid volume status and this is a normal value. The normal range is 3.5 to 5.0 mEq/L. 3 Serum sodium is elevated with fluid loss and this value is low. The normal range is 135 to 145 mEq/L. 4 Urine that is positive for glucose and ketones is observed in patients with diabetes mellitus.

1. The nurse anticipates which fluid movement when administering an isotonic IV fluid to a patient? 1. Fluid moves from the cells into the intravascular space 2. Fluid moves from the intravascular space into the intracellular space 3. Causes no or equal movement of fluid into or out of cells 4. Fluid moves from the intravascular space into the interstitial spaces

1. ANS: 3 Chapter number and title: 10. Overview of Infusion Therapies Chapter learning objective: 2. Describing the characteristics of common IV solutions Chapter page reference: 156 - 158 Heading: Solutions Used in Infusion Therapy Integrated Processes: Nursing Process: Assessment Client Need: Physiological Integrity: Pharmacological and Parenteral Therapies Cognitive Level: Comprehension [Understanding] Concept: Fluid and Electrolyte Balance Difficulty: Moderate Feedback 1 Hypertonic solutions have concentrations higher than plasma and cause fluid to move from the cells into the intravascular space. 2 Hypotonic solutions have a lower solution concentration than plasma and cause fluid to move from the intravascular space into both the intracellular and interstitial spaces. 3 Isotonic solutions have the same or nearly the same osmolarity as plasma and cause no movement of fluid into or out of cells. Isotonic solutions remain in the extracellular compartment in either the intravascular or interstitial compartments. 4 Hypotonic solutions have a lower solution concentration than plasma and cause fluid to move from the intravascular space into both the intracellular and interstitial spaces.

1. The nurse prepares the patient diagnosed with chronic obstructive pulmonary disease (COPD) for which diagnostic test to monitor acid-base balance? 1. Pulse oximetry 2. Bronchoscopy 3. Sputum studies 4. Arterial blood gases

1. ANS: 4 Chapter number and title: 9, Acid-Base Balance Chapter learning objective: 2. Stating the steps for arterial blood gas interpretation Chapter page reference: 142-143 Heading: Acid-Base Balance Overview: Oxygenation Integrated Processes: Nursing Process: Planning Client Need: Physiological Integrity: Reduction of Risk Potential Cognitive Level: Comprehension [Understanding] Concept: Assessment Difficulty: Easy Feedback 1 Pulse oximetry is a noninvasive test that evaluates the oxygen saturation level of blood. 2 A bronchoscopy provides visualization of internal respiratory structures. 3 Sputum studies can provide specific information about bacterial organisms. 4 Arterial blood gas analysis is done to assess alterations in acid-base balance caused by respiratory disorders, metabolic disorders, or both.

The basic principles that provide the foundation of ethical theory and decision making:

1. Beneficence asks the question of who benefits from the action(s) taken by others. 2. Autonomy examines the individual's personal right to make decisions concerning himself or herself and provides acknowledgment and respect for the individual person's choices. 3. Justice examines who will be vulnerable in any actions taken. Questions of equitability are taken into account with the decision making. 4. Fidelity requires the nurse to be accountable for commitments made to others and to self. It also includes the responsibilities that the nurse has to the profession of nursing by virtue of being a member. 5. Nonmaleficence examines issues related to who may be harmed by actions and how any harm can be minimized or averted if harm (even minimal) is an expected or unforeseen outcome. 6. Veracity is the requirement to tell the truth. It also requires that intentional misleading or deception of the patient to influence decisions is not done. Veracity or truth telling and truthful communication are part of making sure harm is averted or minimized. 7. Confidentiality requires that information is not shared beyond those who have a need to know.

(CC 8.3) The nurse assesses for which *clinical manifestations* in the patient with *dehydration*? (SATA)

1. Increased urine specific gravity -seen in pt w/ decreased renal perfusion, dehydration or in response to ADH secretion 2. Increased serum BUN 3. Increased serum osmolality -seen in hyperglycemia, hypernatremia, & severe dehydration

Questions to ask pregnant women considering normal practice

1. Is pregnancy viewed as a healthy time, a vulnerable time, or an illness? 2. Is the birth process viewed as dangerous? 3. Is birth a public or private experience? 4. What type of help is needed/accepted? 5. What is the expected role of the family?

Colloid

1.) *Albumin*: 5% or 25% 2.) *Dextran* (polysacch.): 40 kDa or 70 kDa 3.) *HES* (synthetic starch): 6% or 10% 4.) *Mannitol* (alcohol sugar): 5% or 25%

The community nurse visits the home of a young child who is home from school because of sudden onset of *nausea, vomiting, and lethargy*. The nurse suspects *acute renal failure*. Which *clinical manifestations* support the nurse's suspicions?

1.) *E*dema 2.) *H*ematuria 3.) *E*levated blood pressure -Pediatric manifestations of *acute renal failure* characteristically begin w/ a healthy child who suddenly becomes ill with nonspecific symptoms that indicate a significant illness or injury. These symptoms may include any combination of the following: *nausea, vomiting, lethargy, edema, gross hematuria, oliguria, & HTN

The school nurse is preparing a class session for high school students on ways to *maintain fluid balance* during the summer months. Which interventions should the nurse recommend (SATA)

1.) *Reduce* the intake of *coffee & tea*. 2.) Drink *more fluids during hot weather*. 3.) Drink *flat cola or ginger ale if vomiting*. -Actions to *prevent fluid volume deficit* during the summer months *include increasing fluid intake, drinking flat cola or ginger ale if vomiting*, and *reducing* the intake of *coffee & tea*.

The nurse is concerned that an *older adult* patient is at risk for developing *acute renal failure*. Which information in the patient's history support the nurse's concern?

1.) Diagnosed with *hypotension* 2.) *Recent aortic valve replacement surgery* 3.) Prescribed *high doses of intravenous antibiotics* -Older adults develop acute renal failure more frequently because of the higher incidence of serious illnesses, HYPO-TN, major surgeries, diagnostic procedures, and treatment with nephrotoxic drugs. Decreased kidney function associated with aging also puts the older patient at risk for kidney failure. HYPO-TN, scheduled for aortic valve replacement surgery, and receiving high doses of intravenous antibiotics increase this patient's risk for developing acute renal failure.

The nurse is providing care to a patient who is exhibiting *clinical manifestations of a fluid and electrolyte deficit*. Based on this data, which health-care provider prescriptions does the nurse prepare to implement? (SATA)

1.) Initiate *hypodermoclysis* -Hypodermoclysis, fluid administered subcutaneously, may be employed as a fluid delivery method, especially among older adults. 2.) Closely monitor patient's *I&O's* -Monitoring patient's intake and output is one of several ways to assess the patient's fluid status. 3.) Initiate *IV therapy* -IV fluids may be ordered for the patient with a fluid volume deficit if replacement oral fluids can't be taken in sufficient quantity.

A patient's serum sodium level is *150 mg/dL*. Based on this data, which interventions should the nurse plan for this patient? (SATA)

1.) Instruct on a *low-sodium diet*. -For an elevated sodium level, the electrolyte will need to be restricted, in the form of a low-sodium diet. 2.) Administer *diuretics* as prescribed. -Diuretics will remove excess fluid being held in the body because of the extra sodium.

10. The nurse is caring for a patient who needs to have a peripheral intravenous catheter placed. When the patient requests to have the intravenous (IV) started in the foot, what is the best response by the nurse? 1. "There is an increased risk of infection if an IV is started in your feet or legs." 2. "There is an increased risk of a clot or inflammation if an IV is started in your foot." 3. "Placing an IV in your foot decreases blood flow to your feet." 4. "Placing an IV in your foot is more painful."

10. ANS: 2 Chapter number and title: 10. Overview of Infusion Therapies Chapter learning objective: 8. Developing a teaching plan for a patient receiving infusion therapy Chapter page reference: 159 - 161 Heading: Types of Intravenous Access Devices/Peripheral Venous Access Integrated Processes: Teaching and Learning Client Need: Physiological Integrity: Reduction of Risk Potential Cognitive Level: Comprehension Concept: Skin Integrity Difficulty: Moderate Feedback 1 There is an increased risk of thrombophlebitis, not infection. 2 Veins in the lower extremities are not recommended for infusion therapy in the adult patient because of the high risk for thrombophlebitis. 3 Placing an IV in the foot increases the risk of thrombophlebitis but does not impact arterial blood flow to the lower extremities. 4 Placing an IV in the foot does not produce any difference in pain compared with the upper extremities.

10. The nurse provides care to a patient whose serum potassium level is 3.2 mEq/L. Which healthcare provider order does the nurse question based on this data? 1. Serum chemistries (basic metabolic panel) every morning 2. Continuous cardiac monitoring 3. 10 mEq KCl (potassium chloride) in 100 mL normal saline slow IVP (intravenous pyelogram) 4. 25 mg spironolactone (Aldactone) by mouth daily

10. ANS: 3 Chapter number and title: 8, Fluid and Electrolyte Management Chapter learning objective: 6. Correlating laboratory data and clinical manifestations related to disorders in: Potassium Balance Chapter page reference: 122 - 123 Heading: Electrolyte Disorders/Potassium/Table 8.7 Common Electrolyte Disturbances Integrated Processes: Nursing Process: Analysis Client Need: Physiological Integrity: Physiological Adaptation Cognitive Level: Analysis [Analyzing] Concept: Fluid and Electrolyte Balance Difficulty: Moderate Feedback 1 A chemistry panel/basic metabolic panel is prescribed to monitor kidney status, electrolyte and acid-base balance, and blood glucose level. This is an appropriate order for the patient who is experiencing hypokalemia. 2 Potassium imbalances cause electrocardiogram changes; therefore, continuous cardiac monitoring is an appropriate prescription for this patient. 3 Although this is an appropriate dose of KCl, it is never given by intravenous pyelogram (IVP). The nurse questions this order. 4 A potassium-sparing diuretic, such as spironolactone, is an appropriate prescription for the patient experiencing hypokalemia.

14. The nurse is caring for a patient with congestive heart failure who is admitted to the medical-surgical unit with acute hypokalemia. Which prescribed medication may have contributed to the patient's current hypokalemic state? 1. Cortisol 2. Oxycodone 3. Flexeril 4. Nonsteroidal anti-inflammatory drugs (NSAIDs)

14. ANS: 1 Chapter number and title: 8, Fluid and Electrolyte Management Chapter learning objective: 6. Correlating laboratory data and clinical manifestations related to disorders in: Potassium balance Chapter page reference: 128 - 129 Heading: Electrolyte Disorders /Hypokalemia Integrated Processes: Nursing Process: Assessment Client Need: Physiological Integrity: Pharmacological and Parenteral Therapies Cognitive Level: Comprehension [Understanding] Concept: Medication Difficulty: Easy Feedback 1 Excess potassium loss through the kidneys is often caused by such medications as corticosteroids, potassium-wasting (loop) diuretics, amphotericin B, and large doses of some antibiotics. Cortisol is a type of corticosteroid and can cause hypokalemia. 2 NSAIDs, narcotics, and muscle relaxers would not bring about potassium loss to cause hypokalemia. 3 NSAIDs, narcotics, and muscle relaxers would not bring about potassium loss to cause hypokalemia. 4 NSAIDs, narcotics, and muscle relaxers would not bring about potassium loss to cause hypokalemia.

10. The nurse is providing care to a patient admitted for a traumatic brain injury who has developed respiratory acidosis. Which action by the nurse is the priority? 1. Monitoring peripheral vascular status 2. Reassuring the patient to decrease anxiety 3. Assessing respiratory rate and depth closely 4. Administering sodium bicarbonate as ordered

10. ANS: 3 Chapter number and title: 9, Acid-Base Balance Chapter learning objective: 6. Explaining nursing considerations related to patients with acid-base disorders Chapter page reference: 149 Heading: Acid-Base Disorders/Respiratory Acidosis/Nursing Management/ Nursing Interventions Integrated Processes: Nursing Process: Implementation Client Need: Physiological Integrity: Physiological Adaptation Cognitive Level: Analysis [Analyzing] Concept: Assessment Difficulty: Moderate Feedback 1 This nursing action is more appropriate for the patient diagnosed with metabolic, not respiratory, acidosis. 2 Anxiety is more common for the patient diagnosed with respiratory alkalosis, not respiratory acidosis. 3 Nursing interventions for patients with respiratory acidosis include maintaining the patient's airway, monitoring arterial blood gas levels, monitoring vital signs (especially respiratory rate and depth), administering supplemental oxygen, assisting with intubation if necessary, monitoring potassium levels, administering sedatives cautiously, and providing patient reassurance and teaching as needed. 4 Sodium bicarbonate is prescribed for metabolic acidosis, not respiratory acidosis.

11. The nurse is caring for a patient who is to have a peripherally inserted central catheter (PICC) line inserted tomorrow. Because the patient's current peripheral access line is infiltrated and needs to be restarted, which site would the nurse avoid using? 1. Radial vein 2. Cephalic vein 3. Median cubital vein 4. Dorsal metacarpal veins

11. ANS: 3 Chapter number and title: 10, Overview of Infusion Therapies Chapter learning objective: 3. Comparing peripheral and central venous access, including indications, access devices, and potential complications Chapter page reference: 161 - 164 Heading: Types of Intravenous Access Devices /Central Venous Access Integrated Processes: Nursing Process: Planning Client Need: Physiological Integrity: Pharmacological and Parenteral Therapies Cognitive Level: Application [Applying] Concept: Fluid and Electrolyte Balance Difficulty: Moderate Feedback 1 The radial vein can be used because the veins used for peripherally inserted central catheter (PICC) insertion are usually the larger veins in the upper extremities. If infusion therapy using short peripheral therapy with repeated venipunctures has compromised these veins, PICC placement is much more difficult. 2 The cephalic vein can be used because the veins used for PICC insertion are usually the larger veins in the upper extremities. If infusion therapy using short peripheral therapy with repeated venipunctures has compromised these veins, PICC placement is much more difficult. 3 The median cubital vein, a larger vein in the upper extremity, is often used for PICC lines, so the nurse should attempt to avoid this site to maintain it for the central line. 4 The dorsal metacarpal veins can be used because the veins used for PICC insertion are usually the larger veins in the upper extremities.

11. In reviewing laboratory results for a patient presenting to the Emergency Department with changes in level of consciousness, the nurse correlates which value as placing the patient at greatest risk for seizures? 1. Serum calcium of 12 mg/dL 2. Serum potassium of 3.0 mEq/L 3. Serum sodium of 135 mEq/L 4. Serum magnesium of 2.6 mEq/L

11. ANS: 3 Chapter number and title: 8, Fluid and Electrolyte Management Chapter learning objective: 6. Correlating laboratory data and clinical manifestations related to disorders in sodium balance Chapter page reference: 124 Heading: Electrolyte Disorders /Hyponatremia: Clinical Manifestations Integrated Processes: Nursing Process: Assessment Client Need: Physiological Integrity: Physiological Adaptation Cognitive Level: Analysis [Analyzing] Concept: Fluid and Electrolyte Balance Difficulty: Difficult Feedback 1 Hypercalcemia presents with anorexia, nausea, vomiting, lethargy, non-specific joint and muscle aches, and confusion. Seizure activity may be observed in patients with hypocalcemia. 2 Patients with hypokalemia may experience weakness, lethargy, hyporeflexia, nausea/vomiting, constipation, abdominal cramping and electrocardiographic (ECG) changes (ST depression). 3 Neurological changes such as confusion, muscle twitching, lethargy, and seizures can indicate low sodium levels, especially in older adults. 4 Patients with hypermagnesemia may present with hypotension, bradycardia, drowsiness, lethargy, muscle weakness, and loss of deep tendon reflexes.

11. The nurse is providing care to a patient who is admitted to the hospital with sudden, severe abdominal pain. Which arterial blood gas supports the patient's current diagnosis of respiratory alkalosis? 1. pH is 7.35 and PaO2 is 88. 2. pH is 7.30 and HCO3 is 30. 3. pH is 7.47 and PaCO2 is 25. 4. pH is 7.33 and PaCO2 is 36.

11. ANS: 3 Chapter number and title: 9, Acid-Base Balance Chapter learning objective: 2. Stating the steps for arterial blood gas interpretation Chapter page reference: 149 Heading: Acid-Base Disorders/Respiratory Alkalosis/Arterial Blood Gas Results Integrated Processes: Nursing Process: Assessment Client Need: Physiological Integrity: Reduction of Risk Potential Cognitive Level: Analysis Concept: Assessment Difficulty: Difficult Feedback 1 This arterial blood gas value indicates a normal pH and PaO2, not respiratory alkalosis. 2 Respiratory alkalosis includes a high pH and decreased PaCO2. This PaCO2 is low. 3 Acute pain usually causes hyperventilation, which causes the CO2 to drop and the patient to experience respiratory alkalosis. The pH would denote alkalosis and would be higher than 7.45. HCO3 would trend downward as the kidneys begin to compensate for the alkalosis by excreting HCO3. The PaO2 is likely to be normal unless the patient has been hyperventilating for a long time and is beginning to tire. 4 The pH is normal; the PaCO2 is low and does not correlate with respiratory alkalosis.

12. The nurse performs an hourly check of a patient's intravenous site and notes erythema. The patient denies pain or discomfort. Based on this data, what should the nurse document in the patient's medical record? 1. Grade 1 phlebitis 2. Grade 2 phlebitis 3. Grade 3 phlebitis 4. Grade 4 phlebitis

12. ANS: 1 Chapter number and title: 10, Overview of Infusion Therapies Chapter learning objective: 5. Describing the potential complications of infusion therapy and strategies to prevent these complications Chapter page reference: 165 Heading: Nursing Management of Infusion Therapy/Phlebitis and Infiltration/Table 10.4 - Infusion Nurses Society Phlebitis Scale Integrated Processes: Nursing Process: Assessment Client Need: Physiological Integrity: Reduction of Risk Potential Cognitive Level: Knowledge [Remembering] Concept: Skin Integrity Difficulty: Easy Feedback 1 Grade 1 phlebitis is documented when erythema is noted at the access site with or without pain. 2 Pain at the access site with erythema and/or edema is documented as grade 2 phlebitis. 3 Pain at the access site with erythema, streak formation, and/or palpable cord is documented as grade 3 phlebitis. 4 Pain at the access site with erythema, streak formation, palpable venous cord greater than 1 in. in length, and/or purulent drainage is documented as grade 4 phlebitis.

12. The nurse is providing care to a patient who seeks emergency treatment for headache and nausea. The patient works in a mill without air conditioning. The patient states, "I drink water several times each day, but I seem to sweat more than I am able to replace." Which suggestions does the nurse provide to this patient? 1. Drink juices and carbonated sodas. 2. Eat something salty when drinking water. 3. Eat something sweet when drinking water. 4. Double the amount of water being ingested.

12. ANS: 2 Chapter number and title: 8, Fluid and Electrolyte Management Chapter learning objective: 5. Describing the pathophysiology, clinical presentations, and management of dehydration, hypovolemia, and hypervolemia Chapter page reference: 124 - 125 Heading: Electrolyte Disorders/Hyponatremia: Medical Management Integrated Processes: Teaching and Learning Client Need: Physiological Integrity: Physiological Adaptation Cognitive Level: Application [Applying] Concept: Nutrition Difficulty: Moderate Feedback 1 Juices and carbonated sodas will not help to replace the loss of sodium. 2 Both salt and water are lost through sweating. When only water is replaced, the individual is at risk for salt depletion. Clinical manifestations include fatigue, weakness, headache, and gastrointestinal symptoms such as loss of appetite and nausea. The patient should be instructed to eat something salty when drinking water to help replace the loss of sodium. 3 Eating something sweet will not help replace the loss of sodium. 4 Doubling the amount of water being ingested could lead to hyponatremia and further manifestations.

12. The patient is admitted to the emergency department (ED) with symptoms of a panic attack. Based on this data, the nurse recognizes the patient is at risk for which acid-base disorder? 1. Metabolic alkalosis 2. Metabolic acidosis 3. Respiratory acidosis 4. Respiratory alkalosis

12. ANS: 4 Chapter number and title: 9, Acid-Base Balance Chapter learning objective: 4. Comparing and contrasting major acid-base disorders Chapter page reference: 149 Heading: Acid-Base Disorders/Respiratory Alkalosis/Box 9.4 Causes of Respiratory Alkalosis Integrated Processes: Nursing Process: Assessment Client Need: Physiological Integrity: Reduction for Risk Potential Cognitive Level: Comprehension [Understanding] Concept: pH Regulation Difficulty: Easy Feedback 1 Metabolic alkalosis caused by loss of hydrogen ions usually occurs because of vomiting or gastric suction. Gastric secretions are highly acidic (pH 1-3). 2 Metabolic acidosis is caused by an increased accumulation of metabolic acids that rise in proportion to a decrease in bicarbonate and result in decreased arterial pH. 3 Anxiety and panic attacks will lead to hyperventilation, not hypoventilation. Respiratory acidosis results from altered ventilation leading to CO2 retention secondary to hypoventilation 4 Anxiety disorders increase the risk for the acid-base imbalance respiratory alkalosis because of hyperventilation that leads to loss of carbon dioxide.

13. The nurse is assessing an intravenous (IV) insertion site noting redness, warmth, and mild swelling. The patient reports a burning pain along the course of the vein during medication administration. Which term should the nurse use when documenting these findings in the medical record? 1. Phlebitis 2. Infiltration 3. Extravasation 4. Occlusion

13. ANS: 1 Chapter number and title: 10, Overview of Infusion Therapies Chapter learning objective: 5. Describing the potential complications of infusion therapy and strategies to prevent these complications Chapter page reference: 165 Heading: Nursing Management of Infusion Therapy/Monitoring and Preventing Complications Integrated Processes: Communication and Documentation Client Need: Physiological Integrity: Reduction of Risk Potential Cognitive Level: Comprehension [Understanding] Concept: Skin Integrity Difficulty: Moderate Feedback 1 Redness, warmth, edema, and pain that runs along the course of the vein characterize phlebitis. 2 An infiltrate is defined as fluid entering the tissues, resulting in swelling, coolness, pallor, and discomfort at the site. 3 Extravasation includes a vesicant drug (one that causes blistering when in the tissues but not in the vascular system), so this is not an extravasation. 4 An occlusion develops when the there is no flow through the IV catheter, and the site is not warm and red.

13. A nurse is reviewing the serum chemistry results on a patient who has a nasogastric tube to low intermittent suction secondary to a gunshot to the abdomen 2 days ago. Which electrolyte value does the nurse correlate to the NG suctioning? 1. Serum chloride of 90 mEq/L 2. Serum sodium of 148 mEq/L 3. Serum potassium of 5.2 mEq/L 4. Serum calcium of 11.3 mg/dL

13. ANS: 1 Chapter number and title: 8, Fluid and Electrolyte Management Chapter learning objective: 6. Correlating laboratory data and clinical manifestations related to disorders in chloride. Chapter page reference: 127 Heading: Electrolyte Disorders /Hypochloremia Integrated Processes: Nursing Process: Analysis Client Need: Physiological Integrity: Physiological Adaptation Cognitive Level: Analysis Concept: Fluid and Electrolyte Balance Difficulty: Difficult Feedback 1 Serum chloride decreases in patients with severe vomiting, burns, chronic respiratory acidosis, nasogastric suctioning, metabolic alkalosis, and Addison's disease (adrenal cortex insufficiency). The normal range for serum chloride is 97 to 107 mEq/L. 2 Serum sodium does not increase secondary to nasogastric suctioning. The normal range for serum sodium is 135 to 145 mEq/L 3 Serum potassium usually decreases with nasogastric suctioning. This value is high because the normal range of serum potassium is 3.5 to 5.0 mEq/L. 4 Serum calcium levels are not impacted by nasogastric suctioning. The normal range for serum calcium is 8.2 to 10.2 mg/dL.

13. The nurse completes discharge teaching for a patient with an anxiety disorder. Which patient statement indicates correct understanding of information related to respiratory alkalosis? 1. "I will eat more bananas at breakfast." 2. "I will practice stress reduction." 3. "I will not take antacids when I have heartburn." 4. "I will breathe faster when I am feeling anxious."

13. ANS: 2 Chapter number and title: 9, Acid-Base Balance Chapter learning objective: 6. Explaining nursing considerations related to patients with acid-base disorders Chapter page reference: 149 Heading: Acid-Base Disorders/Respiratory Alkalosis/Box 9.4 Causes of Respiratory Alkalosis Integrated Processes: Teaching and Learning Client Need: Physiological Integrity: Physiological Adaptation Cognitive Level: Analysis [Analyzing] Concept: pH Regulation Difficulty: Moderate Feedback 1 Eating bananas is more appropriate for the patient at risk for metabolic alkalosis who is on diuretics. 2 The patient understands that reducing anxiety can reduce hyperventilation and respiratory alkalosis. Practicing stress reduction can help the patient develop alternative strategies for dealing with anxiety. 3 Taking too many antacids is associated with metabolic alkalosis. 4 Breathing faster will increase hyperventilation and exacerbate respiratory alkalosis.

14. The nurse is reviewing the healthcare provider's orders for a patient who is diagnosed with respiratory alkalosis. Which order is appropriate for this patient's care needs? 1. Monitor arterial blood gas results. 2. Administer oxygen via face mask. 3. Restrict fluids to 2 L per day. 4. Infuse 1 ampule of sodium bicarbonate.

14. ANS: 1 Chapter number and title: 9, Acid-Base Balance Chapter learning objective: 6. Explaining nursing considerations related to patients with acid-base disorders Chapter page reference: 149-150 Heading: Acid-Base Disorders/Respiratory Alkalosis/Treatment Integrated Processes: Nursing Process: Planning Client Need: Physiological Integrity: Reduction of Risk Potential Cognitive Level: Comprehension [Understanding] Concept: pH Regulation Difficulty: Easy Feedback 1 Management of respiratory alkalosis focuses on correcting the imbalance and treating the underlying cause. Arterial blood gases must be ordered before beginning medication or oxygen therapy. 2 Oxygen is not anticipated when providing care to a patient experiencing respiratory alkalosis. 3 A fluid restriction is not required in the treatment of respiratory alkalosis. 4 Sodium bicarbonate is used in the treatment of metabolic acidosis.

14. The nurse correlates the administration of which blood component to the patient with acute blood loss? 1. Platelets 2. Albumin 3. Fresh frozen plasma 4. Packed red blood cells

14. ANS: 4 Chapter number and title: 10, Overview of Infusion Therapies Chapter learning objective: 6. Explaining the procedure for safely administering blood products Chapter page reference: 169 - 171 Heading: Nursing Management of Infusion Therapy/Administration of Blood Products/Table 10.5 - Indicators for Blood Component Transfusion Integrated Processes: Nursing Process: Planning Client Need: Physiological Integrity: Pharmacological and Parenteral Therapies Cognitive Level: Comprehension [Understanding] Concept: Medication Difficulty: Moderate Feedback 1 Platelets are administered for patients who are bleeding as a result of thrombocytopenia or platelet abnormalities. 2 Albumin is administered for volume expansion when crystalloid solutions are not adequate. 3 Fresh frozen plasma is anticipated for a patient with a deficiency of plasma coagulation factors. 4 Packed red blood cells are anticipated for a patient with acute or chronic blood loss and for patients diagnosed with anemia.

15. The nurse is providing care to a patient who is intubated and receiving mechanical ventilation after a motor vehicle accident. The patient is fighting the ventilator and attempting to remove the endotracheal tube. Which nursing action decreases the patient's risk for developing respiratory alkalosis? 1. Apply wrist restraints. 2. Administer a prescribed sedative. 3. Teach the patient to take slow, deep breaths. 4. Recommend to the healthcare provider to increase the tidal volume on the ventilator.

15. ANS: 2 Chapter number and title: 9, Acid-Base Balance Chapter learning objective: 6. Explaining nursing considerations related to patients with acid-base disorders Chapter page reference: 149-150 Heading: Acid-Base Disorders/Respiratory Alkalosis/Treatment Integrated Processes: Nursing Process: Implementation Client Need: Physiological Integrity: Reduction of Risk Potential Cognitive Level: Application [Applying] Concept: pH Regulation Difficulty: Difficult Feedback 1 Applying wrist restraints to a patient who is demonstrating anxiety with an endotracheal tube might exacerbate the patient's condition. 2 For a patient being mechanically ventilated, the only way to reduce rapid respirations might be to provide a sedative. 3 The patient is being mechanically ventilated, which means there is a problem with maintaining the airway. The patient will not be able to take slow, deep breaths at this time. 4 Increasing the rate or tidal volume of the ventilator increases the risk of respiratory alkalosis.

15. Which component should the nurse anticipate will be prescribed for a patient with an elevated prothrombin time (PT) and international normalized ratio (INR) who is at an increased risk for bleeding? 1. Platelets 2. Albumin 3. Fresh frozen plasma 4. Packed red blood cells

15. ANS: 3 Chapter number and title: 10, Overview of Infusion Therapies Chapter learning objective: 6. Explaining the procedure for safely administering blood products Chapter page reference: 169 - 171 Heading: Nursing Management of Infusion Therapy/Administration of Blood Products/Table 10-5 - Indications for Blood Component Transfusion Integrated Processes: Nursing Process: Planning Client Need: Physiological Integrity: Pharmacological and Parenteral Therapies Cognitive Level: Comprehension [Understanding] Concept: Medication Difficulty: Moderate Feedback 1 Platelets are administered for patients who are bleeding as a result of thrombocytopenia or platelet abnormalities. 2 Albumin is administered for volume expansion when crystalloid solutions are not adequate. 3 Fresh frozen plasma is anticipated for a patient with a deficiency of plasma coagulation factors. 4 Packed red blood cells are anticipated for a patient with acute or chronic blood loss and for patients diagnosed with anemia.

15. A patient is prescribed 20 mEq of potassium chloride because of excessive vomiting. The nurse includes which information in explaining the rationale for this medication? 1. It controls and regulates water balance in the body. 2. It is used in the body to synthesize ingested protein. 3. It is vital in regulating muscle contraction and relaxation. 4. It is needed to maintain skeletal, cardiac, and neuromuscular activity.

15. ANS: 4 Chapter number and title: 8, Fluid and Electrolyte Management Chapter learning objective: 7. Explaining nursing considerations related to patients with fluid and electrolyte disorders Chapter page reference: 130 Heading: Electrolyte Disorders /Hypokalemia: Nursing Management Integrated Processes: Teaching and Learning Client Need: Physiological Integrity: Pharmacological and Parental Therapies Cognitive Level: Comprehension [Understanding] Concept: Medication Difficulty: Moderate Feedback 1 Sodium controls and regulates water balance in the body. 2 Magnesium is used in the body to synthesize ingested protein. 3 Calcium is vital in regulating muscle contraction and relaxation. 4 Potassium is the major cation in intracellular fluids, with only a small amount found in plasma and interstitial fluid. Potassium is a vital electrolyte for skeletal, cardiac, and smooth muscle activity.

6. Which intravenous (IV) fluid should the nurse prepare when a patient requires a hypotonic solution? 1. 0.9% Normal saline 2. 5% Dextrose in water 3. 0.33% Sodium chloride 4. 5% Dextrose in lactated Ringer's

6. ANS: 3 Chapter number and title: 10, Overview of Infusion Therapies Chapter learning objective: 2. Describing the characteristics of common IV solutions Chapter page reference: 156 - 158 Heading: Solutions Used in Infusion Therapy/Table 10.1 - Osmolarity of Intravenous Fluids Integrated Processes: Nursing Process: Implementation Client Need: Physiological Integrity: Pharmacological and Parenteral Therapies Cognitive Level: Comprehension [Understanding] Concept: Fluid and Electrolyte Balance Difficulty: Easy Feedback 1 0.9% Normal saline is an isotonic solution. 2 5% Dextrose in water is a hypotonic solution. 3 0.33% Sodium chloride is a hypotonic solution. 4 5% Dextrose in lactated Ringer's is a hypertonic solution.

16. The nurse is caring for a patient with congestive heart failure who is admitted to the medical-surgical unit with acute hypokalemia. Which prescribed medication may have contributed to the patient's current hypokalemic state? 1. Cortisol 2. Oxycodone 3. Flexeril 4. Nonsteroidal anti-inflammatory drugs (NSAIDs)

16. ANS: 1 Chapter number and title: 8, Fluid and Electrolyte Management Chapter learning objective: 6. Correlating laboratory data and clinical manifestations related to disorders in: Potassium balance Chapter page reference: 8-40-8-41 Heading: Hypokalemia Integrated Processes: Nursing Process: Assessment Client Need: Physiological Integrity: Pharmacological and Parental Therapies Cognitive Level: Comprehension [Understanding] Concept: Medication Difficulty: Easy Feedback 1 Excess potassium loss through the kidneys is often caused by such medications as corticosteroids, potassium-wasting (loop) diuretics, amphotericin B, and large doses of some antibiotics. Cortisol is a type of corticosteroid and can cause hypokalemia. 2 NSAIDs, narcotics, and muscle relaxers would not bring about potassium loss to cause hypokalemia. 3 NSAIDs, narcotics, and muscle relaxers would not bring about potassium loss to cause hypokalemia. 4 NSAIDs, narcotics, and muscle relaxers would not bring about potassium loss to cause hypokalemia.

16. The nurse is providing care for an adult patient who is admitted to the emergency department (ED) after losing consciousness. The patient has been fasting and currently has ketones in the urine. The nurse monitors for which acid-base imbalance? 1. Metabolic acidosis 2. Metabolic alkalosis 3. Respiratory acidosis 4. Respiratory alkalosis

16. ANS: 1 Chapter number and title: 9, Acid-Base Balance Chapter learning objective: 4. Comparing and contrasting major acid-base disorders Chapter page reference: 150 Heading: Acid-Base Disorders/Metabolic Acidosis Integrated Processes: Nursing Process: Analysis Client Need: Physiological Integrity: Physiological Adaptation Cognitive Level: Comprehension [Understanding] Concept: pH Regulation Difficulty: Easy Feedback 1 The patient who is fasting is at risk for metabolic acidosis. The body recognizes fasting as starvation and begins to metabolize its own proteins into ketones, which are metabolic acid. Metabolic acidosis is an acid-base imbalance caused by an increased accumulation of metabolic acids that rise in proportion to a decrease in bicarbonate and result in decreased arterial pH. 2 Metabolic alkalosis is an acid-base imbalance caused by an increased loss of acid. Most instances of this increased loss of acid are through the gastrointestinal tract, usually via vomiting or nasogastric suctioning, or through renal excretion. This patient is not at risk for metabolic alkalosis. 3 Respiratory acidosis results from altered ventilation leading to CO2 retention. This patient is not at risk for this acid-base disorder. 4 Respiratory alkalosis an acid-base imbalance caused by an increase in the rate of alveolar ventilation and is often the result of anxiety. This patient is not at risk for respiratory alkalosis.

16. Which component should the nurse anticipate will be prescribed for a patient who is not responding to crystalloids for volume expansion? 1. Platelets 2. Albumin 3. Fresh frozen plasma 4. Packed red blood cells

16. ANS: 2 Chapter number and title: 10, Overview of Infusion Therapies Chapter learning objective: 6. Explaining the procedure for safely administering blood products Chapter page reference: 169 - 171 Heading: Nursing Management of Infusion Therapy/Administration of Blood Products/Table 10.5 Indications for Blood Component Transfusion Integrated Processes: Nursing Process: Planning Client Need: Physiological Integrity: Pharmacological and Parenteral Therapies Cognitive Level: Comprehension [Understanding] Concept: Medication Difficulty: Moderate Feedback 1 Platelets are administered for patients who are bleeding as a result of thrombocytopenia or platelet abnormalities. 2 Albumin is administered for volume expansion when crystalloid solutions are not adequate. 3 Fresh frozen plasma is anticipated for a patient with a deficiency of plasma coagulation factors. 4 Packed red blood cells are anticipated for a patient with acute or chronic blood loss and for patients diagnosed with anemia.

17. Which component should the nurse anticipate will be prescribed for a patient with severe thrombocytopenia? 1. Platelets 2. Albumin 3. Fresh frozen plasma 4. Packed red blood cells

17. ANS: 1 Chapter number and title: 10, Overview of Infusion Therapies Chapter learning objective: 6. Explaining the procedure for safely administering blood products Chapter page reference: 169 - 171 Heading: Indications for Blood Component Transfusion /Administration of Blood Products/Table 10.5 Indications for Blood Component Transfusion Integrated Processes: Nursing Process: Planning Client Need: Physiological Integrity: Pharmacological and Parenteral Therapies Cognitive Level: Comprehension [Understanding] Concept: Medication Difficulty: Moderate Feedback 1 Platelets are administered for patients who are bleeding as a result of thrombocytopenia or platelet abnormalities. 2 Albumin is administered for volume expansion when crystalloid solutions are not adequate. 3 Fresh frozen plasma is anticipated for a patient with a deficiency of plasma coagulation factors. 4 Packed red blood cells are anticipated for a patient with acute or chronic blood loss and for patients diagnosed with anemia.

17. In reviewing a patient's laboratory results before administering digoxin, which laboratory result places the patient at greatest risk for digoxin toxicity? 1. Serum sodium 156 mEq/L 2. Serum potassium 3.0 mEq/L 3. Serum phosphorus of 1.5 mg/dL 4. Serum creatinine of 1.6 mg/dL

17. ANS: 2 Chapter number and title: 8, Fluid and Electrolyte Management Chapter learning objective: 7. Explaining nursing considerations related to patients with fluid and electrolyte disorders Chapter page reference: 130 Heading: Electrolyte Disorders /Hypokalemia: Nursing Management Integrated Processes: Nursing Process: Assessment Client Need: Physiological Integrity: Physiological Adaptation Cognitive Level: Analysis [Analyzing] Concept: Fluid and Electrolyte Balance Difficulty: Difficult Feedback 1 Hypernatremia does not increase the risk of digoxin toxicity. 2 Hypokalemia can potentiate the effects of digitalis by increasing blood levels of digoxin, leading to digoxin toxicity. Symptoms of digoxin toxicity include loss of appetite, nausea, vomiting, cardiac dysrhythmias, and visual disturbances. 3 Hypophosphatemia does increase the risk of digoxin toxicity. 4 Elevations of blood urea nitrogen (BUN) and creatinine indicate decreased renal function, which can result in decreased potassium excretion and hyperkalemia. There is no risk of digoxin toxicity with elevated creatinine levels.

Which action does the nurse take first when preparing to do discharge teaching for the 73-year-old client who is being discharged after prostate surgery? A. Ask what the client knows about self-care after prostate surgery. B. Have family members available during the teaching. C. Obtain written information about postdischarge care. D. Plan to teach early in the morning after the client has eaten.

A

Which important aspect of coordinating care within the interdisciplinary team is facilitated by use of the "SBAR" and "PACE" procedures? A. Communication B. Implementation C. Policymaking D. Protocol development

A

17. The nurse assesses a patient admitted to the hospital for treatment of pneumonia. The patient's pulse oximetry reads 77%. After repositioning the patient and increasing oxygen per the healthcare provider's order, there is no change in the pulse oximeter reading. Which is the priority action for the nurse? 1. Reassessing the patient in 1 hour 2. Obtaining an arterial blood gas analysis 3. Consulting the unit respiratory therapist 4. Documenting the findings in the medical record

17. ANS: 2 Chapter number and title: 9, Acid-Base Balance Chapter learning objective: 6. Explaining nursing considerations related to patients with acid-base disorders Chapter page reference: 150 Heading: Acid-Base Disorders/Metabolic Acidosis: Safety Alert Integrated Processes: Nursing Process: Implementation Client Need: Physiological Integrity: Reduction of Risk Potential Cognitive Level: Application [Applying] Concept: Oxygenation Difficulty: Moderate Feedback 1 This patient is experiencing hypoxia, which can negatively impact outcomes if no action is implemented. 2 Pulse oximetry is considered accurate for oxygen saturations greater than 80%. Arterial blood gas analysis is recommended for oxygen saturations less than 80%. Therefore, this is the best action by the nurse. 3 Although this is an appropriate action by the nurse, this is not the best action as the patient requires treatment for the hypoxia now. 4 Although documenting the findings in the patient's medical record is important, this is not the priority at this time.

18. The nurse is analyzing the patient's arterial blood gas report after a cardiac arrest. Based on a pH of 7.15, the nurse monitors for which clinical manifestation? 1. Dysrhythmias 2. Tetany 3. Convulsions 4. Muscle twitching

18. ANS: 1 Chapter number and title: 9, Acid-Base Balance Chapter learning objective: 1. Describing the significance of acid-base balance for normal function Chapter page reference: 150 Heading: Acid-Base Disorders/Metabolic Acidosis/Complications/ Box 9.7 - Clinical Manifestations of Metabolic Acidosis Integrated Processes: Nursing Process: Analysis Client Need: Physiological Integrity: Reduction of Risk Potential Cognitive Level: Analysis Concept: Assessment Difficulty: Moderate Feedback 1 Metabolic acidosis results from an increased accumulation of metabolic acids that rise in proportion to bicarbonate and leads to a decreased arterial pH. If the pH falls below 7.0, cardiac dysrhythmia can occur. 2 Tetany is observed in patients with metabolic alkalosis. 3 Seizures are observed in patients with metabolic alkalosis. 4 Muscle twitching and cramping are observed in patients with metabolic alkalosis.

18. The nurse provides care to several patients on a medical-surgical unit. Which situation requires the nurse to follow up with the healthcare provider before administering the prescribed treatment? 1. The patient who requires volume expansion and who is prescribed albumin 2. The patient diagnosed with thrombocytopenia who is prescribed fresh frozen plasma 3. The patient diagnosed with symptomatic anemia who is prescribed packed red blood cells 4. The patient diagnosed with neutropenia and infection that is resistant to antibiotics who is prescribed granulocytes

18. ANS: 2 Chapter number and title: 10, Overview of Infusion Therapies Chapter learning objective: 5. Describing the potential complications of infusion therapy and strategies to prevent these complications Chapter page reference: 169 - 171 Heading: Indications for Blood Component Transfusion/Administration of Blood Products/Table 10.5 Indications for Blood Component Transfusion Integrated Processes: Nursing Process: Analysis Client Need: Physiological Integrity: Physiological Adaptation Cognitive Level: Analysis [Analyzing] Concept: Assessment Difficulty: Moderate Feedback 1 Volume expansion when crystalloid solutions are not adequate necessitates an albumin transfusion. 2 Bleeding caused by thrombocytopenia or platelet abnormalities is treated with a platelet transfusion, not a fresh frozen plasma transfusion. 3 Symptomatic anemia along with acute and chronic blood loss is treated with a packed red blood cell transfusion. 4 Neutropenia with infection unresponsive to appropriate antibiotics is treated with a granulocyte transfusion.

18. The nurse is providing care to a patient who is prescribed furosemide for treatment of congestive heart failure (CHF). The patient's serum potassium level is 3.4 mEq/L. Which food should the nurse encourage the patient to eat based on this data? 1. Peas 2. Iced tea 3. Bananas 4. Baked fish

18. ANS: 3 Chapter number and title: 8, Fluid and Electrolyte Management Chapter learning objective: 7. Explaining nursing considerations related to patients with fluid and electrolyte disorders Chapter page reference: 130 Heading: Electrolyte Disorders/Hypokalemia// Nursing Management Integrated Processes: Nursing Process: Implementation Client Need: Physiological Integrity: Reduction of Risk Potential Cognitive Level: Application [Applying] Concept: Nutrition Difficulty: Moderate Feedback 1 Peas are not a potassium-rich food, which is currently needed based on the patient's serum potassium level. 2 Iced tea is not a potassium-rich food, which is currently needed based on the patient's serum potassium level. 3 A potassium level of 3.4 is low, so the client should be encouraged to consume potassium-rich foods. Of the foods listed, the highest in potassium is banana. 4 Baked fish is not a potassium-rich food, which is currently needed based on the patient's serum potassium level.

19. The nurse is caring for a patient admitted with hypertension and chronic renal failure who receives hemodialysis three times per week. The nurse is assessing the patient's diet and notes the use of salt substitutes. When teaching the patient to avoid salt substitute, which rationale supports this teaching point? 1. They can potentiate hyperkalemia. 2. They may cause the client to retain fluid. 3. They may interfere with the hemodialysis. 4. They may interact with the client's antihypertensive medications.

19. ANS: 1 Chapter number and title: 8, Fluid and Electrolyte Management Chapter learning objective: 5. Explaining nursing considerations related to patients with fluid and electrolyte disorders Chapter page reference: 130 Heading: Electrolyte Disorders /Hyperkalemia Integrated Processes: Teaching and Learning Client Need: Physiological Integrity: Basic Care and Comfort Cognitive Level: Application [Applying] Concept: Nutrition Difficulty: Moderate Feedback 1 Many salt substitutes use potassium chloride. Potassium intake is carefully regulated in patients with renal failure, and the use of salt substitutes will worsen hyperkalemia. 2 Increases in weight do need to be reported to the healthcare provider as a possible indication of fluid volume excess, but this is not the reason why salt substitute is to be avoided. 3 Salt substitutes do not impact hemodialysis. 4 The control of hypertension is essential in the management of a client with kidney disease, but salt substitute is not known to interact with antihypertensive medications.

Which nursing activity is best for the charge nurse on the medical-surgical unit to delegate to staff members who are unlicensed assistive personnel (UAPs)? A. Feeding a client whose hands are affected by rheumatoid arthritis B. Increasing the oxygen flow rate for a client who has wheezes C. Positioning a client who has just returned from hip surgery D. Taking vital signs for a client who is having acute chest pain

A

19. The nurse administers a blood transfusion to a patient who begins to experience itching and shortness of breath. The nurse notes the patient has hives and facial flushing. Which is the priority action by the nurse? 1. Monitoring vital signs 2. Stopping the transfusion 3. Elevating the head of the bed 4. Administering prescribed antihistamine

19. ANS: 2 Chapter number and title: 10, Overview of Infusion Therapies Chapter learning objective: 6. Explaining the procedure for safely administering blood products Chapter page reference: 171 Heading: Nursing Management of Infusion Therapy/Administration of Blood Products/Table 10.7 Types of Transfusion Reactions Integrated Processes: Nursing Process: Analysis Client Need: Physiological Integrity: Reduction of Risk Potential Cognitive Level: Analysis [Analyzing] Concept: Safety Difficulty: Moderate Feedback 1 Although monitoring the patient's vital signs is an appropriate nursing action, this is not the priority action by the nurse. When in distress the priority is not to assess. 2 The priority intervention in this situation is to stop the transfusion and notify the healthcare provider. 3 Although this may be an appropriate intervention given the patient is experiencing dyspnea, this is not the priority action. The nurse stops the transfusion because the patient is likely experiencing an allergic reaction. 4 Although administering the prescribed antihistamine is an appropriate intervention, this is not the priority. The priority action is to stop the transfusion and notify the healthcare provider.

19. The nurse assesses a patient who is admitted for the treatment of anorexia nervosa based on a 15-pound weight loss over the last month. When assessing the patient, the nurse monitors for which clinical manifestation? 1. Dyspnea 2. Dry mouth 3. Dysrhythmia 4. Disorientation

19. ANS: 3 Chapter number and title: 9, Acid-Base Balance Chapter learning objective: 4. Comparing and contrasting major acid-base disorders Chapter page reference: 151 Heading: Acid-Base Disorders/Metabolic Acidosis/Complications Integrated Processes: Nursing Process: Assessment Client Need: Physiological Integrity: Physiological Adaptation Cognitive Level: Analysis [Analyzing] Concept: Assessment Difficulty: Difficult Feedback 1 The nurse monitors a patient for dyspnea if the patient is at risk for respiratory acidosis, not metabolic acidosis. 2 Dry mouth is anticipated for the patient at risk for respiratory alkalosis secondary to hyperventilation, not metabolic acidosis. 3 The patient who is admitted for treatment of anorexia nervosa is likely to be experiencing metabolic acidosis caused by starvation. The nurse closely monitors this patient for dysrhythmias because they can occur as a result of metabolic acidosis. Other causes of metabolic acidosis include severe or chronic diarrhea, diabetic ketoacidosis, trauma and shock, and renal failure. 4 Disorientation is expected for the patient who is at risk for developing metabolic alkalosis, not acidosis.

2. The results of a patient's arterial blood gas sample indicate an oxygen level of 72 mm Hg. Based on this result the nurse needs to closely assess which parameter when providing care to this patient? 1. Perfusion 2. Level of consciousness 3. Urine output 4. Fluid and electrolytes

2. ANS: 2 Chapter number and title: 9, Acid-Base Balance Chapter learning objective: 6. Explaining nursing considerations related to patients with acid-base disorders Chapter page reference: 142 Heading: Acid-Base Balance Overview/Table 9.1 - Oxygenation Based Gas Values Integrated Processes: Nursing Process: Assessment Client Need: Physiological Integrity: Reduction of Risk Potential Cognitive Level: Analysis [Analyzing] Concept: Assessment Difficulty: Moderate Feedback 1 Perfusion is affected by a reduction in circulating fluids. 2 An oxygen level of less than 75 mm Hg can be due to hypoventilation. This drop in oxygen will change the patient's level of responsiveness. 3 Urine output is not impacted by oxygenation status and is more related to renal perfusion. 4 With a fluid and electrolyte imbalance, there is another disorder affecting acid-base balance. This might not be affected by oxygen level.

2. The nurse is reviewing new healthcare provider orders on a patient admitted for treatment of severe dehydration. The patient's serum osmolality is 300 mOsm/kg. It is a priority for the nurse to follow up with the provider if which solution is ordered? 1. 5% Dextrose in lactated Ringer's solution (D5LR) 2. Dextrose 5% in 0.45% normal saline 3. 0.45% Normal saline 4. Dextrose 5% in 0.9% normal saline

2. ANS: 3 Chapter number and title: 10. Overview of Infusion Therapies Chapter learning objective: 5. Describing the potential complications of infusion therapy and strategies to prevent these complications Chapter page reference: 157 Heading: Solutions Used in Infusion Therapies/Table 10.1 - Osmolarity of Intravenous Solutions Integrated Processes: Nursing Process: Analysis Client Need: Physiological Integrity: Pharmacological and Parenteral Therapies Cognitive Level: Analysis [Analyzing] Concept: Medication Difficulty: Difficult Feedback 1 5% Dextrose in lactated Ringer's solution (D5LR) is a hypertonic fluid that is indicated in patients with severe dehydration because it promotes fluid to move into the intravascular space. 2 Dextrose 5% in 0.45% normal saline is a hypertonic fluid that is indicated in patients with severe dehydration because it promotes fluid to move into the intravascular space. 3 0.45% Normal saline is not the best solution in this situation because the patient is in need of replacement of intravascular volume. Because this solution is hypotonic, it would cause fluid to move from the intravascular space into the both the intracellular and interstitial spaces. 4 Dextrose 5% in 0.9% normal saline is a hypertonic fluid that is indicated in patients with severe dehydration because it promotes fluid to move into the intravascular space.

2. Which assessment data collected by the nurse indicate that an older adult patient is at risk for dehydration? 1. Poor skin turgor 2. Body mass index of 20.5 3. Blood pressure of 140/98 mm Hg 4. Oral intake of 48 ounces per day

2. ANS: 4 Chapter number and title: 8, Fluid and Electrolyte Management Chapter learning objective: 4. Discussing changes in fluid and electrolyte balance associated with aging Chapter page reference: 116 Heading: Fluid and Electrolyte Regulation/Age-Related Changes Integrated Processes: Nursing Process: Assessment Client Need: Physiological Integrity: Physiological Adaptation Cognitive Level: Comprehension [Understanding] Concept: Assessment Difficulty: Moderate Feedback 1 Skin turgor is a poor indicator of fluid balance in an older adult patient. 2 A body mass index within normal limits would not contribute to dehydration. A body mass index associated with overweight or obesity could be associated with dehydration, because fat cells contain little or no water. 3 An elevated blood pressure could indicate fluid volume overload or sodium sensitivity. 4 A poor intake of water could indicate a loss of the thirst response, which occurs as a normal age-related change. Because the patient only ingests 48 ounces of water each day, this could indicate a reduction in the normal thirst response.

20. A patient with elevated serum potassium is being treated with intravenous insulin and dextrose. The patient states, "I do not have diabetes. Why am I receiving insulin?" Which statement by the nurse is most appropriate? 1. "You may have early diabetes, and you will need further tests to rule this out." 2. "The insulin is given to counteract the glucose we are giving you to help lower your potassium level." 3. "Because your potassium is so high, the insulin helps to decrease the levels by moving potassium into your cells." 4. "The insulin will help decrease your potassium by making your kidneys get rid of potassium."

20. ANS: 3 Chapter number and title: 9, Acid-Base Balance Chapter learning objective: 6. Explaining nursing considerations related to patients with acid-base disorders Chapter page reference: 150-151 Heading: Acid-Base Disorders/Metabolic Acidosis/Treatment Integrated Processes: Teaching and Learning Client Need: Physiological Integrity: Physiological Adaptation Cognitive Level: Analysis [Analyzing] Concept: Fluid and Electrolyte Balance Difficulty: Difficult Feedback 1 The insulin is used to drive potassium into the cells. This patient does not have diabetes mellitus. 2 The insulin is used to drive the potassium into the cells and isn't given to counteract the glucose. 3 Rapid-acting insulin may be needed for patients with diabetes mellitus to reverse diabetic ketoacidosis and drive potassium back into the cells. 4 The insulin helps move potassium into the cells. It does not impact renal clearance of potassium.

20. In monitoring a patient receiving a blood transfusion, the nurse correlates which clinical manifestation as indicative of a hemolytic transfusion reaction? 1. Increased blood pressure 2. Itching 3. Fever 4. Facial flushing

20. ANS: 3 Chapter number and title: 9, Overview of Transfusion Therapy Chapter learning objective: 6. Explaining the procedure for safely administering blood products Chapter page reference: 171 Heading: Nursing Management of Infusion Therapy/Administration of Blood Products/ Table 10.7 - Types of Transfusion Reactions Integrated Processes: Nursing Process: Analysis Client Need: Physiological Integrity: Pharmacological and Parenteral Therapies Cognitive Level: Comprehension Concept: Assessment Difficulty: Moderate Feedback 1 Hypotension is a clinical manifestation of a hemolytic transfusion reaction. 2 Itching is a clinical manifestation of an allergic transfusion reaction. 3 Fever is a clinical manifestation of a hemolytic transfusion reaction. 4 Facial flushing is a clinical manifestation of an allergic transfusion reaction.

20. The nurse is caring for a patient with a potassium level of 5.9 mEq/L. The healthcare provider prescribes both glucose and insulin for the patient. The patient's spouse asks, "Why is insulin needed?" Which response by the nurse is the most appropriate? 1. "The insulin will help his kidneys excrete the extra potassium." 2. "The insulin is safer than other medications that can lower potassium levels." 3. "The insulin lowers his blood sugar levels, and this is how the extra potassium is excreted." 4. "The insulin will cause his extra potassium to move into his cells, which will lower potassium in the blood."

20. ANS: 4 Chapter number and title: 8, Fluid and Electrolyte Management Chapter learning objective: 7. Explaining nursing considerations related to patients with fluid and electrolyte disorders Chapter page reference: 131 - 132 Heading: Electrolyte Disorders /Hyperkalemia: Medical Management Integrated Processes: Teaching and Learning Client Need: Physiological Integrity: Pharmacological and Parenteral Therapies Cognitive Level: Application [Applying] Concept: Medication Difficulty: Moderate Feedback 1 Insulin does not promote renal excretion of potassium. 2 Giving insulin to decrease serum potassium levels is not considered a safer method than other medications that can be used. 3 Serum potassium is lowered by entering the cells; this is not controlled by serum glucose. 4 Serum potassium levels may be temporarily lowered by administering glucose and insulin, which cause potassium to leave the extracellular fluid and enter cells.

21. The nurse is providing care to a patient who is receiving a blood transfusion. Ten minutes after the infusion is initiated, the patient reports a headache. On further assessment the nurse notes that the patient is experiencing dyspnea and feels warm to the touch. Which is the priority nursing action by the nurse? 1. Stopping the transfusion 2. Preparing for a full resuscitation 3. Notifying the healthcare provider 4. Decreasing the rate of the transfusion

21. ANS: 1 Chapter number and title: 10, Overview of Infusion Therapies Chapter learning objective: 6. Explaining the procedure for safely administering blood products Chapter page reference: 171 Heading: Nursing Management of Infusion Therapy/Administration of Blood Products/ Table 10.7 - Types of Transfusion Reactions Integrated Processes: Nursing Process: Implementation Client Need: Physiological Integrity: Pharmacological and Parenteral Therapies Cognitive Level: Application [Applying] Concept: Safety Difficulty: Moderate Feedback 1 The priority nursing action is to stop the transfusion. If the patient is experiencing a transfusion reaction, this will limit the amount of blood administered. 2 There is no need for resuscitation based on the current data. 3 Although the nurse would contact the healthcare provider, this is not the priority. 4 Slowing the rate of the transfusion allows for the blood to continue to be administered; therefore, this is not an appropriate nursing action.

21. The nurse is providing care to a patient who has been vomiting for several days. Which acid-base disorder does the nurse assess for in this patient? 1. Metabolic alkalosis 2. Respiratory alkalosis 3. Metabolic acidosis 4. Respiratory acidosis

21. ANS: 1 Chapter number and title: 9, Acid-Base Balance Chapter learning objective: 4. Comparing and contrasting major acid-base disorders Chapter page reference: 151 Heading: Acid-Base Disorders/Metabolic Alkalosis Integrated Processes: Nursing Process: Assessment Client Need: Physiological Integrity: Physiological Adaptation Cognitive Level: Comprehension [Understanding] Concept: pH Regulation Difficulty: Moderate Feedback 1 Metabolic alkalosis caused by loss of hydrogen ions usually occurs because of vomiting or gastric suction. Gastric secretions are highly acidic (pH 1-3). When these are lost through vomiting or gastric suction, the alkalinity of body fluids increases. This increased alkalinity results from the loss of acid and from selective retention of bicarbonate by the kidneys as chloride is depleted. 2 Respiratory alkalosis is an acid-base imbalance caused by an increase in the rate of alveolar ventilation. Acute alveolar hyperventilation is often the result of anxiety and is commonly referred to as hyperventilation syndrome. 3 Metabolic acidosis caused by an increased accumulation of metabolic acids that rise in proportion to a decrease in bicarbonate and result in decreased arterial pH. 4 Respiratory acidosis results from altered ventilation leading to CO2 retention.

21. The nurse provides care for a patient who is experiencing hypomagnesemia. Which food choice is best for this patient? 1. A medium-sized banana 2. One medium-sized baked potato 3. Three ounces of cooked halibut 4. A half-cup of cooked black-eyed peas

21. ANS: 3 Chapter number and title: 8, Fluid and Electrolyte Management Chapter learning objective: 7. Explaining nursing considerations related to patients with fluid and electrolyte disorders Chapter page reference: 133 Heading: Electrolyte Disorders/Hypomagnesemia/Nursing Management/ Box 8.3 Food Sources for Magnesium Integrated Processes: Nursing Process: Nursing Implementation Client Need: Physiological Integrity: Basic Care and Comfort Cognitive Level: Analysis [Analyzing] Concept: Nutrition Difficulty: Moderate Feedback 1 A medium-sized banana contains 30 mg of magnesium. This is not the best food choice for the patient. 2 A medium-sized baked potato with the skin contains 50 mg of magnesium. This is not the best food choice for the patient. 3 Three ounces of cooked halibut contains 90 mg of magnesium. Of the foods provided, this choice is the most magnesium rich. 4 One-half cup of cooked black-eyed peas contains 45 mg of magnesium. This is not the best food choice for the patient.

22. The nurse monitors for which clinical manifestations in the patient admitted with hypermagnesemia? 1. Elevated blood pressure 2. Bradycardia 3. Increased deep tendon reflexes 4. Hyperventilation

22. ANS: 2 Chapter number and title: 8, Fluid and Electrolyte Management Chapter learning objective: 6. Correlating laboratory data and clinical manifestations related to disorders in magnesium. Chapter page reference: 134 Heading: Electrolyte Disorders /Hypermagnesemia: Clinical Manifestations Integrated Processes: Nursing Process: Assessment Client Need: Physiological Integrity: Physiological Adaptation Cognitive Level: Comprehension [Understanding] Concept: Assessment Difficulty: Moderate Feedback 1 Cardiac symptoms in hypermagnesemia include hypotension caused by vasodilation and dysrhythmias such as bradycardia, atrial fibrillation, and intraventricular conduction delays exhibited by widening of the QRS complexes. 2 Cardiac symptoms in hypermagnesemia include hypotension caused by vasodilation and dysrhythmias such as bradycardia, atrial fibrillation, and intraventricular conduction delays exhibited by widening of the QRS complexes. 3 Central nervous system clinical manifestations include drowsiness, lethargy, muscle weakness, loss of deep tendon reflexes, paralysis, and coma. 4 Respiratory signs include a decrease in respiratory rate that can lead to complete respiratory suppression.

22. The nurse is administering a blood transfusion to an adult patient. The patient reports feeling cold and is shivering 15 minutes after the initiation of the transfusion. The patient's blood pressure has decreased since the last assessment. Which is the nurse's priority action? 1. Slowing the infusion rate and notifying the healthcare provider 2. Slowing the infusion rate and continuing to monitor the blood pressure every 5 minutes 3. Stopping the blood infusion, and infusing normal saline through the existing intravenous (IV) tubing 4. Stopping the blood infusion, removing the tubing from the IV catheter, and replacing it with normal saline

22. ANS: 4 Chapter number and title: 10, Overview of Infusion Therapies Chapter learning objective: 6. Explaining the procedure for safely administering blood products Chapter page reference: 171 Heading: Nursing Management of Infusion Therapy/Administration of Blood Products/Administration of Blood Products/ Table 10.7 - Types of Transfusion Reactions Integrated Processes: Nursing Process: Implementation Client Need: Physiological Integrity: Reduction of Risk Potential Cognitive Level: Application [Applying] Concept: Safety Difficulty: Moderate Feedback 1 The patient is demonstrating clinical manifestations of a transfusion reaction, and the transfusion must be stopped. Then the provider can be contacted. 2 The patient is demonstrating clinical manifestations of a transfusion reaction, and the transfusion must be stopped. The patient will need to be continuously monitored, but stopping the transfusion is the priority action. 3 Stopping the blood infusion and running saline through the blood tubing will administer the blood found in the tubing and could make the transfusion reaction worse. 4 The nurse should completely discontinue the blood infusion, disconnecting the tubing from the IV catheter and placing normal saline or the ordered solution infusing before beginning the blood infusion with new tubing.

22. The nurse provides care to a patient admitted for the treatment of vomiting. When assessing the patient, the nurse closely monitors the patient for which clinical manifestation(s)? 1. Paresthesia 2. Respiratory distress 3. Kussmaul's respirations 4. Muscle twitching and cramps

22. ANS: 4 Chapter number and title: 9, Acid-Base Balance Chapter learning objective: 4. Comparing and contrasting major acid-base disorders Chapter page reference: 152 Heading: Metabolic Alkalosis/ Box 9.9 - Clinical Manifestations of Metabolic Alkalosis Integrated Processes: Nursing Process: Assessment Client Need: Physiological Integrity: Reduction of Risk Potential Cognitive Level: Analysis [Analyzing] Concept: Assessment Difficulty: Moderate Feedback 1 Paresthesia is anticipated for the patient at risk for respiratory alkalosis. 2 Respiratory distress is anticipated for the patient at risk for respiratory acidosis. 3 Kussmaul's respirations are expected for the patient who experiences metabolic acidosis. This is not anticipated based on the patient's admitting diagnosis. 4 Vomiting necessitating admission to the hospital places the patient at risk for metabolic alkalosis. Clinical manifestations of this condition include muscle twitching and cramps.

23. The nurse is monitoring laboratory results on assigned patients. The patient with which laboratory has the highest risk of laryngospasm? 1. Serum calcium 7.5 mg/dL 2. Serum magnesium 3.0 mg/dL 3. Serum potassium 5.6 mEq/L 4. Serum phosphorus 1 mg/dL

23. ANS: 1 Chapter number and title: 8, Fluid and Electrolyte Management Chapter learning objective: 6. Correlating laboratory data and clinical manifestations related to disorders in calcium Chapter page reference: 135 Heading: Electrolyte Disorders /Hypocalcemia: Clinical Manifestations Integrated Processes: Nursing Process: Assessment Client Need: Physiological Integrity: Physiological Adaptation Cognitive Level: Analysis [Analyzing] Concept: Fluid and Electrolyte Balance Difficulty: Difficult Feedback 1 Clinical manifestations of hypocalcemia include positive Trousseau's and Chvostek's signs, tetany, and laryngospasm. 2 Respiratory signs of hypermagnesemia include a decrease in respiratory rate that can lead to complete respiratory suppression, not laryngospasm. 3 Clinical manifestations of hyperkalemia include generalized fatigue, muscle cramps, palpitations, paresthesia, and weakness. 4 Clinical manifestations include alterations in neurological, cardiac, and musculoskeletal function. The most common manifestation of hypophosphatemia is skeletal or smooth muscle weakness, including respiratory insufficiency from diaphragmatic dysfunction. Laryngospasm is not associated with hypophosphatemia.

Which of these hospital staff members will the nurse manager assign to coordinate the discharge of a client who will need community-based rehabilitation services after a traumatic injury? A. The nurse responsible for the client's case management B. The physical therapist who developed the client's exercise program C. The health care provider assigned as the client's medical resident D. The unit-based RN who has cared for the client during the hospital stay

A

23. The nurse monitors for clinical manifestation of metabolic alkalosis in the patient with which disorder? 1. Diarrhea 2. Prolonged vomiting 3. Renal failure 4. Salicylate intoxication

23. ANS: 2 Chapter number and title: 9, Acid-Base Balance Chapter learning objective: 4. Comparing and contrasting major acid-base disorders Chapter page reference: 152 Heading: Acid-Base Disorders/Metabolic Alkalosis/Box 9.8 - Causes of Metabolic Alkalosis Integrated Processes: Nursing Process: Assessment Client Need: Physiological Integrity: Physiological Adaptation Cognitive Level: Application [Applying] Concept: pH Regulation Difficulty: Moderate Feedback 1 Loss of alkali secondary to severe diarrhea or intestinal malabsorption is associated with metabolic acidosis. 2 Most instances of metabolic alkalosis are due to an increased loss of acid through the gastrointestinal tract, usually via vomiting or nasogastric suctioning. 3 Decreased acid excretion by the kidneys in acute or chronic renal failure leads to metabolic acidosis. 4 Salicylate intoxication is associated with metabolic acidosis and respiratory alkalosis.

23. The patient who has been receiving Total Parenteral Nutrition (TPN) is being tapered from this therapy. It is important for the nurse to monitor for which complication if the solution is discontinued too rapidly? 1. Hyperkalemia 2. Hypernatremia 3. Hypoglycemia 4. Hypocalcemia

23. ANS: 3 Chapter number and title: 10. Overview of Infusion Therapies Chapter learning objective: 7. Describing the special precautions required to safely administer parenteral nutrition Chapter page reference: 171 - 172 Heading: Administration of Total Parenteral Nutrition Integrated Processes: Nursing Process: Analysis Client Need: Physiological Integrity: Pharmacological and Parenteral Therapies Cognitive Level: Analysis Concept: Fluid and Electrolyte Balance Difficulty: Moderate Feedback 1 There is no risk of changes in potassium levels related to tapering of TPN. 2 There is no risk of changes in sodium levels related to tapering of TPN. 3 Because of the high concentrations of dextrose, TPN therapy is initiated and discontinued gradually. If the TPN solution is abruptly stopped, the patient may experience hypoglycemia because the body needs time to adjust to the decreased glucose in the solution. 4 There is no risk of changes in calcium levels related to tapering of TPN.

24. The nurse is performing venipuncture to initiate intravenous (IV) therapy. Which indicators should the nurse use when choosing the site for IV therapy? Select all that apply. 1. Choosing a straight vein 2. Avoiding a sclerotic vein 3. Using sites distal to joints 4. Using the dominant arm whenever possible 5. Avoiding areas of the vein with a valve

24. ANS: 1, 2, 3, 5 Chapter number and title: 10, Overview of Infusion Therapies Chapter learning objective: 3. Comparing peripheral and central venous access, including indications, access devices, and potential complications Chapter page reference: 159 - 160 Heading: Types of Intravenous Access Devices/Peripheral Venous Access Integrated Processes: Nursing Process: Implementation Client Need: Physiological Integrity: Pharmacological and Parenteral Therapies Cognitive Level: Application [Applying] Concept: Skin Integrity Difficulty: Moderate Feedback 1 This is correct. Straight veins provide space for the catheter to be inserted easily. 2 This is correct. Sclerotic veins make it difficult to obtain and maintain IV therapy. 3 This is correct. The site should be sufficiently distal to the wrist or elbow joint to avoid bending or kinking of the IV catheter. 4 This is incorrect. It is best, when possible, to use the patient's nondominant arm because movement might be somewhat limited, so the patient should be allowed to use the dominant arm. 5 This is correct. Areas of the vein containing a valve should be avoided because the valve acts as an obstruction for the needle to go through the vein.

24. The nurse is providing care to a patient whose serum calcium levels have increased since a surgical procedure performed 3 days ago. Which intervention does the nurse implement to decrease the risk for the development of hypercalcemia? 1. Monitor vital signs every 8 hours. 2. Encourage ambulation three times a day. 3. Irrigate the Foley catheter one time a day. 4. Recommend turning, coughing, and deep breathing every 2 hours

24. ANS: 2 Chapter number and title: 8, Fluid and Electrolyte Management Chapter learning objective: 7. Explaining nursing considerations related to patients with fluid and electrolyte disorders Chapter page reference: 136 Heading: Electrolyte Disorders /Hypercalcemia Integrated Processes: Nursing Process: Implementation Client Need: Physiological Integrity: Physiological Adaptation Cognitive Level: Application [Applying] Concept: Fluid and Electrolyte Balance Difficulty: Moderate Feedback 1 This intervention has no effect on the development of hypercalcemia. Patients with hypercalcemia may develop bradycardia. 2 Hypercalcemia can occur from immobility. Encouraging early and frequent ambulation of patients at risk for hypercalcemia, as well as adequate hydration, assists in preventing elevated serum calcium. 3 This intervention is not related to the risk for the development of hypercalcemia. 4 This intervention is not related to decreasing the risk for the development of hypercalcemia but is routine post-operative nursing care.

24. The nurse is reviewing the latest arterial blood gas results for a patient with metabolic alkalosis. Which result indicates that the metabolic alkalosis is compensated? 1. pH 7.32 2. HCO3 8 mEq/L 3. PaCO2 48 mm Hg 4. PaCO2 18 mm Hg

24. ANS: 3 Chapter number and title: 9, Acid-Base Balance Chapter learning objective: 3. Explaining the meaning of compensation. Chapter page reference: 151 Heading: Acid-Base Disorders/Metabolic Alkalosis: Arterial Blood Gas Results Integrated Processes: Nursing Process: Analysis Client Need: Physiological Integrity: Physiological Adaptation Cognitive Level: Analysis [Analyzing] Concept: pH Regulation Difficulty: Moderate Feedback 1 A normal pH level is 7.35 to 7.45. A pH of less than 7.35 indicates acidosis. 2 A HCO3 level of 8 mEq/L is low and is most likely associated with metabolic acidosis. In metabolic alkalosis, there is an excess of bicarbonate. 3 To compensate for this imbalance, the rate and depth of respirations decrease, leading to retention of carbon dioxide. With respiratory compensation, the PaCO2 is greater than 45 mm Hg. 4 A PaCO2 level of 18 mm Hg is low and is seen in respiratory alkalosis.

25. Which nursing actions are appropriate when conducting an Allen test? Select all that apply. 1. Rest the patient's arm on the mattress. 2. Support the patient's wrist with a rolled towel. 3. Tell the patient to relax the hand and then clench a fist. 4. Ensure that a second nurse is available to assist with the procedure. 5. Press the patient's radial and ulnar arteries using the index and middle fingers.

25. ANS: 1, 2, 5 Chapter number and title: 9, Acid-Base Balance Chapter learning objective: 2. Stating the steps for arterial blood gas interpretation Chapter page reference: 146 Heading: Arterial Blood Gas Interpretation/Obtaining an Arterial Blood Gas Integrated Processes: Nursing Process: Implementation Client Need: Physiological Integrity: Reduction of Risk Potential Cognitive Level: Application [Applying] Concept: Assessment Difficulty: Moderate Feedback 1 This is correct. Rest the patient's arm on the mattress or bedside stand and support the wrist with a rolled towel. 2 This is correct. Rest the patient's arm on the mattress or bedside stand and support his wrist with a rolled towel. 3 This is incorrect. The nurse will tell the patient to first clench the fist, hold the position for a few seconds, and then hold the hand in a relaxed position. 4 This is incorrect. A second nurse is not required to perform this test. 5 This is correct. The nurse uses the index and middle fingers to press on the patient's radial and ulnar arteries.

25. The nurse educator is developing a class on different types of intravenous catheters for an orientation for new staff. The nurse includes which types of catheters as examples of central venous devices? Select all that apply. 1. Midline catheters 2. PICC lines 3. 20-gauge over-the-needle IVAD 4. Implanted ports 5. Butterfly catheters

25. ANS: 2, 4 Chapter number and title: 10, Overview of Infusion Therapies Chapter learning objective: 3. Comparing peripheral and central venous access, including indications, access devices, and potential complications Chapter page reference: 161 - 164 Heading: Types of Intravenous Access Devices/Central Venous Access Integrated Processes: Teaching and Learning Client Need: Physiological Integrity: Pharmacological and Parenteral Therapies Cognitive Level: Comprehension [Understanding] Concept: Fluid and Electrolyte Balance Difficulty: Easy Feedback 1 This is incorrect. Midline catheters are a type of infusion device that are inserted in a peripheral vein in the upper extremities with tips that terminate distal to the shoulder in either the basilica, cephalic, or brachial vein. Midlines are appropriate for therapies expected to last between 1 and 4 weeks. 2 This is correct. Types of IVADs used to obtain central venous access include nontunneled percutaneous central catheters, tunneled catheters, implanted ports, and peripherally inserted central catheters (PICCs). 3 This is incorrect. A 20-gauge over-the-needle IVAD is a type of peripheral intravenous catheter and is not used for central venous access. 4 This is correct. Types of IVADs used to obtain central venous access include nontunneled percutaneous central catheters, tunneled catheters, implanted ports, and peripherally inserted central catheters (PICCs). 5 This is incorrect. The steel-winged device (often referred to as a butterfly because of the appearance of wings on each side of the needle) is indicated only for short-term or single-dose therapy because the rigid steel needle is more likely to puncture the vein and lead to fluid or medication leaking out of the vein. This is not a central venous catheter.

25. The nurse recognizes which intervention as the priority in the care of the patient with a serum phosphorus level of 2.0 mg/dL? 1. Decrease fluid intake. 2. Strain all urine for kidney stones. 3. Encourage consumption of milk and yogurt. 4. Discourage consumption of a high-calorie diet.

25. ANS: 3 Chapter number and title: 8, Fluid and Electrolyte Management Chapter learning objective: 7. Explaining nursing considerations related to patients with fluid and electrolyte disorders Chapter page reference: 137 Heading: Electrolyte Disorders /Hypophosphatemia Integrated Processes: Nursing Process: Implementation Client Need: Physiological Integrity: Basic Care and Comfort Cognitive Level: Application [Applying] Concept: Fluid and Electrolyte Balance Difficulty: Moderate Feedback 1 Fluid intake does not directly impact phosphorus levels. 2 Decreased phosphorus levels are not associated with renal calculi. 3 A phosphorus level of 2.0 is low, and the patient needs additional dietary phosphorus. Providing phosphorus-rich foods such as milk and yogurt is a good way to provide that additional phosphorus. 4 Caloric consumption is not associated with phosphorus levels.

output

2500 mL/day feces= 100 mL breathing= 400 mL skin= 500mL urine= 1500 mL

intake

2500 mL/day metabolism= 400ml food= 500 mL drink= 1600 mL

26. Which patients may benefit from central intravenous (IV) access? Select all that apply. 1. The patient receiving caustic IV therapy. 2. The patient requiring long-term IV therapy. 3. The patient who is afraid of needles and does not want a catheter in the peripheral extremity. 4. The patient requiring numerous IV infusions that are not compatible and cannot be infused together. 5. The unstable patient requiring reliable IV access for administration of medications required immediately.

26. ANS: 1, 2, 4, 5 Chapter number and title: 10, Overview of Infusion Therapies Chapter learning objective: 3. Comparing peripheral and central venous access, including indications, access devices, and potential complications Chapter page reference: 161 - 164 Heading: Types of Intravenous Access Devices/Central Venous Access Integrated Processes: Nursing Process: Planning Client Need: Physiological Integrity: Pharmacological and Parenteral Therapies Cognitive Level: Comprehension [Understanding] Concept: Fluid and Electrolyte Balance Difficulty: Moderate Feedback 1 This is correct. Central venous access can be very useful for patients requiring long-term IV therapy because the catheter can remain in place for extended periods and IV sites do not have to be changed every few days. 2 This is correct. Caustic medications are less likely to cause phlebitis when administered into the large central veins as opposed to the smaller peripheral veins. 3 This is incorrect. Because of the potential complications from central venous access, it would not be an option considered because of patient preference if short-term IV therapy is required. 4 This is correct. In the critical care areas where patients may receive numerous continuous IV medication drips that might not all be compatible infusing through the same site, a multiple-port central venous access device can provide the best option. 5 This is correct. Patients who are unstable and require rapid administration of medications require reliable IV access that might not be available with peripheral IV lines, and central venous access may be the best option.

26. In analyzing an arterial blood gas, the nurse correlates which findings to an uncompensated respiratory acidosis? Select all that apply. 1. pH 7.30 2. PaO2 82 mm Hg 3. PaCO2 48 mm Hg 4. HCO3 28 mEq/L 5. SaO2 90%

26. ANS: 1, 3 Chapter number and title: 9. Acid-Base Balance Chapter learning objective: 2. Stating the steps for arterial blood gas interpretation Chapter page reference: 146-147 Heading: Arterial Blood Gas Interpretation/Arterial Blood Gas Assessment Integrated Processes: Nursing Process: Assessment Client Need: Physiological Adaptation: Reduction of Risk Potential Cognitive Level: Comprehension [Understanding] Concept: pH Regulation Difficulty: Moderate Feedback 1 This is correct. pH 7.30 is observed in an acidotic state. The normal range is 7.35-7.45. 2 This is incorrect. The PaO2 measures the partial pressure of oxygen in arterial blood and is the most important factor in determining how oxygen binds to hemoglobin. The normal range is 80-95 mm Hg. 3 This is correct. Carbon dioxide is measured by the PaCO2 levels, which indicate the partial pressure of CO2 in the arterial blood. The PaCO2 is used to evaluate the respiratory component of acid-base balance. Primarily regulated by the ventilatory function of the lungs, normal arterial CO2 levels are 35 to 45 mm Hg. PaCO2 greater than 45 mm Hg is related to hypoventilation or excessive CO2 retention and therefore acidosis.

The health-care provider

A facilitator who empowers the consumer to choose the best available options to meet personal needs and cultural values.

26. The nurse recognizes which clinical manifestations as age-related changes that may impact fluid and electrolyte in the older adult? Select all that apply. 1. Increased salivation 2. Increased urine output 3. Decreased sense of smell 4. Decreased visual acuity 5. Changes in taste sensation

26. ANS: 3, 4, 5 Chapter number and title: 8, Fluid and Electrolyte Management Chapter learning objective: 4. Discussing changes in fluid and electrolyte balance associated with aging Chapter page reference: 116 Heading: Fluid an Electrolyte Regulation/Age-Related Changes Integrated Processes: Nursing Process: Assessment Client Need: Physiological Integrity: Physiological Adaptation Cognitive Level: Comprehension [Understanding] Concept: Assessment Difficulty: Moderate Feedback 1 This is incorrect. Alterations in smell also affect taste, impacting oral intake, especially fluids, of older adults. This would lead to dry mucous membranes. 2 This is incorrect. Alterations in smell also affect taste, impacting oral intake, especially fluids, of older adults. These patients typically have decreased urine output secondary to decreased fluid intake. 3 This is correct. With aging, there is a decrease in taste, smell, and thirst, which can impact fluid and electrolyte balance because it affects intake of fluids and food. With aging, there is a decline in olfactory function as a result of the decrease in olfactory fibers and receptors. Loss of these fibers and receptors results in a decrease in olfactory function and the ability to discriminate smells. 4 This is correct. Loss of olfactory fibers and receptors results in a decrease in olfactory function and the ability to discriminate smells. Alterations in smell also affect taste, impacting oral intake, especially fluids, of older adults. 5 This is correct. Alterations in smell also affect taste, impacting oral intake, especially fluids, of older adults.

27. The nurse provides care to a patient who is admitted to the hospital for an aspirin overdose. Which clinical manifestations does the nurse monitor for when assessing this patient? Select all that apply. 1. Confusion 2. Palpitations 3. Blurred vision 4. Respiratory distress 5. Decreased responsiveness

27. ANS: 1, 2, 3 Chapter number and title: 9, Acid-Base Balance Chapter learning objective: 4. Comparing and contrasting major acid-base disorders Chapter page reference: 149 Heading: Acid Base Disorders/Respiratory Alkalosis/ Box 9.5 - Clinical Manifestations of Respiratory Alkalosis Integrated Processes: Nursing Process: Assessment Client Need: Physiological Integrity: Reduction of Risk Potential Cognitive Level: Analysis [Analyzing] Concept: Assessment Difficulty: Difficult Feedback 1 This is correct. The patient admitted to the hospital for treatment of an aspirin overdose is at risk for developing respiratory alkalosis. Confusion is a clinical manifestation associated with this condition. 2 This is correct. The patient admitted to the hospital for treatment of an aspirin overdose is at risk for developing respiratory alkalosis. Palpitations are a clinical manifestation associated with this condition; therefore, it is appropriate for the nurse to monitor the patient for this symptom. 3 This is correct. The patient admitted to the hospital for treatment of an aspirin overdose is at risk for developing respiratory alkalosis. Blurred vision is a clinical manifestation associated with this condition; therefore, it is appropriate for the nurse to monitor the patient for this symptom. 4 This is incorrect. Respiratory distress is a clinical manifestation associated with respiratory acidosis. This patient is at risk for respiratory alkalosis, not acidosis. 5 This is incorrect. Decreased responsiveness is a clinical manifestation associated with respiratory acidosis. This patient is at risk for respiratory alkalosis, not acidosis.

27. The nurse is preparing an educational session for members of a community health center that focuses on ways to maintain fluid balance during the summer months. Which interventions should the nurse recommend? Select all that apply. 1. Drink diet soda. 2. Reduce the intake of coffee and tea. 3. Drink more fluids during hot weather. 4. Drink flat cola or ginger ale if vomiting. 5. Exercise during the hours of 10 a.m. and 2 p.m.

27. ANS: 2, 3, 4 Chapter number and title: 8, Fluid and Electrolyte Management Chapter learning objective: 7. Explaining nursing considerations related to patients with fluid and electrolyte disorders Chapter page reference: 117 - 118 Heading: Fluid Imbalances/Hypovolemia: Medical Management Integrated Processes: Nursing Process: Implementation Client Need: Physiological Integrity: Physiological Adaptation Cognitive Level: Application [Applying] Concept: Fluid and Electrolyte Balance Difficulty: Moderate Feedback 1 This is incorrect. Diet soda often contains caffeine. 2 This is correct. Actions to prevent fluid volume deficit during the summer months include increasing fluid intake, drinking flat cola or ginger ale if vomiting, and reducing the intake of coffee and tea. 3 This is correct. Actions to prevent fluid volume deficit during the summer months include increasing fluid intake, drinking flat cola or ginger ale if vomiting, and reducing the intake of coffee and tea. 4 This is correct. Actions to prevent fluid volume deficit during the summer months include increasing fluid intake, drinking flat cola or ginger ale if vomiting, and reducing the intake of coffee and tea. 5 This is incorrect. Exercising between the hours of 10 a.m. and 2 p.m., considered the hottest time of the day, should be avoided.

27. In monitoring a patient's intravenous (IV) site, the nurse correlates which findings to an infiltration? Select all that apply. 1. Redness at the insertion site 2. Purulent discharge the insertion site 3. Leaking of fluid from the insertion site 4. Blanched skin near the insertion site 5. Skin cool to touch near the insertion site

27. ANS: 3, 4, 5 Chapter number and title: 10, Overview of Infusion Therapies Chapter learning objective: 5. Describing the potential complications of infusion therapy and strategies to prevent these complications Chapter page reference: 165 - 166 Heading: Nursing Management of Infusion Therapy/ Monitoring and Preventing Complications/Phlebitis and Infiltration Integrated Processes: Nursing Process: Assessment Client Need: Physiological Integrity: Reduction of Risk Potential Cognitive Level: Comprehension [Understanding] Concept: Skin Integrity Difficulty: Moderate Feedback 1 This is incorrect. Redness is seen in patients with phlebitis. Phlebitis, inflammation of the vein, is characterized by pain and erythema along the vein. 2 This is incorrect. Purulent drainage around the site is indicative of an infectious or inflammatory process. 3 This is correct. Infiltration occurs when solution or medication is inadvertently infused into the tissue surrounding the vein. Clinical manifestations of infiltration include blanched skin, skin cool to the touch, edema, unexpected pain or burning at the insertion site or along the path of the vein, and leaking of fluid from the insertion site. 4 This is correct. Infiltration occurs when solution or medication is inadvertently infused into the tissue surrounding the vein. Clinical manifestations of infiltration include blanched skin, skin cool to the touch, edema, unexpected pain or burning at the insertion site or along the path of the vein, and leaking of fluid from the insertion site. 5 This is correct. Infiltration occurs when solution or medication is inadvertently infused into the tissue surrounding the vein. Clinical manifestations of infiltration include blanched skin, skin cool to the touch, edema, unexpected pain or burning at the insertion site or along the path of the vein, and leaking of fluid from the insertion site.

28. The nurse is caring for a patient with a central venous catheter (CVC). Which nursing actions should the nurse implement to prevent an air embolism? Select all that apply. 1. Using Luer-Lok connections 2. Frequently checking connections 3. Wearing sterile gloves when accessing any connections 4. Clamping catheters and injection sites when not in use 5. Placing the patient in low-Fowler position to remove the CVC

28. ANS: 1, 2, 4 Chapter number and title: 10, Overview of Infusion Therapies Chapter learning objective: 5. Describing the potential complications of infusion therapy and strategies to prevent these complications Chapter page reference: 166 - 167 Heading: Nursing Management of Infusion Therapy/Monitoring and Preventing Complications/Central Line Complications Integrated Processes: Nursing Process: Implementation Client Need: Physiological Integrity: Reduction of Risk Potential Cognitive Level: Analysis [Analyzing] Concept: Fluid and Electrolyte Balance Difficulty: Difficult Feedback 1 This is correct. The nurse should use Luer-Lok connections to prevent an air embolism. 2 This is correct. The nurse should frequently check all connections to prevent air from entering the IV line. 3 This is incorrect. Wearing sterile gloves when accessing any connections will not prevent an air embolism. 4 This is correct. Clamping catheters and injection sites when not in use will help to prevent an air embolism. 5 This is incorrect. The patient should be placed in the supine position for removal of the CVC.

28. Which risk factors exhibited by the patient presenting in the emergency department (ED) would place the patient at risk for metabolic acidosis? Select all that apply. 1. Pneumonia 2. Abdominal fistulas 3. Acute renal failure 4. Hypovolemic shock 5. Chronic obstructive pulmonary disease

28. ANS: 2, 3, 4 Chapter number and title: 9, Acid-Base Balance Chapter learning objective: 4. Comparing and contrasting major acid-base disorders Chapter page reference: 150 Heading: Acid Base Disorders/Metabolic Acidosis;/Box 9.6 - Causes of Metabolic Acidosis Integrated Processes: Nursing Process: Assessment Client Need: Physiological Integrity: Physiological Adaptation Cognitive Level: Comprehension [Understanding] Concept: Assessment Difficulty: Moderate Feedback 1 This is incorrect. Chronic obstructive pulmonary disease and pneumonia place the patient at risk for respiratory acidosis with the increased retention of carbon dioxide in the blood. 2 This is correct. Abdominal fistulas that lead to loss of alkali can cause metabolic acidosis. 3 This is correct. Decreased acid excretion by the kidneys in acute or chronic renal failure can lead to metabolic acidosis. 4 This is correct. Hypovolemic shock can lead to anaerobic metabolism that can cause metabolic acidosis. 5 This is incorrect. Chronic obstructive pulmonary disease and pneumonia place the patient at risk for respiratory acidosis with the increased retention of carbon dioxide in the blood.

28. The nurse is providing care to a patient who is exhibiting clinical manifestations of a fluid and electrolyte deficit. Which of the following orders does the nurse implement to address this disorder? Select all that apply. 1. Administer diuretics 2. Administer antibiotics 3. Place the patient in high-Fowler's position 4. Monitor patient's I&O 5. Initiate intravenous therapy

28. ANS: 3, 4, 5 Chapter number and title: 8, Fluid and Electrolyte Management Chapter learning objective: 5. Describing the pathophysiology, clinical presentations, and management of dehydration, hypovolemia, and hypervolemia Chapter page reference: 117 - 118 Heading: Fluid Imbalances /Hypovolemia: Medical Management Integrated Processes: Nursing Process: Implementation Client Need: Physiological Integrity: Physiological Adaptation Cognitive Level: Application [Applying] Concept: Fluid and Electrolyte Balance Difficulty: Moderate Feedback 1 This is incorrect. Diuretics may be ordered to reduce fluid volume excess. 2 This is incorrect. Antibiotics are not used for fluid and electrolyte imbalance. 3 This is correct. There is no indication for a high-Fowler's position. The patient may be placed in the supine position with severe fluid volume deficit. 4 This is correct. Monitoring patient's intake and output (I&O) is one of several ways to assess the patient's fluid status. 5 This is correct. Intravenous fluids may be ordered for the patient with a fluid volume deficit if replacement oral fluids cannot be taken in sufficient quantity.

29. The nurse should intervene if a patient receiving a blood transfusion develops the following clinical manifestations of circulatory overload. Select all that apply. 1. Dyspnea 2. Decreased urine output 3. Bradycardia 4. Increased blood pressure 5. Jugular venous distention

29. ANS: 1, 4, 5 Chapter number and title: 10, Overview of Infusion Therapies Chapter learning objective: 5. Describing the potential complications of infusion therapy and strategies to prevent these complications Chapter page reference: Heading: Nursing Management of Infusion Therapy/Administration of Blood Products/Table 10.7 - Types of Transfusion Reactions Integrated Processes: Nursing Process: Assessment Client Need: Psychological Integrity: Reduction of Risk Potential Cognitive Level: Comprehension [Understanding] Concept: Safety Difficulty: Moderate Feedback 1 This is correct. Clinical manifestations of circulatory overload include dyspnea, headache, jugular vein distention, edema, and increased blood pressure. 2 This is incorrect. Urine output would most likely increase with circulatory overload. 3 This is incorrect. The heart rate may remain at baseline or increase with circulatory overload. 4 This is correct. Clinical manifestations of circulatory overload include dyspnea, headache, jugular vein distention, edema, and increased blood pressure. 5 This is correct. Clinical manifestations of circulatory overload include dyspnea, headache, jugular vein distention, edema, and increased blood pressure.

29. The nurse correlates which clinical manifestations to the patient at risk for hypovolemic shock? Select all that apply. 1. Blood pressure of 110/70 2. Heart rate of 146 3. Urine output of 0 to 10 mL/hour 4. Cool, clammy skin 5. Increased bowel sounds

29. ANS: 2, 3, 4 Chapter number and title: 8, Fluid and Electrolyte Management Chapter learning objective: 4. Discussing changes in fluid and electrolyte balance associated with aging Chapter page reference: 118 Heading: Fluid Imbalances/ Hypovolemia: Fluid Volume Deficit—Complications Integrated Processes: Nursing Process: Assessment Client Need: Physiological Integrity: Physiological Adaptation Cognitive Level: Application [Applying] Concept: Assessment Difficulty: Difficult Feedback 1 This is incorrect. This blood pressure is within normal limits. Hypovolemic shock is manifested by hypotension, tachycardia, and signs of organ hypoperfusion. 2 This is correct. The heart rate is increased. Complications of fluid volume deficit occur with losses of large amounts of fluid volume. Hypovolemic shock can develop as evidenced by hypotension, tachycardia, and signs of organ hypoperfusion such as cool, clammy skin, oliguria progressing to anuria (lack of urine output), decreased level of consciousness, and tachypnea. 3 This is correct. Complications of fluid volume deficit occur with losses of large amounts of fluid volume. Hypovolemic shock can develop as evidenced by hypotension, tachycardia, and signs of organ hypoperfusion such as cool, clammy skin, oliguria progressing to anuria (lack of urine output), decreased level of consciousness, and tachypnea. 4 This is correct. Complications of fluid volume deficit occur with losses of large amounts of fluid volume. Hypovolemic shock can develop as evidenced by hypotension, tachycardia, and signs of organ hypoperfusion such as cool, clammy skin, oliguria progressing to anuria (lack of urine output), decreased level of consciousness, and tachypnea. 5 This is incorrect. Bowel sounds decrease in hypovolemic shock as a result of decreased perfusion of the gastrointestinal tract.

29. The nurse is caring for the patient experiencing hypovolemic shock and metabolic acidosis. Which nursing actions are appropriate for this patient? Select all that apply. 1. Limiting the intake of fluids 2. Administering sodium bicarbonate 3. Monitoring electrocardiogram for conduction problems 4. Keeping the bed in the locked and low position 5. Monitoring weight on admission and discharge

29. ANS: 2, 3, 4 Chapter number and title: 9, Acid-Base Balance Chapter learning objective: 6. Explaining nursing considerations related to patients with acid-base disorders Chapter page reference: 150 Heading: Acid-Base Disorders/Metabolic Acidosis: Treatment Integrated Processes: Nursing Process: Implementation Client Need: Physiological Integrity: Physiological Adaptation Cognitive Level: Application [Applying] Concept: pH Regulation Difficulty: Moderate Feedback 1 This is incorrect. The treatment for hypovolemic shock includes the administration of fluids, not limiting fluids. 2 This is correct. Administering sodium bicarbonate and monitoring electrocardiograms (ECGs) are appropriate for the patient with shock. 3 This is correct. Administering sodium bicarbonate and monitoring ECGs are appropriate for the patient with shock. 4 This is correct. The patient recovering from hypovolemic shock is at risk for injury, so the bed should be kept in the locked and low position. 5 This is incorrect. Patients being treated for hypovolemia will require daily weights, not a weight on admission and then discharge.

-Provide the family with information on hereditary cancer risks -Offer to refer the daughters to a genetic counselor. -Arrange for the client to have genetic testing

A hospice nurse is providing care to a patient diagnosed with ovarian cancer. The patient is concerned that her two daughters are at an increased risk for cancer and asks the nurse for help. Which actions by the nurse are appropriate? Select all that apply

3. The nurse plans care for a hospitalized patient. Which data necessitate the inclusion of interventions to address a fluid volume deficit? 1. Urine output of 30 mL/hour 2. Heart rate of 110 bpm 3. Weight gain of 10 pounds in 3 days 4. Plus-3 edema in bilateral lower extremities

3. ANS: 2 Chapter number and title: 8, Fluid and Electrolyte Management Chapter learning objective: Reviewing basic concepts related to fluid and electrolyte balance Chapter page reference: 117 Heading: Fluid Imbalances/Hypovolemia: Fluid Volume Deficit/Clinical Manifestations Integrated Processes: Nursing Process: Planning Client Need: Physiological Integrity: Reduction of Risk Potential Cognitive Level: Comprehension [Understanding] Concept: Assessment Difficulty: Moderate Feedback 1 Expected urine output for an adult patient is 30 mL/hour. A decreased urine output would necessitate interventions to address a fluid volume deficit. 2 An increased heart rate is indicative of a fluid volume deficit. 3 Weight loss, not weight gain, supports the inclusion of interventions to address a fluid volume deficit. 4 Dependent edema supports the inclusion of interventions to address fluid volume excess, not a fluid volume deficit.

3. In providing care to a patient with a severe fluid deficit, which prescribed intravenous solution does the nurse recognize as most effective in expanding plasma volume? 1. 0.9 % Normal saline 2. Dextrose 10% 3. 0.45 % Normal saline 4. Albumin

3. ANS: 4 Chapter number and title: 10. Overview of Infusion Therapies Chapter learning objective: 2. Describing the characteristics of common IV solutions Chapter page reference: 156 - 158 Heading: Solutions Used for Infusion Therapy Integrated Processes: Nursing Process: Analysis Client Need: Physiological Integrity: Pharmacological and Parenteral Therapies Cognitive Level: Analysis [Analyzing] Concept: Fluid and Electrolyte Balance Difficulty: Difficult Feedback 1 0.9% Normal saline is an isotonic fluid and is a crystalloid solution. 2 Dextrose 10% is a hypertonic crystalloid solution and would not be used as a plasma volume expander. 3 0.45% Normal saline is a hypotonic crystalloid and promotes fluid moving out of the intravascular space leading to depletion of intravascular fluid volume. 4 Colloidal solutions are often referred to as plasma volume expanders because the larger molecules do not diffuse through cell membranes and draw fluid into the intravascular space. Colloidal solutions are used to maintain intravascular volume and prevent shock after major blood or fluid losses. Examples of colloidal solutions include albumin, dextran, and mannitol.

3. In caring for patients with acid-base disorders, it is important that the nurse understand what regarding the renal buffering system? 1. This is the fastest buffering system. 2. Levels of PaCO2 are regulated by the renal buffers. 3. Hydrogen ions are excreted if the body is in alkalosis. 4. Bicarbonate is excreted if the body is in alkalosis.

3. ANS: 4 Chapter number and title: 9, Acid-Base Balance Chapter learning objective: 1. Describing the significance of acid-base balance for normal function Chapter page reference: 143 Heading: Acid-Base Balance Overview/Renal Buffers Integrated Processes: Nursing Process: Assessment Client Need: Physiological Integrity: Physiological Adaptation Cognitive Level: Comprehension [Understanding] Concept: pH Regulation Difficulty: Moderate Feedback 1 The renal buffers are the most effective, yet the slowest-acting, buffering system and buffer by regulating bicarbonate levels. 2 Levels of PaCO2 are regulated by the lungs. 3 If the body is in a state of acidosis, then the kidneys excrete hydrogen ions (H+) and reabsorb HCO3-. This results in urine becoming more acidic, bicarbonate levels increasing, and the pH increasing. 4 If the body is in a state of alkalosis, then the kidneys excrete HCO3- and reabsorb hydrogen ions (H+). This results in urine becoming more alkaline, a drop in the blood bicarbonate levels, and a decrease in pH.

30. The nurse is providing care to a patient who is admitted with manifestations of metabolic alkalosis. Which diagnostic test results are consistent with this? Select all that apply. 1. Serum glucose level 142 mg/dL 2. Blood pH 7.47 3. Arterial HCO3 34 mEq/L 4. Bilateral lower lobe infiltrates noted on chest x-ray 5. Electrocardiogram changes consistent with hypokalemia

30. ANS: 2, 3, 5 Chapter number and title: 9, Acid-Base Balance Chapter learning objective: 4. Comparing and contrasting major acid-base disorders Chapter page reference: 151 Heading: Acid-Base Disorders/Metabolic Alkalosis: Arterial Blood Gas Results Integrated Processes: Nursing Process: Assessment Client Need: Physiological Integrity: Physiological Adaptation Cognitive Level: Analysis [Analyzing] Concept: Fluid and Electrolyte Balance Difficulty: Difficult Feedback 1 This is incorrect. Serum glucose level is not used to confirm the diagnosis of metabolic alkalosis. 2 This is correct. In metabolic alkalosis, the blood pH will be greater than 7.45 and the bicarbonate level greater than 28 mEq/L. 3 This is correct. In metabolic alkalosis, the blood pH will be greater than 7.45 and the bicarbonate level greater than 28 mEq/L. 4 This is incorrect. The presence of bilateral lower lobe infiltrates on chest x-ray would not contribute to the development of metabolic alkalosis. This finding might be the result of metabolic alkalosis if the client's respiratory status is compromised. 5 This is correct. The ECG pattern shows changes similar to those seen with hypokalemia. To increase serum hydrogen ion levels in metabolic alkalosis, serum potassium is pumped into the cell in exchange for the serum hydrogen ions. This decreases the serum potassium levels while the intracellular potassium levels increase, resulting in hypokalemia.

30. A patient's serum sodium level is 150 mg/dL. Based on this information, which interventions should the nurse plan for this patient? Select all that apply. 1. Elevate the head of the bed. 2. Instruct on a low-sodium diet. 3. Monitor heart rate and rhythm. 4. Administer diuretics as prescribed. 5. Administer potassium supplement as prescribed.

30. ANS: 2, 4 Chapter number and title: 8, Fluid and Electrolyte Management Chapter learning objective: 6. Correlating laboratory data and clinical manifestations related to disorders in: Sodium balance Chapter page reference: 126 - 127 Heading: Electrolyte Disorders/Hypernatremia: Medical Management Integrated Processes: Nursing Process: Planning Client Need: Physiological Integrity: Physiological Adaptation Cognitive Level: Application [Applying] Concept: Fluid and Electrolyte Balance Difficulty: Moderate Feedback 1 This is incorrect. Elevating the head of the bed would be appropriate if the patient were demonstrating signs of fluid volume overload. This is not known at this time and would not be a routine intervention with an elevated sodium level. 2 This is correct. For an elevated sodium level, the electrolyte will need to be restricted, in the form of a low-sodium diet. 3 This is incorrect. Monitoring of heart rate and rhythm would be more appropriate with a potassium imbalance. 4 This is correct. Diuretics will remove excess fluid being held in the body because of the extra sodium. 5 This is incorrect. A potassium imbalance is not associated with a sodium imbalance. More information is needed before this intervention would be planned or implemented.

What is touch for Native American?

A light touch of a person's hand rather than a firm handshake is used when greeting a person

31. The nurse educates a patient who is prescribed furosemide (Lasix) for congestive heart failure on foods rich in potassium. Which patient menu choices indicate to the nurse a correct understanding of the information presented? Select all that apply. 1. Pasta 2. Spinach 3. Applesauce 4. A sweet potato 5. Low-fat milk

31. ANS: 2, 4, 5 Chapter number and title: 8, Fluid and Electrolyte Management Chapter learning objective: 7. Explaining nursing considerations related to patients with fluid and electrolyte disorders Chapter page reference: 120 Heading: Potassium/Table 8.2 - Potassium Content of Common Foods Integrated Processes: Teaching and Learning Client Need: Physiological Integrity: Basic Care and Comfort Cognitive Level: Application [Applying] Concept: Nutrition Difficulty: Difficult Feedback 1 This is incorrect. Pasta is a low-potassium food; therefore, this choice indicates a need for additional teaching regarding foods rich in potassium. 2 This is correct. Spinach contains 839 mg of potassium in 1 cup cooked; therefore, this choice indicates a correct understanding of the information presented. 3 This is incorrect. Applesauce is a low-potassium food; therefore, this choice indicates a need for additional teaching regarding foods rich in potassium. 4 This is correct. A sweet potato contains 475 mg potassium in 1/2 cup cooked; therefore, this choice indicates a correct understanding of the information presented. 5 This is correct. Low-fat milk contains 407 mg of potassium in 1 cup; therefore, this choice indicates a correct understanding of the information presented.

4. Which intravenous (IV) fluid should the nurse prepare when a patient requires an isotonic solution? 1. 0.9% Normal saline 2. 2.5% Dextrose in water 3. 0.33% Sodium chloride 4. 5% Dextrose in lactated Ringer's

4. ANS: 1 Chapter number and title: 10, Overview of Infusion Therapies Chapter learning objective: 2. Describing the characteristics of common IV solutions Chapter page reference: 156 - 158 Heading: Solutions Used in Infusion Therapy Integrated Processes: Nursing Process: Implementation Client Need: Physiological Integrity: Pharmacological and Parenteral Therapies Cognitive Level: Comprehension Concept: Medication Difficulty: Easy Feedback 1 An example of an isotonic solution is 0.9% normal saline. 2 An example of a hypotonic solution is 2.5% dextrose in water. 3 An example of a hypotonic solution is 0.33% sodium chloride. 4 An example of a hypertonic solution is 5% dextrose in lactated Ringer's solution.

4. A patient is admitted for evaluation of a bowel obstruction and has a nasogastric tube placed to low intermittent suction. The physician orders arterial blood gases, and the following results are reported: pH 7.37 PaCO2 33 mm Hg PaO2 94 mm Hg HCO3 19 mm Hg SaO2 98% What is the nurse's interpretation of these results? 1. Fully compensated metabolic acidosis 2. Uncompensated metabolic acidosis 3. Uncompensated respiratory alkalosis 4. Partially compensated respiratory alkalosis

4. ANS: 1 Chapter number and title: 9, Acid-Base Balance Chapter learning objective: 2. Stating the steps for arterial blood gas interpretation Chapter page reference: 146-147 Heading: Arterial Blood Gas Assessment Integrated Processes: Nursing Process: Analysis Client Need: Physiological Integrity: Physiological Adaptation Cognitive Level: Application [Applying] Concept: pH Regulation Difficulty: Difficult Feedback 1 This is fully compensated metabolic acidosis because the pH is within normal limits but less than 7.40. The HCO3 of 19 indicates metabolic acidosis, and the decreased PaCO2 demonstrates compensation through increased respiratory rate (hyperventilation). 2 The patient has a metabolic acidosis, but it is fully compensated because the pH is within normal limits and the PaCO2 is low, which is consistent with compensation. 3 Although the PaCO2 is low, the pH is more on the acidotic end (less than 7.40), which is consistent with acidosis. 4 Although the PaCO2 is low, the pH is more on the acidotic end (less than 7.40), which is consistent with acidosis.

4. In reviewing laboratory results for a female patient suspected of having a fluid imbalance, the nurse correlates which laboratory value with a diagnosis of dehydration? 1. Hemoglobin of 10.5 g/dL 2. Hematocrit 49% 3. Serum potassium 3.8 mEq/L 4. Serum osmolality 230 mOsm/kg

4. ANS: 2 Chapter number and title: 8, Fluid and Electrolyte Management Chapter learning objective: 5. Describing the pathophysiology, clinical presentations, and management of dehydration, hypovolemia, and hypervolemia Chapter page reference: 117 Heading: Hypovolemia: Fluid Volume Deficit/ Laboratory Values Integrated Processes: Nursing Process: Assessment Client Need: Physiological Integrity: Reduction of Risk Potential Cognitive Level: Application [Applying] Concept: Fluid and Electrolyte Balance Difficulty: Moderate Feedback 1 A normal hemoglobin value for a female is 11.7 to 15.5 g/dL. The hemoglobin is not directly affected by fluid volume status. 2 The hematocrit measures the volume of whole blood that is composed of red blood cells. Because the hematocrit is a measure of the volume of cells in relation to plasma, it is affected by changes in plasma volume. The hematocrit increases with severe dehydration. The normal hematocrit value for a female is 36% to 48%. 3 Serum potassium is not an electrolyte used to determine an alteration in fluid balance. Serum sodium values usually change related to fluid volume changes. 4 Serum osmolality is a measure of the solute concentration of the blood and is used to evaluate fluid balance. Normal values are 275 to 295 mOsm/kg. An increase in serum osmolality indicates a fluid volume deficit; a decrease reflects fluid volume excess.

5. The nurse is analyzing the intake and output record for a patient being treated for dehydration. The patient weighs 176 lbs and had a 24-hour intake of 2,000 mL and urine output of 1,200 mL. Based on this data, which conclusion by the nurse is the most appropriate? 1. Treatment has not been effective. 2. Treatment needs to include a diuretic. 3. Treatment is effective and should continue. 4. Treatment has been effective and should end.

5. ANS: 3 Chapter number and title: 8, Fluid and Electrolyte Management Chapter learning objective: 7. Explaining nursing considerations related to patients with fluid and electrolyte disorders Chapter page reference: 117 - 118 Heading: Hypovolemia: Fluid Volume Deficit/ Medical Management Integrated Processes: Nursing Process: Evaluation Client Need: Physiological Integrity: Physiological Adaptation Cognitive Level: Analysis [Analyzing] Concept: Assessment Difficulty: Difficult Feedback 1 Treatment has been effective. 2 A diuretic is not needed because the patient is being treated for dehydration. 3 Urinary output is normally equivalent to the amount of fluids ingested; the usual range is 1,500 to 2,000 mL in 24 hours, or 40 to 80 mL in 1 hour (0.5 mL/kg per hour). Patients whose intake substantially exceeds output are at risk for fluid volume excess; however, the patient is dehydrated. The extra fluid intake is being used to improve body fluid balance. The patient's output is 40 mL/hour, which is within the normal range. 4 Treatment has been effective; however, it should continue until the intake and output are more balanced. Ending treatment now could further jeopardize this client's fluid balance.

5. The nurse is providing care to patient with the following laboratory values: pH 7.31; PaCO2 48 mm Hg; and HCO3 24 mEq/L. The nurse correlates these values to which acid-base disorder? 1. Metabolic acidosis 2. Metabolic alkalosis 3. Respiratory acidosis 4. Respiratory alkalosis

5. ANS: 3 Chapter number and title: 9, Acid-Base Balance Chapter learning objective: 2. Stating the steps for arterial blood gas interpretation Chapter page reference: 148 Heading: Acid-Base Disorders/Respiratory Acidosis: Arterial Blood Gas Results Integrated Processes: Nursing Process: Planning Client Need: Physiological Integrity: Reduction of Risk Potential Cognitive Level: Analysis Concept: pH Regulation Difficulty: Moderate Feedback 1 Arterial blood gas (ABG) results for metabolic acidosis are pH less than 7.35 and HCO3- less than 22 mEq/L. Uncompensated metabolic acidosis has a decreased pH, normal PaCO2, and normal HCO3. With respiratory compensation, the PaCO2 is less than 35 mm Hg. 2 ABG results for metabolic alkalosis are pH greater than 7.45 and HCO3- greater than 26 mEq/L. Uncompensated metabolic alkalosis has an increased pH, normal PaCO2 and increased HCO3. With respiratory compensation, the PaCO2 is greater than 45 mm Hg. 3 The ABG results for respiratory acidosis are pH less than 7.35 and PaCO2 greater than 45 mm Hg. If the pH is decreased and the PaCO2 is increased with a normal HCO3, it is uncompensated respiratory acidosis. With metabolic compensation, the HCO3- is greater than 26 mEq/L. 4 ABG results for respiratory alkalosis are pH greater than 7.45 and PaCO2 less than 35 mm Hg. Uncompensated respiratory alkalosis has an increased pH, decreased PaCO2, and normal HCO3. With metabolic compensation, then the HCO3- is less than 22 mEq/L.

5. Which intravenous (IV) fluid should the nurse prepare when a patient requires a hypertonic solution? 1. 0.9% Normal saline 2. 2.5% Dextrose in water 3. 0.33% Sodium chloride 4. 5% Dextrose in lactated Ringer's

5. ANS: 4 Chapter number and title: 10, Overview of Infusion Therapies Chapter learning objective: 2. Describing the characteristics of common IV solutions Chapter page reference: 156 - 158 Heading: Solutions Used in Infusion Therapy/Table 10.1 - Osmolarity of Intravenous Solutions Integrated Processes: Nursing Process: Implementation Client Need: Physiological Integrity: Pharmacological and Parenteral Therapies Cognitive Level: Comprehension [Understanding] Concept: Fluid and Electrolyte Balance Difficulty: Easy Feedback 1 0.9% Normal saline is an isotonic solution. 2 2.5% Dextrose in water is a hypotonic solution 3 0.33% Sodium chloride is a hypotonic solution. 4 5% Dextrose in lactated Ringer's is a hypertonic solution.

6. The nurse is providing care for a patient admitted to the unit with respiratory failure and respiratory acidosis. The nurse correlates which data from the patient's history as the probable cause for the current diagnosis? 1. Recent severe pulmonary infection 2. A recent trip to South America 3. Recent recovery from a cold virus 4. Use of ibuprofen for the control of pain

6. ANS: 1 Chapter number and title: 9, Acid-Base Balance Chapter learning objective: 4. Comparing and contrasting major acid-base disorders Chapter page reference: 148 Heading: Acid-Base Disorders/Respiratory Acidosis/Box 9.2 Causes of Respiratory Acidosis Integrated Processes: Nursing Process: Analysis Client Need: Physiological Integrity: Physiological Adaptation Cognitive Level: Comprehension [Understanding] Concept: pH Regulation Difficulty: Moderate Feedback 1 Severe pulmonary infections, bronchial obstruction, and atelectasis increase the risk of respiratory acidosis because of decreased ventilation and carbon dioxide retention. 2 A recent trip to South America would not constitute a respiratory risk factor for acidosis. 3 Recent recovery from a cold would not likely put the patient at risk for respiratory acidosis. 4 Ibuprofen does not pose a threat to the respiratory health of the patient.

6. The nurse provides care to a patient who is prescribed 0.45% normal saline (NS) by intravenous (IV) infusion. Which data cause the nurse to question the healthcare provider regarding this IV fluid order? 1. Urine output 40 mL/hour 2. Blood pressure 100/60 mm Hg 3. Respiratory rate 12 breaths/min 4. Serum sodium level of 145 mEq/L

6. ANS: 2 Chapter number and title: 8, Fluid and Electrolyte Management Chapter learning objective: 6. Correlating laboratory data and clinical manifestations of sodium balance. Chapter page reference: 119 Heading: Hypovolemia: Fluid Volume Deficit /Medical Management Table 8.4 Common IV Fluids: Crystalloids and Colloids Integrated Processes: NP Analysis Client Need: Physiological Integrity: Physiological Adaptation Cognitive Level: Analysis [Analyzing] Concept: Assessment Difficulty: Moderate Feedback 1 This urine output is within normal limits; therefore, there is no reason to question the intravenous (IV) prescription based on this data. 2 Hypotonic IV fluid, such as 0.45% normal saline (NS), shifts fluid out of the vessels and into the cells. Because of this fluid shift, hypotension may be worsened. Therefore, the patient's blood pressure causes the nurse to question the healthcare provider about this prescription. 3 A respiratory rate of 12 breaths/min is within normal limits; therefore, there is no reason to question the IV prescription based on this data. 4 Hypotonic IV fluid, such as 0.45% NS, shifts fluid out of the vessels and into the cells. This may cause hyponatremia to occur. The patient's serum sodium level is on the high end of normal; therefore, this data does not cause the nurse to question this prescription. A low-serum-sodium level would necessitate the nurse to question this order.

7. The nurse prepares to initiate intravenous (IV) access for an older adult patient who requires a blood transfusion. Which gauge needle is best for the nurse to use for this procedure? 1. 18 2. 20 3. 22 4. 24

7. ANS: 2 Chapter number and title: 10, Overview of Infusion Therapies Chapter learning objective: 3. Comparing peripheral and central venous access, including indications, access devices and potential complications Chapter page reference: 159 - 160 Heading: Types of Intravenous Access Devices/Peripheral Venous Access/Table 10.3 - Peripheral Catheter Gauge Selections Integrated Processes: Nursing Process: Planning Client Need: Physiological Integrity: Pharmacological and Parenteral Therapies Cognitive Level: Application [Applying] Concept: Medication Difficulty: Moderate Feedback 1 Although an 18-gauge catheter is appropriate for the administration of blood, this is not the best choice given the patient's age. 2 A 20-gauge catheter is appropriate for blood transfusion. Because the patient is an older adult, this is the best choice for the nurse to use for this procedure. 3 A 22-gauge catheter is appropriate for continuous or intermittent infusions in small veins but too small for blood transfusion. 4 A 24-gauge catheter is appropriate for continuous or intermittent infusions in fragile veins but too small for blood transfusion.

7. The nurse is caring for a patient who is receiving intravenous fluids postoperatively after cardiac surgery. The nurse correlates the patient's risk for fluid volume excess to which cause? 1. Decreased mobility as a result of surgery and pain 2. Administration of intravenous fluids 3. Decreased levels of aldosterone 4. Increased levels of antidiuretic hormone

7. ANS: 4 Chapter number and title: 8, Fluid and Electrolyte Management Chapter learning objective: 2. Describing the role of endocrine, renal, and respiratory systems in the regulation of fluid and electrolyte balance Chapter page reference: 120 Heading: Hypervolemia: Fluid Volume Excess/Causes Integrated Processes: NP Analysis Client Need: Physiological Integrity: Reduction of Risk Potential Cognitive Level: Application [Applying] Concept: Fluid and Electrolyte Balance Difficulty: Moderate Feedback 1 Fluid volume excess is not caused by inactivity. 2 It is unlikely that the fluid volume excess experienced by the patient is caused by intravenous fluids. 3 Aldosterone secretion is increased in stressful conditions like surgery. 4 Antidiuretic hormone (ADH) and aldosterone levels are commonly increased as a result of the stress response before, during, and immediately after surgery. This increase leads to sodium and water retention.

7. A patient is admitted to the emergency department for the treatment of a drug overdose causing acute respiratory acidosis. Which substance noted on the toxicology report is the most likely cause for the current diagnosis? 1. PCP 2. Cocaine 3. Marijuana 4. Oxycodone

7. ANS: 4 Chapter number and title: 9, Acid-Base Balance Chapter learning objective: 4. Comparing and contrasting major acid-base disorders Chapter page reference: 148 Heading: Acid-Base Disorders/Respiratory Acidosis/Box 9.2 Causes of Respiratory Acidosis Integrated Processes: Nursing Process: Assessment Client Need: Physiological Integrity: Physiological Adaptation Cognitive Level: Comprehension [Understanding] Concept: Assessment Difficulty: Moderate Feedback 1 PCP is a hallucinogenic agent. 2 Cocaine is a stimulant. 3 Marijuana is not considered a drug that depresses the central nervous system or respiratory center. 4 Oxycodone is an opiate narcotic. Excessive use or overdose of narcotic substances can lead to respiratory depression and respiratory acidosis.

8. The nurse is providing care to a trauma patient who will require the rapid administration of large volumes of fluid in addition to a blood transfusion. Which gauge should the nurse use when initiating intravenous (IV) access for this patient? 1. 18 2. 20 3. 22 4. 24

8. ANS: 1 Chapter number and title: 10, Overview of Infusion Therapies Chapter learning objective: 4. Describing the equipment used to provide infusion therapy Chapter page reference: 159 - 161 Heading: Types of Intravenous Access Devices /Peripheral Venous Access Integrated Processes: Nursing Process: Planning Client Need: Physiological Integrity: Pharmacological and Parenteral Therapies Cognitive Level: Comprehension [Understanding] Concept: Medication Difficulty: Moderate Feedback 1 An 18-gauge catheter is appropriate to initiate IV access for a patient who requires both rapid administration of large volumes of fluid and a blood transfusion. 2 A 20-gauge catheter is appropriate for blood transfusion but is not best for the rapid administration of large volumes. 3 A 22-gauge catheter is too small for blood and rapid fluid administration. 4 A 24-gauge catheter is too small for blood and rapid fluid administration.

8. The nurse is providing care to a patient who is admitted after a morphine overdose. Which acid-base imbalance does the nurse assess for in this patient? 1. Metabolic acidosis 2. Metabolic alkalosis 3. Respiratory acidosis 4. Respiratory alkalosis

8. ANS: 3 Chapter number and title: 9, Acid-Base Balance Chapter learning objective: 4. Comparing and contrasting major acid-base disorders Chapter page reference: 148 Heading: Acid-Base Disorders/Respiratory Acidosis/Box 9.2 Causes of Respiratory Acidosis Integrated Processes: Nursing Process: Assessment Client Need: Physiological Integrity: Physiological Adaptation Cognitive Level: Comprehension [Understanding] Concept: pH Regulation Difficulty: Moderate Feedback 1 Metabolic acidosis is related to an accumulation of hydrogen ions and not typically observed in the patient with an opiate overdose. 2 Metabolic alkalosis is associated with loss of hydrogen ions and not caused by an opiate overdose. 3 Morphine is a narcotic and generally acts to decrease or suppress respirations; therefore, this patient is probably hypoventilating. The expected acid-base imbalance would be respiratory acidosis. 4 Respiratory alkalosis is associated with hyperventilation. Morphine depresses respiratory rate.

8. The nurse is planning care for the patient with acute renal failure and incorporates the nursing diagnosis of Excess Fluid Volume. Which assessment data support this nursing diagnosis? 1. Wheezing in the lungs 2. Generalized weakness 3. Urine output of 20 mL/hour 4. Pitting edema in the lower extremities

8. ANS: 4 Chapter number and title: 8, Fluid and Electrolyte Management Chapter learning objective: 2, Describing the role of endocrine, renal, and respiratory systems in the regulation of fluid and electrolyte balance Chapter page reference: 120 Heading: Hypervolemia: Fluid Volume Excess/Clinical Manifestations Integrated Processes: Nursing Process: Analysis Client Need: Physiological Integrity: Physiological Adaptation Cognitive Level: Comprehension [Understanding] Concept: Fluid and Electrolyte Balance Difficulty: Moderate Feedback 1 Wheezing in the lungs is an assessment consistent with asthma. 2 Generalized weakness is not typically observed in patients with fluid volume excess. 3 Urine output of 20 mL/hour is low and is associated with fluid volume deficit. 4 The patient in acute renal failure will likely be edematous, as the kidneys are not producing urine.

9. The nurse is planning care for an older adult patient with respiratory acidosis. Which intervention does the nurse include in this patient's plan of care? 1. Perform chest physiotherapy. 2. Reduce environmental stimuli. 3. Administer intravenous sodium bicarbonate. 4. Administer prescribed intravenous fluids carefully.

9. ANS: 1 Chapter number and title: 9, Acid-Base Balance Chapter learning objective: 6. Explaining nursing considerations related to patients with acid-base disorders Chapter page reference: 148 Heading: Acid-Base Disorders/Respiratory Acidosis: Possible Respiratory Treatments Integrated Processes: Nursing Process: Implementation Client Need: Physiological Integrity: Physiological Adaptation Cognitive Level: Application [Applying] Concept: pH Regulation Difficulty: Moderate Feedback 1 Respiratory acidosis results in a drop in the blood pH, reduced level of oxygen, and retaining of carbon dioxide. Chest physiotherapy facilitates the removal of secretions from the lungs, which can encourage deep breathing to decrease carbon dioxide levels and improve oxygenation. 2 Reducing environmental stimuli would be more appropriate for the patient with respiratory alkalosis who is anxious and hyperventilating. 3 Sodium bicarbonate is indicated in the treatment of metabolic acidosis. 4 Careful administration of intravenous fluids is important in the older patient with metabolic alkalosis because this population is at risk as a result of their fragile fluid and electrolyte status.

9. The nurse is providing care to an older adult patient who is receiving intravenous (IV) fluids at 150 mL/hour. It is important that the nurse assess for which clinical manifestations that could indicate fluid volume excess in this patient? 1. Flattened neck veins 2. Elevated blood pressure 3. Bradycardia 4. Skin tenting

9. ANS: 2 Chapter number and title: 8, Fluid and Electrolyte Management Chapter learning objective: 4. Discussing changes in fluid and electrolyte balance associated with aging Chapter page reference: 120 Heading: Hypervolemia: Fluid Volume Excess/Clinical Manifestations Integrated Processes: Nursing Process: Assessment Client Need: Physiological Integrity: Pharmacological and Parenteral Therapies Cognitive Level: Analysis [Analyzing] Concept: Assessment Difficulty: Moderate Feedback 1 Flat neck veins are indicative of fluid volume deficit/hypovolemia 2 The blood pressure may increase if fluids are administered too quickly, and older adults may not be able to tolerate the increased fluid. 3 Bradycardia is not associated with fluid volume excess. 4 Skin tenting is associated with fluid volume deficit. Assessing skin turgor may not be as effective in the older adult because of loss of skin elasticity associated with aging.

9. The nurse is initiating intravenous (IV) therapy for an adult patient who requires IV fluid infusion for 2 to 3 days and might require blood administration. Which would the nurse choose as the best option for IV catheterization? 1. Butterfly 2. Midline catheter 3. Short over-the-needle catheter 4. Implantable venous access device

9. ANS: 3 Chapter number and title: 10, Overview of Infusion Therapies Chapter learning objective: 4. Describing the equipment used to provide infusion therapy Chapter page reference: 159 - 161 Heading: Types of Intravenous Access Devices/Peripheral Venous Access Integrated Processes: Nursing Process: Planning Client Need: Physiological Integrity: Pharmacological and Parenteral Therapies Cognitive Level: Application [Applying] Concept: Medication Difficulty: Moderate Feedback 1 This steel-winged device, often referred to as a butterfly because of the appearance of wings on each side of the needle, is indicated only for short-term or single-dose therapy because the rigid steel needle is more likely to puncture the vein and lead to fluid or medication leaking out of the vein. 2 Midline catheters are inserted in a peripheral vein in the upper extremities with tips that terminate distal to the shoulder in either the basilica, cephalic, or brachial vein. Midlines are appropriate for therapies expected to last between 1 and 4 weeks. 3 The short over-the-needle IV catheter would be the best choice because the needle is removed and only the catheter remains in place, so it is more likely to last for 2 days without infiltrating. 4 Implantable venous access devices are used when IV fluid needs are anticipated for several months.

Academy of medical-surgical nursing nurses strategic framework

>>Mission: To promote excellence in medical-surgical nursing. >>Vision: MedSN use their powerful voice and focused action to continually improve patient care. >>Values: MedSN is distinct specialty with its own body of knowledge. We believe patients receiving better care when MedSN: *Engage in ongoing professional development *Use evidence based practice *Speak with a unified voice *Serve as leaders on health care terms *Have the necessary resources to deliver excellent care *Practice in a healthy work environment. >>Strategic Message The AMSN is a vibrant community of medical-surgical nurses who care about: *Improving patient care *Developing personally and professionally *Advocating for the specialty of medical-surgical nursing *Connecting with other nurses who share their connection and commitment.

A previously stable postoperative client on the medical-surgical unit now has a blood pressure of 88/40 mm Hg and a heart rate of 124 beats/min. After placing the client in Trendelenburg position, which action does the nurse perform next? A. Activate the Rapid Response Team. B. Call for a Code Blue. C. Determine the cause of the changes. D. Re-check the vital signs in 5 minutes.

A

Bedside computers are an example of informatics used in health care primarily for which purpose? A. Documenting interdisciplinary care B. Enhancing collaboration and coordination of care C. Offering clients access to e-mail and the Internet D. Retrieving data for evidence-based practice

A

The nursing student has been assigned to the hospital's Rapid Response Team (RRT). Which statement by the student indicates a correct understanding of the RRT member's purpose? A. "I will be caring for clients in the hospital." B. "I will be riding along in the hospital's ambulance." C. "I will be admitting clients to the hospital." D. "I will be observing Code Blue resuscitations."

A

When developing a standardized plan of care for clients with a diagnosis of pneumonia, how does the nurse find the best information about providing optimal nursing care? A. Access a website that reports on randomized controlled studies on nursing care for clients with pneumonia. B. Research the most recent articles in nursing magazines that discuss care for clients with pneumonia. C. Review the chart to determine what primary health care provider's prescriptions are frequently written for clients with pneumonia. D. Survey experienced RNs about which nursing actions are effective when caring for clients with pneumonia.

A

The nurse is providing care to a patient following hemodialysis. The patient is experiencing *tachycardia* and *decreased urine output* along with *skin* that is *pale and cool to the touch*. Which *goal of hemodialysis* does the nurse determine the patient has *not* met based on the current data?

A *reduction* of *extracellular fluid* The patient receiving hemodialysis is expected to have a reduction of extracellular fluid, not a fluid deficit that puts the patient at risk.

The nurse receives shift report on a pediatric medical-surgical unit. The nurse has been assigned four patients for the shift. Which child does the nurse plan to *assess first* based on the increased risk for dehydration?

A 15-month-old child with *tachypnea* The pediatric patient with the greatest risk for dehydration is the child who is under 2 years of age experiencing tachypnea which increases insensible fluid loss.

Living Wills

A living will is a legal document created by a competent person that provides the person's desires for medical care in the event the person is unable to independently make decisions regarding care. This document describes specifics about life-prolonging treatments the person will allow and those that will not be allowed and becomes effective only when the person becomes incapacitated to the point he or she cannot make decisions for himself or herself.

Ethnicity

A cultural pattern shared by people with the same cultural heritage. Language, preferred diet, specific customs, family roles, and religious beliefs are often shared among those with the same.

Culture

A defined set of learned values, beliefs, customs and behaviors that is shares by a common social group and is passed down through generations of family.

Culture and Death: Native American

A dying person may be surrounded by a positive celebratory atmosphere of family and children

The nurse is morally obligated to care for the patient unless the risk exceeds responsibility

A patient diagnosed with acquired immune deficiency syndrome (AIDS) is admitted to the acute care floor. Which stance regarding the care for this patient is supported by the American Nurses Association (ANA) Code of Ethics?

35 mL/hr

A patient is admitted to the emergency department (ED) for dehydration. The patient is 154 lbs. Which urine output indicate the rehydration efforts for this patient have been effective?

Gastrointestinal

A patient is admitted to the emergency department (ED) for fluid volume deficit. Which body system should the nurse focus to determine the cause of this imbalance when assessing this patient?

C. 20-gauge midline catheter

A patient is admitted with osteomyelitis and is going to require 3 to 4 weeks of IV antibiotics. Which IVAD is most appropriate for this patient? A. 16-gauge central venous B. 18-gauge over-the-needle IVAD C. 20-gauge midline catheter D. 22-gauge steel-winged device

Communicate only necessary information

A patient is diagnosed with a sexually transmitted infections (STI) and states to the nurse, "Promise you will not tell anyone about my condition." Which action should the nurse take, when considering the Health Insurance Portability and Accountability Act (HIPAA) of 1996?

C. 20

A patient is ordered to receive a blood transfusion because of anemia. What is the recommended minimum gauge of the IV catheter to administer blood? A. 16 B. 18 C. 20 D. 22

It is needed to maintain skeletal, cardiac, and neuromuscular activity

A patient is prescribed 20 mEq of potassium chloride due to excessive vomiting. Which is the rationale for this drug the nurse should provide to the patient?

Call the agency patient advocacy department

A patient tells the nurse, "I don't really like the nurse on the first shift; she treats me bad." Which action by the nurse is appropriate in order to advocate for this patient?

Excess Fluid Volume

A patient with acute renal failure has jugular vein distention, lower extremity edema, and elevated blood pressure. Based on this data, which nursing diagnosis is the most appropriate?

-Instruct on a low-sodium diet. -Administer diuretics as prescribed.

A patient's serum sodium level is 150 mg/dL. Based on this data, which interventions should the nurse plan for this patient? Select all that apply.

Culture and the Older Adult:

A positive attitude is toward life and health is encouraged in most cultures. Most cultures look to elders as a source of wisdom and elders play a role in raising and disciplining grandchildren.

Medical-surgical nursing

A specialty area characterized by competencies that are applicable for patients across the health-care continuum from acute care hospitals to home setting. The largest subspecialty in nursing, medical-surgical nurses are eligible for certification in this practice through AMSN or ANCC.

Which factors have led to the expansion of medical-surgical nursing from traditional acute care facilities to other settings?Select all that apply A. Increase in health-care complexity. B. Shortage of RN's C. Changes in insurance coverage D. Shortened hospital lengths of stay E. Low salaries for RN's in hospitals

A, C, D. Rationale: changes in healthcare delivery, legislation, and insurance coverage, have led to medical-surgical nursing being practiced in many different practice settings, including hospitals, same day surgical centers, acute rehabilitation centers, ambulatory care clinics, long term care settings, and patients' homes

A nursing student has been assigned to the hospital's Rapid Response Team (RRT). Which statement by the student indicates a correct understanding of the RRT member's purpose? A. "I will be caring for clients in the hospital." B. "I will be riding along in the hospital's ambulance." C. "I will be admitting clients to the hospital." D. "I will be observing code 'blue' resuscitations."

A. "I will be caring for clients in the hospital." Rationale A. Members of the RRT are critical care experts who are on site in the hospital and are available at any time.

When transferring a client who was admitted with chest pain from the emergency department (ED), which information is essential for the ED nurse to communicate to the nurse on the medical-surgical unit? Select all that apply. A. "The client is being admitted for ongoing monitoring of pain and vital signs." B. "The client has private insurance and is also covered by Medicare." C. "Nitroglycerin and morphine sulfate were given to relieve the pain." D. "Frequent reassurance is needed because the client has a high anxiety level." E. "The client has a family history of heart disease and hypertension." F. "A coronary arteriogram should be scheduled as soon as possible."

A. "The client is being admitted for ongoing monitoring of pain and vital signs." C. "Nitroglycerin and morphine sulfate were given to relieve the pain." D. "Frequent reassurance is needed because the client has a high anxiety level." E. "The client has a family history of heart disease and hypertension." F. "A coronary arteriogram should be scheduled as soon as possible." Rationale Because nursing care is not affected by the client's ability to pay, information about the type of insurance that the client has is not essential to communicate during a "hand-off" report for client care.

When developing a standardized plan of care for clients with a diagnosis of pneumonia, how does the nurse find the best information about providing optimal nursing care? A. Access a website that reports on randomized controlled studies on nursing care for clients with pneumonia. B. Research the most recent articles in nursing magazines that discuss care for clients with pneumonia. C. Review the chart to determine what physician prescriptions are frequently written for clients with pneumonia. D. Survey experienced RNs about which nursing actions are effective when caring for clients with pneumonia.

A. Access a website that reports on randomized controlled studies on nursing care for clients with pneumonia.

A previously stable postoperative client on the medical-surgical unit now has a blood pressure of 88/40 and a heart rate of 124. After placing the client in Trendelenburg position, which of these actions should the nurse take next? A. Activate the Rapid Response Team. B. Call for a Code Blue. C. Determine the cause of the changes. D. Recheck the vital signs in 5 minutes.

A. Activate the Rapid Response Team.

Which action should the nurse take first when preparing to do discharge teaching for a 73-year-old client who is being discharged after prostate surgery? A. Ask what the client knows about self-care after prostate surgery. B. Have family members available during the teaching. C. Obtain written information about post-discharge care. D. Plan to teach early in the morning after the client has eaten.

A. Ask what the client knows about self-care after prostate surgery. Rationale A. When planning education, the nurse's initial action should be to assess whether the client is receptive to teaching and to identify the client's current knowledge level.

A nursing student is following a medical-surgical nurse and identifies a way to improve a procedure used for client care. What does the student nurse do next? A. Asks the nurse about the rationale for doing the procedure that way B. Corrects the medical-surgical nurse about the way the procedure should be done C. Goes to the charge nurse with the idea D. Places a note in the suggestion box

A. Asks the nurse about the rationale for doing the procedure that way Rationale A. Although all nurses are responsible for quality improvement, it is important to assess before implementing. Asking why something is done a certain way is a good form of assessment.

A 78-year-old Mexican American client is admitted to the hospital for hypertension. His wife passed away 6 months ago, and his only child still lives in Mexico. Which intervention does the nurse use in dealing with this client following his admission? A. Assess the client for his use of folk medicine. B. Call the client's child and explain why his father needs him. C. Present all information on hypertension to the client in one teaching session. D. Provide a pamphlet on hypertension for initial education.

A. Assess the client for his use of folk medicine. Rationale A. Assessing the client's use of folk medicine will help determine his care.

A nurse is appointed to a hospital committee whose goal is to "improve nursing practice." Which areas of practice are included in the committee's task? Select all that apply. A. Attentiveness/surveillance of clients B. Mandatory reporting C. Medication administration D. Participation in professional organizations E. Prevention of errors or complications F. Teaching clients about their care regimens

A. Attentiveness/surveillance of clients B. Mandatory reporting C. Medication administration E. Prevention of errors or complications

A nurse educator is instructing newly hired registered nurses about patient-centered care. Which competency categories are included in this content? Select all that apply. A. Attitudes B. Environments C. Judgments D. Knowledge E. Skills F. Values

A. Attitudes D. Knowledge E. Skills Rationale Knowledge, skills, and attitudes are the client-centered care competencies that are needed to ensure quality care. These characteristics, which were developed by the Institute of Medicine (IOM) and the Quality and Safety Education for Nurses (QSEN) groups, are the areas that are cited and enumerated by both groups. A delineation (and examples of each) of knowledge, skills, and attitudes constitutes competent nursing practice.

Which important aspect of coordinating care within the interdisciplinary team is facilitated by use of the "SBAR" and "PACE" procedures? A. Communication B. Implementation C. Policy making D. Protocol development

A. Communication Rationale A. SBAR and PACE are acronyms for hand-off methods of communication used by health care organizations to share information between shifts and between departments.

Bedside computers are an example of informatics used in health care primarily for which purpose? A. Documenting interdisciplinary care B. Enhancing collaboration and coordination of care C. Offering clients access to e-mail and the Internet D. Retrieving data for the evidence-based practice

A. Documenting interdisciplinary care Rationale A. The largest application of health care informatics is the growing trend of the use of electronic medical records (EMRs) for documenting interdisciplinary care. Computers may be located at the client's bedside or in the treatment room for ease of access.

Which nursing activity is best for the charge nurse on the medical-surgical unit to delegate to unlicensed assistive personnel (UAP) staff members? A. Feeding a client whose hands are affected by rheumatoid arthritis B. Increasing the oxygen flow rate for a client who has wheezes C. Positioning a client who has just returned after hip surgery D. Taking vital signs for a client who is having acute chest pain

A. Feeding a client whose hands are affected by rheumatoid arthritis Rationale A. Although all of these actions may sometimes be delegated to UAP, the client with rheumatoid arthritis is the most stable of the clients described here.

Culture and food: The seventh day adventist

Encourages a vegetarian diet

PROVIDERS

The health care professional who, after review of the history and physical examination and results from laboratory and radiology findings, makes a diagnosis and plan of medical care. These include MD's, CRNPS and PA's.

Cortisol

The nurse is caring for a patient with congestive heart failure who is admitted to the medical-surgical unit with acute hypokalemia. Which prescribed medication may have contributed to the patient's current hypokalemic state?

Which of these hospital staff members will the nurse manager assign to coordinate the discharge of a client who will need community-based rehabilitation services after a traumatic injury? A. The nurse who is responsible for the client's case management B. The physical therapist who developed the client's exercise program C. The physician who is assigned as the client's medical resident D. The unit-based RN who has cared for the client during the hospital stay

A. The nurse who is responsible for the client's case management Rationale A. The case manager role includes coordination of acute care and post-discharge community services for the client.

*Intra*cellular

Potassium Magnesium Phosporus

The student nurse performs the initial assessment on a newly admitted patient. The student is aware that it is important for this assessment to be accurate for which reason? 1 Future assessments will be compared with the baseline. 2 Future assessments will become the new baseline. 3 Future assessments will be disregarded if they are not the same as the baseline. 4 Future assessments will be incorporated into the initial baseline.

ANS 1 All assessments following the initial assessment will be compared to the baseline assessment performed on admission.

A patient is admitted with acute abdominal pain. Which finding requires immediate attention? 1 BP 100/50, P 96, abdominal distention 2 Temperature 99° F, flatulence, nausea 3 Urinary frequency and dysuria 4 Temperature 99.2° F, amber-colored urine

ANS 1 Decreased BP, increased pulse rate, and abdominal distention are symptoms of altered bowel function and possible shock that require immediate intervention. The other signs and symptoms do not require immediate intervention.

The nurse is appointed to a hospital committee whose goal is to "improve the safety of nursing practice." Which areas of practice are included in the committee's task? (Select all that apply.) A. Attentiveness/surveillance of clients B. Mandatory reporting C. Medication administration D. Participation in professional organizations E. Prevention of errors or complications F. Teaching clients about their care regimens

ABCE

When transferring a client who was admitted with chest pain from the emergency department (ED), which information is essential for the ED nurse to communicate to the nurse on the medical-surgical unit? (Select all that apply.) A. "The client is being admitted for ongoing monitoring of pain and vital signs." B. "The client has private insurance and is also covered by Medicare." C. "Nitroglycerin and morphine sulfate were given to relieve the pain." D. "Frequent reassurance is needed because the client has a high anxiety level." E. "The client has a family history of heart disease and hypertension." F. "A coronary arteriogram should be scheduled as soon as possible."

ACDEF

The nurse educator is instructing newly hired registered nurses about client-centered care. Which competency categories are included in this content? (Select all that apply.) A. Attitudes B. Environments C. Judgments D. Knowledge E. Skills F. Values

ADE

A patient with a life-threatening condition is brought to the emergency department. The LPN/LVN is to care for the patient. What is the nurse's initial role? 1 Perform a thorough patient assessment and make tentative nursing diagnoses. 2 Determine a nursing diagnoses and set priorities for expected goals. 3 Assess the patient completely and determine what outcomes are realistic. 4 Assess the major problems and begin interventions.

ANS 1 Following the steps of the nursing process is necessary in order to plan and implement effective patient care. Assessment, establishing a nursing diagnosis, planning and implementation of nursing interventions, and evaluation of patient goals are the steps followed.

Which of these findings can the LPN/LVN obtain by observation? 1 The patient's hair is dirty. 2 The patient's breath has an odor of alcohol. 3 The patient's heartbeat is abnormal. 4 The patient's skin is clammy.

ANS 1 Observation involves looking and noting during data collection.

In a hospital, a patient who is having trouble breathing is very upset because the LPN/LVN has to help the patient bathe. The patient says to the nurse, "I don't think you should have to bathe me." The nurse's response is based on which of these understandings about the nurse's role? Meeting one's basic needs may require assistance from someone until one is able to manage independently. Patients must be encouraged to meet their own needs at all times. Hospital patients deserve to have basic needs cared for by someone else regardless of their ability to care for themselves. When a patient indicates that he or she is uncomfortable about getting assistance, then no assistance should be given.

ANS 1 The goal of care is to work toward the patient functioning as independently as possible. The nurse should explain that assistance is given in an attempt to help the patient achieve independence.

The LPN/LVN is interviewing a young married woman who is of a different culture than the nurse. The woman's husband is present and answers all the questions the nurse poses. What should the nurse first consider as a reason for his actions? 1 This may be a normal cultural practice. 2 Spouse abuse may be occurring. 3 The length of time this couple has been married may account for his actions. 4 The husband may understand the questions better than his wife.

ANS 1 The nurse should first consider that this may be a normal cultural practice. The nurse should investigate other reasons if it is found that this is not normal cultural practice.

When discussing the nursing process, the student nurse correctly states, "The nursing process is designed to provide a means for measuring __________." 1 patient outcomes 2 acuity of the patients on a nursing unit 3 expenses associated with care 4 appropriate tasks for delegation

ANS 1 The nursing process allows for measurement of patient outcomes by evaluating if established patient goals have been met.

The student nurse is studying the nursing process. Which statement best describes the nursing process? 1 A goal-directed, orderly series of activities 2 An attempt to define nursing practice 3 A plan to describe nursing functions 4 A theory of operative nursing standards

ANS 1 The nursing process is a series of steps planned and followed in an attempt to achieve a patient goal.

The LPN/LVN is to interview a newly admitted patient. For which of these reasons should the nurse review the patient's records before beginning the interview? 1 To check the accuracy of those records 2 To avoid asking the patient to repeat information already obtained 3 To determine other people's opinion of the patient's problems 4 To learn as much about the disease process as possible

ANS 2 It is very annoying for a patient to be repeatedly asked the same questions. Reviewing the records before interviewing the patient may prevent repetition.

The nurse is administering a tap-water enema. When the patient begins to complain of abdominal cramping, the nurse should: 1 stop administration of the enema. 2 lower the height of the container of water. 3 clamp the enema tubing and withdraw it slowly. 4 advance the tubing 1 to 2 inches.

ANS 2 Lowering the height of the enema container will decrease the pressure of the fluid running in, relieving some of the patient's discomfort. There is no indication of how much of the enema the patient has received. Advancing the tubing may increase the patient's discomfort.

The nurse is assessing a dark-skinned patient with anemia. Where is the best place to check for pallor? 1 Nail beds 2 Hard palate 3 Sclerae 4 Buccal mucosa

ANS 2 The hard palate is the best place to assess for pallor in a dark-skinned patient. Nail beds and buccal mucosa are not helpful areas for assessing for anemia unless it is severe. Sclerae are white; they are not a useful area for making this type of assessment.

The nurse is making assignments for the day's care. Which assignment should not be given to the nursing assistant? 1 Emptying the Foley (indwelling) catheter drainage bag 2 Administering a Fleet (small-volume) disposable enema 3 Measuring intake and output 4 Feeding a patient with depression

ANS 2 The nursing assistant should not be assigned to give the Fleet enema; this is considered medication administration. Nursing assistants may be assigned any of the other tasks.

The nurse is assessing a postoperative patient. Which intervention will best determine if peristalsis has returned? 1 Ask the patient if he or she has been passing gas. 2 Place a stethoscope on the abdomen and listen for bowel sounds. 3 Percuss the abdomen to determine whether there is bowel activity. 4 Palpate the abdomen to determine whether it reacts to touch.

ANS 2 While asking the patient if he or she has been passing gas will assist in determining the return of peristalsis, auscultation is the method that will give the nurse objective information regarding the presence of peristalsis.

When doing an initial assessment, why should the LPN/LVN be particularly interested in the patient's educational background? 1 This will help determine what the patient's expectations are regarding care. 2 This will help determine whether the patient is motivated toward a healthy lifestyle. 3 This will help determine the best method for presenting health care information. 4 This will help determine how the patient will react to stress.

ANS 3 It is important to present health care information to the patient in a way the patient can best understand the information. Determining the patient's educational level will assist in this process.

The LPN/LVN is assigned to a patient whom she has not cared for previously. For which of these reasons is it essential for the nurse to do a brief chart review and read the nurse's notes for the past 24 hours? 1 To ensure that proper care has been given in the past 2 To ensure that the same care will be given on this shift 3 To establish priorities for care to be given on this shift 4 To determine the identity of the patient's health care providers

ANS 3 Reviewing the chart will help the nurse to determine what has occurred on the previous shift, which will help in determining priorities for the present shift.

The nurse working on the renal unit is preparing to make first rounds of the day. Which patient should the nurse visit first? 1 The patient maintained on hemodialysis 2 The patient with glomerulonephritis 3 The patient who is 1 hour post-kidney transplant 4 The patient scheduled for an intravenous pyelogram (IVP)

ANS 3 The patient who is 1 hour post-kidney transplant is the least stable patient. The other patients are lower acuity patients and can be seen after the transplant patient.

The nurse should visit which patient first? 1 The patient with insulin-dependent diabetes and a blood glucose level of 95 2 The patient with hypertension being maintained on lisinopril 3 The patient with chest pain and a history of angina 4 The patient with Raynaud's disease

ANS 3 The patient with chest pain and a history of angina needs further evaluation now. The patient with insulin-dependent diabetes and a blood glucose level of 95 is within normal limits. The patient with hypertension being maintained on lisinopril is stable due to his medication regimen. The patient with Raynaud's disease is in no distress.

While sitting in the cafeteria, a nurse overhears two nursing students discussing a patient admitted for chemical detoxification. What is the best action by the nurse? 1 Report the incident to their instructor. 2 Report the incident to the nursing supervisor. 3 Discuss the incident with the students immediately. 4 Ignore the incident.

ANS 3 The students are violating confidentiality; the nurse should ask them to stop before anything else can be overheard.

After interviewing a newly admitted patient, the student nurse notices that the LPN/LVN reviews the data with the patient. The student is aware that this action occurs for what reason? 1 It is usually a hospital policy. 2 This shows how well the nurse has listened. 3 It is a mandate of the Patient's Bill of Rights. 4 To confirm the accuracy of the information.

ANS 4 At the end of the admission process, it is important for the nurse to confirm that data have been collected and interpreted accurately.

The nurse is caring for a male patient of Chinese descent. What should the nurse consider when caring for this patient? 1 He will most likely enjoy being with other members of that ethnic group. 2 He will have the same beliefs as others in that ethnic group. 3 He will appreciate being identified as a member of that ethnic group. 4 He may not necessarily adhere to the beliefs of that ethnic group.

ANS 4 Being a part of a particular ethnic background does not mean that the person follows the practices of that group. It is important to treat the person as an individual and not assume anything based on ethnic background.

The nurse is assisting the RN in planning care for a patient. Which nursing diagnosis should receive the highest priority? 1 Alteration in comfort 2 Alteration in mobility 3 Alteration in skin integrity 4 Alteration in oxygenation

ANS 4 Oxygenation status should be given priority.

To apply the nursing process, the LPN/LVN must understand which principle? 1 The nursing process is the only way for nurses to solve problems. 2 The patient has to follow the care plan as the nurse designed it. 3 The physician has to approve the care plan. 4 Each patient is a unique individual.

ANS 4 The nursing process is one way of solving the health-related problems of the patient, and it is important to incorporate each patient's unique circumstances into the plan of care.

Which intervention is most helpful in determining the need for oxygen therapy for a patient with chronic obstructive pulmonary disease (COPD)? 1 Ask the patient whether he needs supplemental oxygen. 2 Assess the patient's level of fatigue. 3 Evaluate the complete blood count (CBC). 4 Use a pulse oximeter on the patient's ear lobe.

ANS 4 This will obtain measurement of oxygen saturation. Distal digits of fingers other than the thumb may be used also. Asking the patient if he needs oxygen may not be adequate. Assessing the patient's fatigue level may not be adequate because other factors may be responsible for fatigue. Evaluating the patient's CBC will not provide oxygenation status.

The nurse is caring for a patient with congestive heart failure who is admitted to the medical-surgical unit with acute hypokalemia. Which prescribed medication may have contributed to the patient's current hypokalemic state? 1) Cortisol 2) Demerol 3) Skelaxin 4) Nonsteroidal anti-inflammatory drugs (NSAIDs)

ANS: 1 Excess potassium loss through the kidneys is often caused by such medications as corticosteroids, potassium-wasting (loop) diuretics, amphotericin B, and large doses of some antibiotics. Cortisol is a type of corticosteroid and can cause hypokalemia.

The nurse is instructing a patient with heart failure about a prescribed sodium-restricted diet. Which patient statement indicates that additional teaching is required? 1) "I can use as much salt substitute as I want." 2) "I have to read the labels on foods to find out the sodium content." 3) "I have to limit the intake of food with baking soda or baking powder." 4) "I can use spices and lemon juice to add flavor to food when cooking."

ANS: 1 Low-sodium salt substitutes are not really sodium-free. They may contain half as much sodium as regular salt. The patient should be instructed to use salt substitutes sparingly because larger amounts often taste bitter instead of salty.

The nurse is caring for a patient admitted with hypertension and chronic renal failure who receives hemodialysis three times per week. The nurse is assessing the patient's diet and notes the use of salt substitutes. When teaching the patient to avoid salt substitute, which rationale supports this teaching point? 1) They can potentiate hyperkalemia. 2) They will cause the client to retain fluid. 3) They will increase the risk of AV fistula infection. 4) They will interact with the client's antihypertensive medications.

ANS: 1 Many salt substitutes use potassium chloride. Potassium intake is carefully regulated in patients with renal failure, and the use of salt substitutes will worsen hyperkalemia.

Culture and Death: Unitarian Universalist faith

Prefers cremation to burial

What are cold illnesses/conditions?

Pregnancy, earache, chest pain, paralysis, GI diseases, rheumatism and tuberculosis.

An older adult patient, who lives in a long-term care facility, presents in the emergency department (ED) due to fever, nausea, and vomiting over the past two days. The patient denies thirst. The urine dipstick indicates a decreased urine specific gravity. Which medical diagnosis should the nurse anticipate when planning care for this patient? 1) Dehydration 2) Hypertension 3) Fluid overload 4) Congestive heart failure

ANS: 1 Older adult patients are less able to concentrate their urine, making them susceptible to dehydration. In addition, there is a deficit of the thirst response. However, fever, nausea, and vomiting resulting from these changes are not considered normal. The patient's symptoms of nausea and vomiting suggest decreased intake and increased output through vomiting, placing the client at risk for dehydration.

The nurse is concerned that an older adult patient is at risk for developing acute renal failure. Which information in the patient's history support the nurse's concern? Select all that apply. 1) Diagnosed with hypotension 2) Recent aortic valve replacement surgery 3) Total hip replacement surgery five years ago 4) Taking medication for type 2 diabetes mellitus 5) Prescribed high doses of intravenous antibiotics

ANS: 1, 2, 5 1. This is correct. Older adults develop acute renal failure more frequently because of the higher incidence of serious illnesses, hypotension, major surgeries, diagnostic procedures, and treatment with nephrotoxic drugs. Decreased kidney function associated with aging also puts the older patient at risk for kidney failure. Hypotension, scheduled for aortic valve replacement surgery, and receiving high doses of intravenous antibiotics increase this patient's risk for developing acute renal failure. 2. This is correct. Older adults develop acute renal failure more frequently because of the higher incidence of serious illnesses, hypotension, major surgeries, diagnostic procedures, and treatment with nephrotoxic drugs. Decreased kidney function associated with aging also puts the older patient at risk for kidney failure. Hypotension, scheduled for aortic valve replacement surgery, and receiving high doses of intravenous antibiotics increase this patient's risk for developing acute renal failure. 5. This is correct. Older adults develop acute renal failure more frequently because of the higher incidence of serious illnesses, hypotension, major surgeries, diagnostic procedures, and treatment with nephrotoxic drugs. Decreased kidney function associated with aging also puts the older patient at risk for kidney failure. Hypotension, scheduled for aortic valve replacement surgery, and receiving high doses of intravenous antibiotics increase this patient's risk for developing acute renal failure.

The community nurse visits the home of a young child who is home from school because of sudden onset of nausea, vomiting, and lethargy. The nurse suspects acute renal failure. Which clinical manifestations support the nurse's suspicions? Select all that apply. 1) Edema 2) Wheezing 3) Hematuria 4) Postural hypotension 5) Elevated blood pressure

ANS: 1, 3, 5 1. This is correct. Pediatric manifestations of acute renal failure characteristically begin with a healthy child who suddenly becomes ill with nonspecific symptoms that indicate a significant illness or injury. These symptoms may include any combination of the following: nausea, vomiting, lethargy, edema, gross hematuria, oliguria, and hypertension. 3. This is correct. Pediatric manifestations of acute renal failure characteristically begin with a healthy child who suddenly becomes ill with nonspecific symptoms that indicate a significant illness or injury. These symptoms may include any combination of the following: nausea, vomiting, lethargy, edema, gross hematuria, oliguria, and hypertension. 5. This is correct. Pediatric manifestations of acute renal failure characteristically begin with a healthy child who suddenly becomes ill with nonspecific symptoms that indicate a significant illness or injury. These symptoms may include any combination of the following: nausea, vomiting, lethargy, edema, gross hematuria, oliguria, and hypertension.

The nurse is providing care to a patient who seeks emergency treatment for headache and nausea. The patient works in a mill without air conditioning. The patient states, "I drink water several times each day but I seem to sweat more than I am able to replace." Which suggestions should the nurse provide to this patient? 1) Drink juices and carbonated sodas. 2) Eat something salty when drinking water. 3) Eat something sweet when drinking water. 4) Double the amount of water being ingested.

ANS: 2 Both salt and water are lost through sweating. When only water is replaced, the individual is at risk for salt depletion. Symptoms include fatigue, weakness, headache, and gastrointestinal symptoms such as loss of appetite and nausea. The client should be instructed to eat something salty when drinking water to help replace the loss of sodium.

An older adult patient, who appears intermittently confused, is admitted to the hospital after a fall. Based on the current data, which is the patient at an increased risk for developing? 1) Brain attack 2) Dehydration 3) Hemorrhage 4) Kidney damage

ANS: 2 During the aging process, the thirst mechanism declines. In a patient with an altered level of consciousness, this can increase the risk of dehydration and high serum osmolality.

The nurse is providing care to an older adult patient who is receiving intravenous (IV) fluids at 150 mL/hr. The patient is currently exhibiting crackles in the lungs, shortness of breath, and jugular vein distention. Which complication of IV fluid therapy does the nurse suspect the patient is experiencing? 1) Speed shock 2) Fluid volume excess 3) Anaphylactic reaction 4) Pulmonary embolism

ANS: 2 Fluid volume excess may occur when older adult patients receive intravenous fluids rapidly.

The nurse is providing care to a patient whose serum calcium levels have increased since a surgical procedure performed three days prior. Which intervention should the nurse implement to decrease the risk for the development of hypercalcemia? 1) Monitor vital signs every eight hours 2) Encourage ambulation three times a day 3) Irrigate the Foley catheter one time a day 4) Recommend turning, coughing, and deep breathing every two hours

ANS: 2 Hypercalcemia can occur from immobility. Ambulation of the client helps to prevent leaching of calcium from the bones into the serum.

A patient with acute renal failure has jugular vein distention, lower extremity edema, and elevated blood pressure. Based on this data, which nursing diagnosis is the most appropriate? 1) Risk for Infection 2) Excess Fluid Volume 3) Ineffective Renal Tissue Perfusion 4) Risk for Altered Cardiac Perfusion

ANS: 2 Jugular vein distention, edema, and elevated blood pressure are indications of excessive fluid. The diagnosis Excess Fluid Volume should be selected to guide this patient's care.

-Diagnosed with hypotension -Recent aortic valve replacement surgery -Prescribed high doses of intravenous antibiotics

The nurse is concerned that an older adult patient is at risk for developing acute renal failure. Which information in the patient's history support the nurse's concern? Select all that apply.

The nurse is reviewing laboratory values for a female patient suspected of having a fluid imbalance. Which laboratory value evaluated by the nurse supports the diagnosis of dehydration? 1) Hematocrit 30% 2) Hematocrit 53% 3) Serum potassium 3.8 mEq/L 4) Serum osmolality 230 mOsm/kg

ANS: 2 The hematocrit measures the volume of whole blood that is composed of RBCs. Because the hematocrit is a measure of the volume of cells in relation to plasma, it is affected by changes in plasma volume. The hematocrit increases with severe dehydration.

The nurse is providing care to a patient following hemodialysis. The patient is experiencing tachycardia and decreased urine output along with skin that is pale and cool to the touch. Which goal of hemodialysis does the nurse determine the patient has not met based on the current data? 1) Cardiac decompensation 2) A reduction of extracellular fluid 3) The effects of rapidly infused intravenous fluids 4) The pharmacological effects of a diuretic infused in the dialysate

ANS: 2 The patient receiving hemodialysis is expected to have a reduction of extracellular fluid, not a fluid deficit that puts the patient at risk.

The nurse receives shift report on a pediatric medical-surgical unit. The nurse has been assigned four patients for the shift. Which child does the nurse plan to assess first based on the increased risk for dehydration? 1) A 4-year-old child with a broken leg 2) A 15-month-old child with tachypnea 3) A 16-year-old child with migraine headaches 4) A 10-year-old child with cellulitis of the left leg

ANS: 2 The pediatric patient with the greatest risk for dehydration is the child who is under 2 years of age experiencing tachypnea which increases insensible fluid loss.

The school nurse is preparing a class session for high school students on ways to maintain fluid balance during the summer months. Which interventions should the nurse recommend Select all that apply. 1) Drink diet soda. 2) Reduce the intake of coffee and tea. 3) Drink more fluids during hot weather. 4) Drink flat cola or ginger ale if vomiting. 5) Exercise during the hours of 10 am and 2 pm.

ANS: 2, 3, 4 2. This is correct. Actions to prevent fluid volume deficit during the summer months include increasing fluid intake, drinking flat cola or ginger ale if vomiting, and reducing the intake of coffee and tea. 3. This is correct. Actions to prevent fluid volume deficit during the summer months include increasing fluid intake, drinking flat cola or ginger ale if vomiting, and reducing the intake of coffee and tea. 4. This is correct. Actions to prevent fluid volume deficit during the summer months include increasing fluid intake, drinking flat cola or ginger ale if vomiting, and reducing the intake of coffee and tea.

A patient's serum sodium level is 150 mg/dL. Based on this data, which interventions should the nurse plan for this patient? Select all that apply. 1) Elevate the head of the bed. 2) Instruct on a low-sodium diet. 3) Monitor heart rate and rhythm. 4) Administer diuretics as prescribed. 5) Administer potassium supplement as prescribed.

ANS: 2, 4 2. This is correct. For an elevated sodium level, the electrolyte will need to be restricted, in the form of a low-sodium diet. 4. This is correct. Diuretics will remove excess fluid being held in the body because of the extra sodium.

Which intervention should the nurse implement for a patient whose serum phosphorus level is 2.0 mg/dL? 1) Enforce contact precautions 2) Strain all urine for kidney stones 3) Encourage consumption of milk and yogurt 4) Discourage the consumption of a high-calorie diet

ANS: 3 A phosphorus level of 2.0 is low, and the client will need additional dietary phosphorus. Providing phosphorus-rich foods such as milk and yogurt is a good way to provide that additional phosphorus.

The nurse is providing care to a patient who is prescribed furosemide as part of the treatment for congestive heart failure (CHF). The patient's serum potassium level is 3.4 mEq/L. Which food should the nurse encourage the patient to eat based on this data? 1) Peas 2) Iced tea 3) Bananas 4) Baked fish

ANS: 3 A potassium level of 3.4 is low, so the client should be encouraged to consume potassium-rich foods. Of the foods listed, the highest in potassium is banana.

The nurse is teaching a group of children and their parents about the prevention of heat-related illness during exercise. Which statement by a parent indicates an appropriate understanding of the preventive techniques taught during the teaching session? 1) "My child only needs to hydrate at the end of an exercise session." 2) "Water is the drink of choice to replenish fluids that are lost during exercise." 3) "I will have my child stop every 15-20 minutes during the activity for fluids." 4) "It is important for my child to wear dark clothing while exercising in the heat."

ANS: 3 During activity, stopping for fluids every 15-20 minutes is recommended.

The nurse is providing care to a patient who is diagnosed with multisystem fluid volume deficit. The patient is currently experiencing tachycardia and decreased urine output along with skin that is pale and cool to the touch. The patient has a decreased urine output. Which probable cause to the patient's symptoms should the nurse include when educating the family? 1) Congestive heart failure 2) Rapidly infused intravenous fluids 3) Natural compensatory mechanisms 4) Pharmacological effects of a diuretic

ANS: 3 The internal vasoconstrictive compensatory reactions within the body are responsible for the symptoms exhibited. The body naturally attempts to conserve fluid internally specifically for the brain and heart.

A patient is admitted to the emergency department (ED) for fluid volume deficit. Which body system should the nurse focus to determine the cause of this imbalance when assessing this patient? 1) Genitourinary 2) Cardiovascular 3) Gastrointestinal 4) Musculoskeletal

ANS: 3 The most common cause of fluid volume deficit is excessive loss of gastrointestinal fluids, which can result from vomiting, diarrhea, suctioning, intestinal fistulas, or intestinal drainage. Other causes of fluid losses include chronic abuse of laxatives and/or enemas.

The nurse is analyzing the intake and output record for a patient being treated for dehydration. The patient weighs 176 lbs. and had a 24-hour intake of 2,000 mL and urine output of 1,200 mL. Based on this data, which conclusion by the nurse is the most appropriate? 1) Treatment has not been effective. 2) Treatment needs to include a diuretic. 3) Treatment is effective and should continue. 4) Treatment has been effective and should end.

ANS: 3 Urinary output is normally equivalent to the amount of fluids ingested; the usual range is 1,500-2,000 mL in 24 hours, or 40-80 mL in 1 hour (0.5 mL/kg per hour). Patients whose intake substantially exceeds output are at risk for fluid volume excess; however, the patient is dehydrated. The extra fluid intake is being used to improve body fluid balance. The patient's output is 40 mL/hr, which is within the normal range.

The nurse is providing care to a patient who is exhibiting clinical manifestations of a fluid and electrolyte deficit. Based on this data, which health-care provider prescriptions does the nurse prepare to implement? Select all that apply. 1) Administer diuretics 2) Administer antibiotics 3) Initiate hypodermoclysis 4) Closely monitor patient's I&O's 5) Initiate intravenous therapy

ANS: 3, 4, 5 3. This is correct. Hypodermoclysis, fluid administered subcutaneously, may be employed as a fluid delivery method, especially among older adults. 4. This is correct. Monitoring patient's intake and output is one of several ways to assess the patient's fluid status. 5. This is correct. Intravenous fluids may be ordered for the patient with a fluid volume deficit if replacement oral fluids cannot be taken in sufficient quantity.

Which data collected by the nurse during the assessment process places the older adult patient at risk for dehydration? 1) Poor skin turgor 2) Body mass index of 20.5 3) Blood pressure of 140/98 mmHg 4) Water intake of 2 glasses per day

ANS: 4 A poor intake of water could indicate a loss of the thirst response, which occurs as a normal age-related change. Since the patient only ingests two glasses of water each day, this could indicate a reduction in the normal thirst response.

The nurse is caring for a patient who is receiving intravenous fluids postoperatively following cardiac surgery. The nurse is aware that this patient is at risk for fluid volume excess. The family asks why the patient is at risk for this condition. Which response by the nurse is the most appropriate? 1) "Fluid volume excess is caused by inactivity." 2) "Fluid volume excess is caused by the intravenous fluids." 3) "Fluid volume excess is caused by new onset liver failure caused by the surgery." 4) "Fluid volume excess is common due to increased levels of antidiuretic hormone in response to the stress of surgery."

ANS: 4 Antidiuretic hormone (ADH) and aldosterone levels are commonly increased following the stress response before, during, and immediately after surgery. This increase leads to sodium and water retention. Adding more fluids intravenously can cause a fluid volume excess and stress upon the heart and circulatory system.

A patient is admitted to the emergency department (ED) for dehydration. The patient is 154 lbs. Which urine output indicate the rehydration efforts for this patient have been effective? 1) 20 mL/hr 2) 25 mL/hr 3) 30 mL/hr 4) 35 mL/hr

ANS: 4 Expected urine output for an adult patient is 0.5 mL/kg/hr. The patient currently weighs 70 kg; therefore, adequate urine output would be at least 35 mL/hr.

The nurse is providing care to an adult patient admitted with dehydration and hyponatremia. Which medical condition supports the current nursing diagnosis of Electrolyte Imbalance? 1) Osmotic pressure 2) Hydrostatic pressure 3) Isotonic dehydration 4) Hypotonic dehydration

ANS: 4 Hypotonic dehydration occurs when fluid loss is characterized by a proportionately greater loss of sodium than water, causing serum sodium to fall below normal levels.

A patient is prescribed 20 mEq of potassium chloride due to excessive vomiting. Which is the rationale for this drug the nurse should provide to the patient? 1) It controls and regulates water balance in the body. 2) It is used in the body to synthesize ingested protein. 3) It is vital in regulating muscle contraction and relaxation. 4) It is needed to maintain skeletal, cardiac, and neuromuscular activity.

ANS: 4 Potassium is the major cation in intracellular fluids, with only a small amount found in plasma and interstitial fluid. Potassium is a vital electrolyte for skeletal, cardiac, and smooth muscle activity.

The nurse is caring for a patient with a potassium level of 5.9 mEq/L. The health-care provider prescribes both glucose and insulin for the patient. The patient's spouse asks, "Why is insulin needed?" Which response by the nurse is the most appropriate? 1) "The insulin will help his kidneys excrete the extra potassium." 2) "The insulin is safer than other medications that can lower potassium levels." 3) "The insulin lowers his blood sugar levels and this is how the extra potassium is excreted." 4) "The insulin will cause his extra potassium to move into his cells, which will lower potassium in the blood."

ANS: 4 Serum potassium levels may be temporarily lowered by administering glucose and insulin, which cause potassium to leave the extracellular fluid and enter cells.

The nurse is planning care for the patient with acute renal failure. The nurse plans the patient's care based on the nursing diagnosis of Excess Fluid Volume. Which assessment data supports this nursing diagnosis? 1) Wheezing in the lungs 2) Generalized weakness 3) Bowel sounds positive in four quadrants 4) Pitting edema in the lower extremities

ANS: 4 The patient in acute renal failure will likely be edematous, as the kidneys are not producing urine.

Hypercalcemia

Abdominal pain, constipation, bone pain, decreased deep tendon reflexes, hypertension, thirst, lethargy, muscle weakness

-Patient -Family -Community

According to Provision 2 of the American Nurses Association (ANA) Code of Ethics, which member of the health-care team is the nurse's primary commitment? Select all that apply

How are food classified as hot or cold?

According to the effects on the body when metabolized rather than their thermal temperature

Cultural interventions

Achieved when health-care info is presented in a way that includes specific cultural styles, colors, pictures, symbols

Pharmacist

Actively involved in the mamagement of the patients medications including: • Reconciling the patient's medications between transitions of care (from home to the hospital and then from hospital to home) • Medication order review for appropriateness and adjustment to the hospital formulary • Drug interaction monitoring • Adverse drug reaction monitoring • Monitoring of therapeutic drug levels • Medication counseling to the patient and family members • Patient teaching on new or complex medications

What are alternative healers for Asian?

Acupuncturist, accpressurist, herbalist

Ad hoc members

Ad hoc members of the ICT may include: • Interpreter • Chaplain • Legal counsel • Palliative care coordinator • Substance abuse counselor • Post discharge call nurses

The nurse is providing care to a patient who is intubated and receiving mechanical ventilation after a motor vehicle crash. The patient is fighting the ventilator and attempting to remove the endotracheal tube. Which nursing action decreases the patient's risk for developing respiratory alkalosis?

Administer a prescribed sedative. -for a pt mechanically ventilated, the only way to reduce rapid respirations might be to provide a sedative.

The nurse is reviewing new orders provided by the health-care provider for a critical care patient with *metabolic acidosis*. Which prescription should the nurse question?

Administer one ampule of sodium bicarbonate now. -administering bicarbonate to correct acidosis increases the risk for *hypernatremia, hyperosmolarity & fluid volume excess*

What is family organization for Hispanics?

All members of the family may be involved in health-care decisions

Helping the family by providing information and allowing them to voice their concerns

An adolescent patient diagnosed with leukemia decides to stop chemotherapy treatments. The patient's parents, however, want the health-care team to continue all treatments. Which action by the nurse is appropriate when providing care to this patient and family?

Advance Directives

An advance directive is a document that is designed to communicate the final wishes of patients in the event they cannot speak for themselves. The directive is a set of precise instructions for the type and duration of care the patient wants.

National Center of Complementary and Alternative Medicine (NCCAM)

An agency of the National Institutes of Health (NIH) that conducts research on the effectiveness of specific CAM therapies and documents their findings in medical journals

Ethics Committees

An ethics committee consists of health-care providers from many different disciplines. 1. The autonomy model facilitates decision making for the competent patient. 2. The patient benefit model uses substituted judgment and facilitates decision making for the incompetent patient. 3. The social justice model considers broad social issues and is accountable to the institution.

Dehydration

An older adult patient, who appears intermittently confused, is admitted to the hospital after a fall. Based on the current data, which is the patient at an increased risk for developing?

Dehydration

An older adult patient, who lives in a long-term care facility, presents in the emergency department (ED) due to fever, nausea, and vomiting over the past two days. The patient denies thirst. The urine dipstick indicates a decreased urine specific gravity. Which medical diagnosis should the nurse anticipate when planning care for this patient?

American Nurses Credentialing center (ANCC)

Another available certification in medical-surgical nursing where nurses are given the credentials Registered nurses-board certified(RN-BC)

Hypocalcemia

Anxiety, confusion, irritability, paresthesias, *+ Chvostek's/Trousseau* sign, tetany, twitching, tremors

What is verbal communication for African Americans?

Asking personal questions of someone met for the first time is seen as improper

Social Workers (SW's)

Are the professionals who assess the physchosocial functioning of patients and families.

Stereotyping

Assuming that all the people from a specific cultural or ethnic group behave or belief the same way.

What is non verbal communication for Hispanics?

Avoidance of eye contact is a sign of attentiveness and respect

American Nursing Credentialing center (ANCC) Magnent Recognition Program

Awarded to health-care facilities that demonstrate excellence in the recruitment, recognition, and retention of nursing staff as well as excellence in patient care and quality. Because quality patient care is associated with a foundation i evidence-based practice findings, the ASMNS scope of medical-surgical nursing practice provides dissemination of evidene-based practice guidelines for medical-surgical nurses.

The nurse has recently been assigned to a medical-surgical clinical rotation. According to the scope of medical-surgical nursing, what type of client assignments does the nurse expect to see? A. Hospitalized children with acute and chronic illnesses B. Hospitalized adults with acute and chronic illnesses C. Older adults in a nursing home D. Working adults in a corporate setting

B

The nurse is using the concepts from SHARE (targeted solutions to ensure successful communication among care providers) to improve communication in a hospital setting. Which action indicates an understanding of these principles? A. Allowing various informed sources to contribute information B. Developing a standardized checklist form C. Encouraging the use of individualized methods of data transfer D. Minimizing input from the interdisciplinary team ("pit crew")

B

The nursing student asks the supervising nurse whether a certain fall protocol used on the nursing unit is effective. To demonstrate effectiveness, what does the supervising nurse identify? A. Information about how to implement a fall protocol and what nurses need to document B. Data about the number of falls after the protocol was introduced compared with previous fall rates C. The number of clients who currently have a fall protocol in place D. National statistics about the use of fall protocols to prevent serious injury from falls

B

Which nursing action demonstrates use of the principle of justice? A. A 32-year-old client is prevented from falling during the initial postoperative period following her hysterectomy. B. A 67-year-old client with dementia is shown the same respect as his 47-year-old roommate with prostate cancer. C. An 82-year-old client is provided access to the hospital Patient Advocate for processing of a complaint. D. The parents of a 13-year-old are included in discussions about the course of their teen's treatment and care.

B

Which role of the medical-surgical nurse is demonstrated when writing a plan of care for a client who is newly admitted to the hospital? A. Advocate B. Caregiver C. Communicator D. Educator

B

Which nursing action demonstrates use of the principle of justice? A. A 32-year-old client is prevented from falling during the initial postoperative period following her hysterectomy. B. A 67-year-old client with dementia is shown the same respect as his 47-year-old roommate with prostate cancer. C. An 82-year-old client is provided access to the hospital Patient Advocate for processing of a complaint. D. The parents of a 13-year-old are included in discussions about the course of their teen's treatment and care.

B. A 67-year-old client with dementia is shown the same respect as his 47-year-old roommate with prostate cancer. Rationale B. The principle of justice refers to equality—all clients should be treated equally and fairly as is demonstrated by the respect shown to the client with dementia. A similar example is seen in the text.

"I can use as much salt substitute as I want."

The nurse is instructing a patient with heart failure about a prescribed sodium-restricted diet. Which patient statement indicates that additional teaching is required?

Which role of the medical-surgical nurse is demonstrated by writing a care plan for the client who is newly admitted to the hospital? A. Advocate B. Caregiver C. Communicator D. Educator

B. Caregiver Rationale B. In the caregiver role, medical-surgical nurses assess clients, analyze collected information to determine their needs, develop nursing diagnoses and collaborative problems, plan care and carry out the plan with the healthcare team, and evaluate the care given.

A nurse is asked to collaborate with others to implement an interdisciplinary (ID) plan of care for a client. Which health care team members are essential for the client's daily care regimen? Select all that apply. A. Anesthesiologist B. Case Manager C. Health Care Provider D. Occupational therapist

B. Case Manager C. Health Care Provider

A nurse is using the concepts from SHARE (targeted solutions to ensure successful communication among care providers) to improve communication in a hospital setting. Which action indicates an understanding of these principles? A. Allowing various informed sources to contribute information B. Developing a standardized checklist form C. Encouraging the use of individualized methods of data transfer D. Minimizing input from the interdisciplinary team ("pit crew")

B. Developing a standardized checklist form Rationale B. The development and use of checklists allows for a more thorough inclusion of data, and in a more organized manner.

Which principal nursing actions best support a focus on client safety? Select all that apply. A. Client restraints B. Handwashing C. Preoperative checklists D. Respect for others E. Five rights of drug administration

B. Handwashing C. Preoperative checklists E. Five rights of drug administration Rationale The number one way to prevent infection in clients! This is a major method that facilitates a safe care environment for the client.

A nurse has recently been assigned to a medical-surgical clinical rotation. According to the scope of medical-surgical nursing, what type of client assignments does the nurse expect to see? A. Hospitalized children with acute and chronic illnesses B. Hospitalized adults with acute and chronic illnesses C. Older adults in a nursing home D. Working adults in a corporate setting

B. Hospitalized adults with acute and chronic illnesses Rationale The scope of medical-surgical nursing, sometimes called adult health nursing, is to promote health and prevent illness or injury in clients from 18 to 100 years of age or older. The most common practice setting is the acute care hospital.

Hyponatremia

Headache, lethargy, confusion, convulsions, nausea/vomiting, coma

The nurse correlates which physiological response to the secretion of natriuretic peptide hormones?

Increase in urine output -Natriuretic hormones are released in response to increased BV and BP. As a result of the release of ANP and BNP, reabsorption of sodium by the kidney and increase in glomerular filtration rate results in increased U.O that is high in sodium

Which outcome of interprofessional teams is associated with the increased focus on educating health professional students together?

Increased safety and quality

The nurse is asked to collaborate with others to implement an interdisciplinary plan of care for a client. Which health care team members are essential for the client's daily care regimen? (Select all that apply.) A. Anesthesiologist B. Case manager C. Health care provider D. Occupational therapist E. Chaplain

BC

Which principal nursing actions best support a focus on client safety? (Select all that apply.) A. Client restraints B. Handwashing C. Preoperative checklists D. Respect for others E. Five rights of drug administration

BCE

Culture and Death: African-Americans

Believe dying in the home brings back luck to the house. Prefer death occur in a hospital and may prefer that professionals prepare body for burial

Culture and Death: Judaism and other cultures

Believe that dying persons must have someone with them as their soul leaves body. Traditionally body is not left alone until burial which is in 24 hrs or ASAP. No flowers permitted after 7 days, body dressed in shroud, no metal objects including nails, mirror covered and immediate family sits on low hard benches during the mourning period.

Culture and Teaching: Asians

Believe the head is scared and therefore must not be touched or patted. Palpating the fontanel of an infant may be interpreted as a disrespectful action unless properly explained and accepted by parents

Ethnocentric

Believing one's culture, beliefs and values to be superior to others

Cultural Sensitivity

By observing and demonstrating knowledge of culturally appropiate verbal language, body language, use of personal space, and gestures of respect toward family members.

Patient protectional and affordable care act

Increasing access, improving quality and safety and lowering cost are basic principles. Transitional care programs have demonstrated that with the prevention of avoidable readmissions, patient safety and quality of care improve and costs are lowered.

What is the culture for adolescent?

Independence in adolescents is not valued equally by all cultures. These practices can be symbolic rituals or celebrations.

Pitting edema in the lower extremities

The nurse is planning care for the patient with acute renal failure. The nurse plans the patient's care based on the nursing diagnosis of Excess Fluid Volume. Which assessment data supports this nursing diagnosis?

The medical-surgical nurse implements SBAR to promote effectiveness in which aspect of quality and safety in patient care?

Communication

What two factors are considered providing culturally competent care?

Communication and personal space

Unique patient situations requiring or enhanced by interprofessional collaboration

Complex patients with multisystem disease require the expertise of many disciplines. Examples include transplant patients, those with a chronic disease such as CHF or chronic pulmonary obstructive disease patients, and oncology patients. Another example is the catastrophic traumatic injury patient involving orthopedic, neurological, cardiovascular, and respiratory issues.

Cultural care

Consists of health-promotion activities intiated by a culturally competent health-care worker who enables a patient to modify health behaviors toward beneficial outcomes while respecting the patient's cultural values, beliefs, and practices

The patient is receiving sodium bicarbonate intravenously (IV) for correction of acidosis secondary to diabetic coma. The nurse assesses the patient to be *lethargic, confused, and breathing rapidly*. Which is the nurse's priority response to the current situation?

Continue the infusion, because the patient is still in *acidosis*, and notify the provider.

Beliefs

Cultural teachings of practices and values that are handed down for generations and determine how one behaves and responds to daily life and health-care practices.

Culture and Teaching: Native American, Chinese and Japanese

Cultures suggest that silence indicates respect for another person

What are alternative healers for Hispanics?

Curandero, espiritualista, yerno

The nurse is working in the intensive care unit. When does the nurse call the Rapid Response Team (RRT)? A. An 87-year-old client awakens mildly confused, then reorients quickly. B. A newly admitted client requests pain medication. C. A postoperative client's dressing has mild bloody drainage. D. A postoperative client's blood pressure suddenly drops.

D

The nurse supports the client and family in deciding on a "Do Not Resuscitate" order. Which ethical principle that guides nursing clinical decision making is demonstrated in this situation? A. Beneficence B. Justice C. Legality D. Self-determination

D

A nurse is working in the intensive care unit. When does the nurse call the Rapid Response Team (RRT)? A. An 87-year-old client awakens mildly confused, then reorients quickly. B. A newly admitted client requests pain medication. C. A postoperative client's dressing has mild bloody drainage. D. A postoperative client's blood pressure suddenly drops.

D. A postoperative client's blood pressure suddenly drops. Rationale D. The RRT should be called whenever a client has a slow or sudden deterioration in clinical condition.

Case Management (CM)

May come from a variety of backgrounds, typically an RN with an advanced nursing degree or a social worker. This team member utilizes the processes of assessing, planning, facilitating, advocating, and providing available resources to meet the individuals health needs with sound quality and cost-effective outcomes in mind.

Culture and Teaching: Russian, French and Spanish

May interpret silence as a agreement with speaker

Culture and Illness: Islamic faith

May request Koran be kept at the bedside and that nothing be placed on top of it

A nurse supports the client and family in deciding on a "Do Not Resuscitate" order. Which ethical principle that guides nursing clinical decision making is demonstrated in this situation? A. Beneficence B. Justice C. Legality D. Self-determination

D. Self-determination Rationale D. Self-determination refers to the idea that clients are autonomous individuals capable of making informed decisions about their care. When the client is not capable of self-determination, the nurse is ethically obligated to protect the client as an advocate in the professional scope of practice.

LPN

May work under the direction of a RN and charge nurse. The nurse can perform specific clinical function in the hospital as well as in outpatient care settings as directed by the RN.

What are alternatives for Native Americans?

Medicine man, Shaman

Alternative medicine

Medicines used instead of Western medical care

Culture and Food: Jewish religion

Mixing dairy and meat products at the same time are prohibited, pork is avoided, all meats must be specially prepared or koshered, only fish with scales are permitted

Culture and Death: Central American

Death with dignity in the home setting is preferred over a hospital setting, because death is considered a spiritual event

(CC 8.4) Which change in fluid and electrolyte balance is most closely associated with aging?

Decreased olfactory function

Values

Deep feelings about what is right or wrong, good or bad.

Hyperchloremia

Deep, rapid respirations, lethargy, tachypnea, decreased cognitive ability, and hypertension

What are hot illnesses/ conditions?

Dental problems, sore throat, rashes and kidney disorders.

Hypermagnesemia

ECG changes, widened QRS, hypotension, bradycardia, drowsiness, lethargy, decreased deep tendon reflexes

Care Transitions Program

Developed by Eric Coleman at the University of Colorado Heath sciences center. Employs nurses are "transition coaches" to manage chronically ill or seriously ill patients as they transition between health-care settings. Patients receive tools to learn self management skills that include recognizing and responding to "RED FLAGS" that indicate worsening of their condition, managing medications, managing their own personal health record, and completing follow-up care, with their PCP.

Culture and Food: Hispanic

Diets incorporate the concept of cold foods, such as veggies, fruits and dairy products and hot foods such as garlic, grains and elected cuts of meat.

What is non verbal communication for Native American?

Direct eye contact is considered disrespectful

What is non verbal communication for African Americans?

Direct eye contact is often considered rude

What is non verbal communication for Asians?

Direct eye contact with superiors is often considered disrespectful

Hypernatremia

Disorientation, hallucinations, agitation, restlessness, confusion, seizures, lethargy, tachycardia, dry mucous membranes, skin flushed, agitation, thirst

Culture and Food: Mormons

Don't consume alcohol or any beverage containing caffeine and many fast on the first Sunday of each month

Culture and Illness: Orthodox Jews

Don't turn on electricity nor do they use the telephone from sunset Friday night to sunset Saturday. Medical behaviors/appts must be postponed without endangering the patient. Shaving can be done with an electric razor but a razor blade must not touch the skin.

The nurse is reviewing the health-care provider orders for a patient who is diagnosed with *respiratory alkalosis*. Which prescription is appropriate for this patient's care needs?

Draw arterial blood gas analysis. -management of respiratory alkalosis focuses on correcting & treating the underlying cause. ABGs must be ordered prior to beginning meds or oxygen therapy

Teach Back

Effective teaching strategy used to enhance patient education setting. Patient teaching can be conducted in any setting (hospital bedside, clinic, or home) by any member of the team.

What is family organization for Asians?

Emphasis may be on family needs rather than individual needs. The opinion of the elder is sought.

What is family organization for Native American?

Emphasis tends to be on the family rather than on individual needs

Which intervention should the nurse implement for a patient whose *serum phosphorus level is 2.0 mg/dL*?

Encourage consumption of milk and yogurt A phosphorus level of 2.0 is low, and the client will need additional dietary phosphorus. Providing phosphorus-rich foods such as milk and yogurt is a good way to provide that additional phosphorus.

(Box 9.6) Causes of *Metabolic Acidosis*

Excess of acid • Renal failure • Ketoacidosis • Anaerobic metabolism • Starvation • Salicylate intoxication Loss of base • Diarrhea • Intestinal fistulas

What is verbal communication for Hispanics?

Expression of negative feelings is considered impolite

Entry-level nursing programs, including associates, bachelor, and master (including clinical nurse leader), and doctoral degrees

Focus on the preparation of nurses to practice competently in the various medical-surgical settings.

interprofessional education (IPE)

Future generations of ICTs are being shaped and interprofessional education (IPE) is a part of that development. It is defined as two or more disciplines learning how they work together in the hospital setting and outpatient settings.

Transcellular

GI, respiratory, urinary tracts, glandular, intraocular, cerebrospinal

Self-Determination Act of 1991

Granted patients in the US the legal right to full disclosure of medical information to allow individuals to participate in their own care

Rapid response teams (RRT)

Grew as a result of TCAB efforts. These teams in acute care settings are compromised of clinicians who provide critical care expertise at the patients bedside or point of care and typically include a critical care provider or intensivist, critical care RN, pharmacist, and respiratory therapist. They respond when a patient shows signs of deterioration in an effort to respond and reverse the problem before a cardiac arrest occurs.

What is verbal communication for Asians?

High respect for those in position of authority

The medical-surgical nurse needs to be knowledgeable of which organization that scores patient-centered care via a telephone survey of selected patients after discharge?

Hospital consumer assessment of healthcare providers and systems (HCAHPS)

Culture and Health Beliefs:

Humoral theory: many non-western cultures believe that balances between hot and cold affect health and illness.

What is Native American culture for pregnancy for husbands?

Husband avoids eating meat while the woman is in labor and delivery

Orthodox Jewish religion husband during pregnancy?

Husband may not participate in prenatal classes or during labor and hey may not view the infant during the birth process. Only verbal encouragement is usually allowed?

What do Arabic, Chinese, Cuban and Ethiopian pregnancy for husband?

Husband remains in control of decisions but does not otherwise participate in the birthing process.

crystalloid solutions

IV solutions composed of electrolytes dissolved in water and include dextrose solutions, sodium chloride solutions, balanced electrolyte solutions, and alkalizing and acidifying solutions

colloidal solutions

IV solutions composed of larger molecules, usually protein or starch, suspended in fluid and are not a true solution frequently referred to as plasma volume expanders because the larger molecules do not diffuse through cell membranes and draw fluid into the intravascular space used to maintain intravascular volume and prevent shock after major blood or fluid losses examples include albumin, dextran, and mannitol

Patient-centered care

Importance is demonstrated by its incorporation into TJC's accreditation standards for hospitals. Interprofessional coordination and teamwork are also essential to effective patient outcomes in the complex health system in the US. Medical-surgical nurses work closely in interprofessional teams including providers, social workers, pharmacists, and other disciplines to plan and implement the best evidence-based practice.

Competencies Related to the Nursing Process

Important to the practice of medical-surgical nursing:Competencies in the nursing process, clinical decision making, evidence based practice, patient-centered care, quality and safety, and interprofessional practice.

Culturally competent care

Improve communication, enhance education, and aid in achieving the established goals of Healthy People 2020.

Understanding culture as related to health care will

Improve services and health-care outcomes. Enhance cost effectiveness of health care. Reduce errors caused by misunderstandings. Assist in reaching Healthy People 2020 goals.

Personal space

In American culture, an intimate zone to be up to 1.5 feet, personal distance is 1.5 to 4 feet, social distance is 4 to 12 feet, and public distance is 12 feet

B. 31 drops/min

In order to deliver 0.9% sodium chloride at 125 mL/hr, the nurse has selected an infusion set with a drop factor of 15 drops/mL. What is the correct drop rate require to deliver this volume? A. 15 drops/min B. 31 drops/min C. 60 drops/min D. 125 drops/min

Culture and Teaching

In the Us, the myriad of cultures creates an opportunity for misunderstandings and misinterpretations of health-care teachings. People from cultures that place a high value on pleasing others may answer questions with information they think others want to hear to maintain a harmonious relationship

Hypermagnesemia treatment

Increase fluids

"I don't mind doing anything within reason, but you have a responsibility to be considerate to the staff as well."

The nurse is providing care for a postpartum patient who states, "I know my rights and you have to do what I tell you!" Which response by the nurse is appropriate?

Which is the *priority* nursing action when providing care to a patient who is admitted with *metabolic alkalosis*?

Monitoring *oxygen saturation* -the depressed respiratory drive that often accompanies metabolic alkalosis can lead to hypoxemia and impaired oxygenation of the tissues

The values of the parents

The nurse is providing care to a 3-year-old child whose parents decide to decline further treatment for cancer, which has metastasized. There is a conflict between the child's parents and the rest of the family. Which should the nurse consider when determining the appropriate action for this patient?

Guided care program

Is an example of a PCMH that has improved patient outcomes and quality and reduced costs through nursing interventions

A patient is prescribed *20 mEq of potassium chloride due to excessive vomiting*. Which is the rationale for this drug the nurse should provide to the patient?

It is needed to *maintain skeletal, cardiac, and neuromuscular activity*. Potassium is the major cation in intracellular fluids, with only a small amount found in plasma and interstitial fluid. *Potassium is a vital electrolyte for Skeletal, Cardiac, & SM activity*.

Culture and Food

Many cultures and religions include specific foods as an integral part of holiday celebrations and may restrict consumption of specific foods.

CAM therapies include

Massage, energy healing, acupuncture and acupressure, reflexology, homeopathy, biofeedback, aromatherapy, guided imagery, herbal remedies, and others

How does the United States view birth control vs different cultures?

Most cultures there's respect for age so menopause is an insignificant event. In the United States,

Hyperkalemia

Muscle cramps, paresthesia to weakness, ECG changes (abnormal rhythms, widened QRS complex)

Hyperphosphatemia

Muscle cramps, paresthesia, weakness, decreased DTRs, ECG changes

Project RED (Re-engineered Discharge) and Project BOOST (Better outcomes for older adults thought safe transition)

National programs that also seek to improve care as patients transition from acute care settings to post-acute care settings by improving the discharge process.

Transitional Care model(TCM)

Nurse-led multidisciplinary program that was developed by advanced practice nurse Mary Naylor at the at the University of Pennsylvania. Using evidenced-based care coordination methods, the TCM collaborates with other health care team members including providers, nurses, social workers, discharge planning, and pharmacists to plan care. The TCM focuses on the chronically ill, high risk adults. Common medical and surgical conditions include heart failure, DM, CKD, and post surgical hip surgery.

American Nurses Association Code of Ethics

Nursing first began to discuss a code of ethics in 1896, when a group of nursing leaders met to discuss the practice of nursing in the United States. The forerunners to the official code of ethics were generally based on the (Florence) Nightingale Pledge, which was and is still administered to graduating nurses. That first organization to address the code of ethics for nurses eventually became the American Nurses Association (ANA), and the first official code of ethics for nurses, the Code for Professional Nurses, was accepted by the ANA House of Delegates in 1950. The Code for Professional Nurses has been updated and revised over the years to the Code of Ethics for Nurses (the Code) we use today (Box 4.1). It reflects the moral traditions, values, and trust that the public has come to associate with nursing. The obligations that nurses have toward themselves, patients, and communities are guided by ethical principles. There are no right or wrong behaviors identified in ethical principles, only guidelines for the ethical practice of nursing.

Communication

Obtain interpreters yo aid in data collection and in providing information in the patient's native language. Gestures and body language are important. Some cultures value eye to eye contact, whereas others tend to avoid direct eye contact

Transitional Care

Patient centered and typically manage the transitions of patients from acute care to pose-acute care settings. The goals are to avoid poor health outcomes, ensure continuity of care, and safe transition of care between health care settings.

Also included in the affordable care act are programs and initiatives focusing on

Patient centered medical homes (PCMH)

Culture and Teaching: Asian, Native American, and Muslim

Patients consider direct eye contact impolite and may stare at the floor as a symbol of respect during health care worker, which should not be be misinterpreted as not paying attention

SBAR

One approach to decreasing communication barriers and focuses on a standard way to state the situation, background, assessment, and recommendation.

Medical-surgical nursing certification board(MSNCB)

Over the years, medical surgical nursing has developed into a specialty practice area, and RNs who meet requirements are eligble for certification in medical-surgical nursing. These nurses earn the credential Certified medical-surgical registered nurse. (CMSRN)

The nurse is reviewing the latest ABG results for a patient with *metabolic alkalosis*. Which result indicates that the *metabolic alkalosis is compensated*?

PaCO2 48 mmHg

Hypomagnesemia

Paresthesias, tetany, twitching, tachycardia, nausea/vomiting, seizures, anorexia, and dysrhythmias such as rapid heart rate and PVCs, ventricular tachycardia, emotional lability, and deep tendon reflexes increased

The TCN, a component of the TCM is an example of a

Patient Care coordinator

Culture and Teaching: Hispanics

Patients may view extended eye contact as related to the evil eye, which will bring bad luck

(CC 8.7) Which nursing action is the priority in the patient with a serum calcium of 6.0 mg/dL?

Place an intubation tray at the bedside -in pt w/ HYPOcalcemia, due to potential of laryngeal stridor, cardiac dysrhythmias & seizures, emergency equipment such as intubation equipment, tracheostomy set, code/emergency cart & defibrillator must be ready

Culture and Death: Hindu

Priest may place a thread around the neck or wist of the deceased as a blessing before cremation which is preferred over burial

Culture and Illness: Roman Catholic

Priest notified to come before patient loses consciousness. Abstinence from solid food is required for at least 15 mins before clergy offers. Rosary beads/medallions may be pinned to gown or bed of the ill

Culture and Food: Hindu

Prohibits consumption of all meat

Agency for Healthcare Research and Quality (AHRQ)

Promotes behaviors and policies related to cultural competence in health care; put in place by USDHHS of Minority Health and Cross Cultural Health Care

Recommended standard of CLAS

Promoting attitudes, behavior, knowledge, and skills necessary to work effectively in a culturally diverse work environment

U.S. Office of Minority Health

Provides information concerning culturally and linguistically (language) appropriate health services (CLAS)

Cultural awareness

Recognizing the history of patients' ancestry or culture and how their customs influence handling of problems, issues or teachings.

Culture and Illness: Jehovah Witnesses

Receiving blood transfusions or medications containing blood products violates religious guidelines

Personal values

The nurse is providing care to a client who is considered brain dead. The family has opted to end care and the health-care provider asks the nurse to pull the endotracheal (ET) tube. The nurse is uncomfortable with this request. Which is the reason the nurse is experiencing difficulty with this task?

Transitional Care: Required Competencies of the Transitional Care Nurse

Required Competencies of the Transitional Care Nurse • Clinical expertise • Evidence-based care experience • Understanding of health-care system, delivery models, and roles of various health-care professionals • Knowledge of community-based social services • Experience with management of complex patients • Comfort with independent role • Experience in hospital and home-care settings • Flexibility and adaptability • Assertiveness, maturity, and confidence • Effective communication skills • Natural ability to collaborate and facilitate relationships with patients, families, and other members of the health-care team

Time-outs

Requires all personnel involved in the procedure to stop to make sure that the patient is identified, the correct anatomical site is identified, and all equipment is in working order, and special procedures for marking surgical sites.

Culture and Death: Islamic culture

Requires body to be placed in a position facing Mecca. Persons do not own their bodies therefore cremation, autopsy and organ donation are prohibited

The nurse is providing care to patient with the following laboratory values: pH - 7.31; PaCO2 - 48 mmHg; and a normal HCO3. Which condition should the nurse plan care for based on the current data?

Respiratory acidosis -to compensate for this imbalance, the rate and depth of respirations decrease, leading to retention of CO2. PaCO2 will be elevated.

In designing an evidence-based practice guideline for a medical-surgical unit, which source provides the most reliable information?

Results from research studies

Culture and Adult: Roman Catholic Church and the Mormon Church

Roman Catholic Church and the Mormon Church allow only natural family planning, using abstinence as the technique of choice

Core aspects of CAM

Self-care, wellness and illness prevention

Focus of health care

Shifting from the hospital to the home and community with a focus on wellness and prevention of illness.

Approach to jewelry strings

Should not be removed unless medically necessary

SBAR Approach to effective communication

Situation- Brief statement of the problem or issue being addressed. Background- Data related to the current situation. Assessment- Summary of causes, significance, severity of situation. Recommendation- Specific actions needed to address the situation. The SBAR approach method provides a consistent process for communication, particularly in high-risk situations. Of particular value is the focus on making a recommendation that focuses on a definite approach to addressing the issues or clinical problem.

*Extra*cellular

Sodium Chloride Bicarbonate

A reduction of extracellular fluid

The nurse is providing care to a patient following hemodialysis. The patient is experiencing tachycardia and decreased urine output along with skin that is pale and cool to the touch. Which goal of hemodialysis does the nurse determine the patient has not met based on the current data?

Natural compensatory mechanisms

The nurse is providing care to a patient who is diagnosed with multisystem fluid volume deficit. The patient is currently experiencing tachycardia and decreased urine output along with skin that is pale and cool to the touch. The patient has a decreased urine output. Which probable cause to the patient's symptoms should the nurse include when educating the family?

-Initiate hypodermoclysis -Closely monitor patient's I&O's -Initiate intravenous therapy

The nurse is providing care to a patient who is exhibiting clinical manifestations of a fluid and electrolyte deficit. Based on this data, which health-care provider prescriptions does the nurse prepare to implement? Select all that apply.

Bananas

The nurse is providing care to a patient who is prescribed furosemide as part of the treatment for congestive heart failure (CHF). The patient's serum potassium level is 3.4 mEq/L. Which food should the nurse encourage the patient to eat based on this data?

Eat something salty when drinking water

The nurse is providing care to a patient who seeks emergency treatment for headache and nausea. The patient works in a mill without air conditioning. The patient states, "I drink water several times each day but I seem to sweat more than I am able to replace." Which suggestions should the nurse provide to this patient?

Take the patient's issue to the hospital ethics committee

The nurse is providing care to a patient who states, "My doctor is refusing to treat me because I am noncompliant with his recommendations." Which is the priority nursing action in this situation?

Encourage ambulation three times a day

The nurse is providing care to a patient whose serum calcium levels have increased since a surgical procedure performed three days prior. Which intervention should the nurse implement to decrease the risk for the development of hypercalcemia

-Emphasizing the importance of the test to the patient -Offering counseling regarding the testing -Encouraging the patient to reconsider the decision to be tested throughout the pregnancy

The nurse is providing care to a pregnant patient with a history of drug use. The patient refuses testing for human immunodeficiency virus (HIV) despite the recommendation of her nurse-midwife. Which actions by the nurse are appropriate in this situation? Select all that apply

Hypotonic dehydration

The nurse is providing care to an adult patient admitted with dehydration and hyponatremia. Which medical condition supports the current nursing diagnosis of Electrolyte Imbalance?

HYPOvolemia CM's

Weight loss, loss of skin turgor, concentrated urine output, oliguria (low urine output), thirst, and dry mucous membranes are indications of fluid volume deficit additional: weak, rapid peripheral pulses, flattened neck veins, hypotension, anxiety, restlessness, and cool, clammy, pale skin

"This patient is of sound mind and is capable of making independent decisions regarding health care. It really is the patient's decision to make."

The nurse is providing care to an older adult patient who has decided to discontinue the prescribed hemodialysis. The patient's family, however, is not supportive of this decision. When using the theory of principles-based reasoning, which statement from the nurse is appropriate?

short over-the-needle catheters

flexible catheter is introduced into the vein over a metal needle that is then removed and discarded therapies should be expected to last less than a week, and indications include hydration and administration of pain medications and some antibiotics

Fluid volume excess

The nurse is providing care to an older adult patient who is receiving intravenous (IV) fluids at 150 mL/hr. The patient is currently exhibiting crackles in the lungs, shortness of breath, and jugular vein distention. Which complication of IV fluid therapy does the nurse suspect the patient is experiencing?

The patient has a right to informed consent

The nurse is providing care to an older adult patient who is scheduled for surgery. During the preoperative assessment, the nurse discovers that the patient does not have an adequate understanding of the procedure. Which is the reason for the nurse to take action in this situation?

Autonomy

The nurse is providing care to an older adult patient with terminal cancer who has opted to discontinue treatment and go home. The patient's family, however, wants to continue treatment. The nurse agrees to be present while the patient tells the family. Which ethical patient principle is the nurse supporting?

Hematocrit 53%

The nurse is reviewing laboratory values for a female patient suspected of having a fluid imbalance. Which laboratory value evaluated by the nurse supports the diagnosis of dehydration?

"I will have my child stop every 15-20 minutes during the activity for fluids."

The nurse is teaching a group of children and their parents about the prevention of heat-related illness during exercise. Which statement by a parent indicates an appropriate understanding of the preventive techniques taught during the teaching session?

Standard seven

addresses ethical nursing practice

Transitional Care Nurse

The TCN can be an advanced practice nurse or a Bachelors level nurse. They typical caseload for TCNs is 15-20 patients

Examples of cultures for adolescent

Some chinese cultures, for example, do not recognize adolescence as a period of development Jewish religion, usually have a bar mitzvah

What is verbal communication for Native American?

Speaks in a low tone of voice and expects listener to be attentive

Culture and Food: Islamic

Specifically prohibits pork, medications with a gelatin base might contain pork products

Culture and Illness: Buddhist and Hindu

Suggest illness is the result of sins committed in a previous life or for the atonement of sins in the present life. Time for concentrated meditation may be requested or viewed as required for healing

Culture and Death: Hmong

Suggests a person must be well dressed at the time of death. Health care worker lets family know death is near so family can bring desired clothes to the hospital. Internal metal objects must be removed from body before burial and metal objects should be prohibited from touching body after death.

Teach Back

Teach back is a patient education strategy that involves imparting knowledge and then asking for the information to be restated to ensure patient understanding. This teaching should be accompanied by written material for the patient to use as an ongoing resource. The components of teach back include: • Providing patient education in understandable plain language, avoiding medical jargon • Asking patients to repeat back what they understood in their own words in a nonshaming fashion—it is NOT a quiz! • Avoiding the use of close-ended questions such as "Do you understand?" or "Do you have any questions?" • Re-explaining any areas of confusion • Reassessing understanding by asking the patient to repeat back again • Talk slowly and teach in chunks, frequently assessing understanding

Culture and Death: Buddhist

Teaches acceptance of inevitability of death and believes the person's state of mind at the moment of death influences rebirth. Suicide, violent death or child death may require special rituals because the state of mind at death may have not been optimal

ANA scope and standards of practice

The ANA first published the Standards of Nursing Practice in 1973. This document defined a generic level of competency common to all practicing nurses and used the nursing process as the framework. Since that time, there have been revisions and updates. In 2015, Nursing: Scope and Standards of Practice (3rd ed.) was published to guide nursing practice. The scope of nursing practice describes the who, what, when, where, why, and how of nursing practice. Nursing today is practiced in many different settings requiring knowledge and skill in the biological, physical, social, and behavioral sciences. Registered nurses are licensed by the state in which they practice and work collaboratively with other health-care professionals. Nursing is a science and an art, and the scope of nursing practice is a fluid interaction of the two that is foundational to the nursing profession.

Culture and Adult: The Christian Science religion

The Christian Science religion approaches health care with a spiritual framework and may decline preventative medicines, some vaccinations and birth-control drugs are allowed

Code of ethics

The Code serves as a guide for the nurse; it does not tell the nurse what the right answer is in situations for ethical behavior. This relationship is based upon respect for the patient and the need to advocate for the patient's rights and not based upon any characteristics of the patient. Provisions Four, Five, and Six provide guidance for the nurse in the workplace environment, focusing on accountability for the nursing care provided and maintaining competency.

Culture and Adult: Islamic

The Islamic culture permits contraception but forbids abortion

Culture and Adult: The Urinarian Universalist Association

The Urinarian Universalist Association advocates birth-control practices and supports the woman's choice related to abortion

Aculturation

The adjustment to a new culture which results in differences in practice within the same cultural group.

Cultural stereotyping

The assumption that all people of one culture behave and believe the same thing

Cultural competence

The awareness of, acceptance of and respect for beliefs, values, traditions and practice that are different from one's own.

Culture and Health Beliefs: Asian Culture

The forces of yang (light heat or dryness) and yin (darkness, cold, or wetness) influence the balance and harmony of a person's state of health.

-Edema -Hematuria -Elevated blood pressure

The community nurse visits the home of a young child who is home from school because of sudden onset of nausea, vomiting, and lethargy. The nurse suspects acute renal failure. Which clinical manifestations support the nurse's suspicions? Select all that apply

Euthanasia has legal implications along with moral and ethical ones

The hospice nurse is providing care to a terminal patient who has asked about guidance and support in ending life. Which should the nurse recognize in regards to making an ethical and moral decision in this circumstance?

South American, Vietnamese, and West Indian husband for pregnancy?

The husband is expected to be nearby

C. Causes no or equal movement of fluid into or out of cells

The nurse anticipates which fluid movement when administering an isotonic IV fluid to a patient? A. Causes fluid to move from the cells into the intravascular space B. Causes fluid to move from the intravascular space into the intracellular space C. Causes no or equal movement of fluid into or out of cells D. Causes fluid to move from the intravascular space into the interstitial spaces

Treatment is effective and should continue

The nurse is analyzing the intake and output record for a patient being treated for dehydration. The patient weighs 176 lbs. and had a 24-hour intake of 2,000 mL and urine output of 1,200 mL. Based on this data, which conclusion by the nurse is the most appropriate?

They can potentiate hyperkalemia.

The nurse is caring for a patient admitted with hypertension and chronic renal failure who receives hemodialysis three times per week. The nurse is assessing the patient's diet and notes the use of salt substitutes. When teaching the patient to avoid salt substitute, which rationale supports this teaching point?

"Fluid volume excess is common due to increased levels of antidiuretic hormone in response to the stress of surgery."

The nurse is caring for a patient who is receiving intravenous fluids postoperatively following cardiac surgery. The nurse is aware that this patient is at risk for fluid volume excess. The family asks why the patient is at risk for this condition. Which response by the nurse is the most appropriate?

"The insulin will cause his extra potassium to move into his cells, which will lower potassium in the blood."

The nurse is caring for a patient with a potassium level of 5.9 mEq/L. The health-care provider prescribes both glucose and insulin for the patient. The patient's spouse asks, "Why is insulin needed?" Which response by the nurse is the most appropriate

A 15-month-old child with tachypnea

The nurse receives shift report on a pediatric medical-surgical unit. The nurse has been assigned four patients for the shift. Which child does the nurse plan to assess first based on the increased risk for dehydration?

A. Precautions for preventing infection B. Signs and symptoms to report D. Purpose of infusion therapy

The nurse recognizes that education for patients receiving infusion therapy should include which information? (Select all that apply.) A. Precautions for preventing infection B. Signs and symptoms to report C. Manufacturer of the IVAD D. Purpose of infusion therapy E. Calculation of infusion rates

C. 3 0-no clinical symptoms 1-erythema at access site with or without pain 2-pain at access site with erythema and/or edema 3-pain at access site with erythema, streak formation, and/or palpable cord 4-pain at access site with erythema, streak formation, palpable venous cord greater than 1 inch in length, and/or purulent drainage

The nurse recognizes the presence of pain, redness, and streak formation at the site of a peripheral IV site as what grade of phlebitis? A. 1 B. 2 C. 3 D. 4

B. Hyperglycemia

The nurse recognizes the prevention of which complication as the primary rationale for initiating Total Parenteral Nutrition at a slow rate? A. Infection B. Hyperglycemia C. Discomfort D. Air embolism

A. Patient who sustained trauma presenting to the emergency department B. Patient with osteomyelitis receiving IV antibiotics at home D. Patient with cancer receiving parenteral chemotherapy

The nurse recognizes which patient may need infusion therapy? (Select all that apply.) A. Patient who sustained trauma presenting to the emergency department B. Patient with osteomyelitis receiving IV antibiotics at home C. High school teacher during an annual health appraisal D. Patient with cancer receiving parenteral chemotherapy E. Patient with multiple sclerosis in remission

Project BOOST

The objectives are led by the society of hospital medicine, are to identify patients at risk for readmission on admission, reduce 30-day readmission rates, decrease length of stay, and improve communication of patient care information during discharge.

Academy of Medical-surgical nurses (AMSN)

The professional nursing organizasion for medical-surgical nurses, the foundation of all nursing practices has its roots in this practice area. (medical-surgical nursing that it)

Physical Therapists (PT)

The professional who assesses and provides training and education on physical strength, gait, and mobility for the patient.

Occupational Therapist (OT)

The professional who assesses and retrains the patient to perform activities of daily living. (brushing teeth, bathing, cooking, dressing, etc.)

Registered Dietitian/Nutritionalist (RD)

The professional who assesses the patients nutritional needs, develops meal plans, and provides education about dietary modifications related to the individuals disease process.

Speech-language pathologist (SLP)

The professional who assesses, diagnoses, and treats patients with disorders relating to speech, language, swallowing, voice, and cognitive communication.

Respiratory Therapist (RT)

The professional who specializes in airway management.

Registered Nurse (RN)

The professional who utilizes the nursing process to care for the patient: to assess, plan, implement, and evaluate. The plan is implemented and evaluated; revisions are made on the basis of the evaluation. Because 24/7 round-the-clock responsibility of RN's they are the eyes and ears of the health-care team members. The RN provides feedback to the provider on the efficacy of medical interventions and reports normal and abnormal objective or subjective assessment findings. The RN also functions in a charge nurse role. In that role, the nurses focus is broader than a specific nursing assignment but includes a focus on the complete nursing unit: patients being admitted, patients being discharged, patients experiencing difficulties.

Rights

The rights theory can be useful if it is used in conjunction with another ethical theory; it cannot stand alone.

Overview of transitional care

The safety of the patient is at risk during transitions between care settings, particularly following an acute hospitalization. The readmission rates of hospitalized patients, particularly medicare beneficiaries, are one driving factor in the call for improved transitional care services. 20% of patients are hospitalized within 30 days of discharge. These readmission rates are often attributed to a lack or coordination of care as patients are discharged to rehab facilities, long-term care agencies, or back to their homes.

-Reduce the intake of coffee and tea. -Drink more fluids during hot weather. -Drink flat cola or ginger ale if vomiting.

The school nurse is preparing a class session for high school students on ways to maintain fluid balance during the summer months. Which interventions should the nurse recommend Select all that apply.

Teleology

The teleological perspective is sometimes called the greater good perspective. The term telos is a Greek term meaning "end, purpose, or goal." This perspective determines the rightness or wrongness of a decision or behavior by the end results of the action. Using this perspective, a patient in a vegetative state on a life-support system can be removed from the ventilator and allowed to die. The health-care team has a duty not only to the patient but also to the family and society in general. If the deontological perspective is used in the decision as to whether to place the device, the device will be placed because of the duty to do no harm.

Deontology

The term deontology is derived from the Greek term deontos, which means "a duty to obey." Using this perspective, a patient in a vegetative state will not be removed from life support if the removal of the assistive device will hasten death. The health-care team has a duty to do no harm, and removing life support would result in death.

Values

The value system of the nurse, as well as the values of the organization in which the registered nurse is employed and those of other health-care professionals, is fundamental to ethical behavior.

Complementary medicine

Therapies used with Western therapies

Justice

Throughout the Code, the predominating principle is justice. Nursing has a strong commitment to social justice at every level, whether that commitment is for a single patient or a group of people. Social justice relates to the equal treatment of all persons irrespective of race, ethnicity, gender, social status, or religion.

What is touch for Hispanics?

Touch is common between two people having a conversation

What is touch for African American?

Touching another's hair is considered offensive

What is touch for Asians?

Touching the head of a newborn is considered bad luck. Prefers not to shake hands with opposite sex.

Culture and Health Beliefs: What do Latin American, African, Hispanic, Haitian and Chinese believe?

Traditional cold remedies to read hot diseases and hot remedies to treat cold diseases.

Transition Guide

Typically an RN with an advance practice degree, assists patients during and after transitions (such as hospital to home or skilled facility) with the goals of ensuring a smooth transition, identifying and managing readmission risks, and assisting with aligning possible resources to aid in the patients plan of care.

Home care coordinator (HCCS)

Typically an RN, identifies home-care needs proactively while the patient is in the hospital/skilled nursing facility or emergency department or in lieu of hospitalization, the HCC may coordinate care from a providers office.

Hypokalemia

Weakness, lethargy, hyporeflexia, ECG changes (ST depression), PVCs, nausea/vomiting, constipation, abdominal cramping

Hypophosphatemia

Weakness, slurred speech, irritability, confusion, increased bleeding

Culture and Illness: Religious Mormons

Wear a sacred undergarment that should not be removed unless an emergency occurs

Knowing the Organization you are going to work for

When a nurse is seeking employment, it is important that he or she examine the beliefs of the organization before accepting a position. For example, it is important that the registered nurse is knowledgeable about the procedures performed at a place of employment that focuses on reproductive services in case they conflict with nurse's individual opinions about birth control, abortion, or other reproductive practices.

Investigating all aspects of the situation

Which action is appropriate when dealing with an ethical dilemma in practice?

-The need to support coworkers in their efforts to improve working conditions -The need to ensure that clients receive care and are not abandoned -Loyalty to the nurse's employer

Which are ethical issues for the nurse to consider prior to deciding whether or not to honor the picket line during a strike situation? Select all that apply

Water intake of 2 glasses per day

Which data collected by the nurse during the assessment process places the older adult patient at risk for dehydration?

Beneficence

Which ethical principle is the nurse assessing when asking who benefits from the actions of others?

Fidelity

Which ethical principle requires the nurse to be accountable for commitments made to self or others?

Encourage consumption of milk and yogurt

Which intervention should the nurse implement for a patient whose serum phosphorus level is 2.0 mg/dL?

Determine exactly what needs to be decided

Which is the priority nursing action for the ethical decision-making process?

Three

Which number of alternative solutions should be included when conducting ethical decision-making?

Social justice

Which professional value is the nurse demonstrating by volunteering time to work in a local free clinic?

-The hospital's patient rights statement -State and federal patient rights legislation

Which should the nurse be aware of when preparing to act as a patient advocate in the hospital setting? Select all that apply

"It provides standards for professional nursing practice."

Which statement best describes the American Nurses Association (ANA) Code of Ethics for professional nurses?

"It guides nurses in their professional behavior and relationships."

Which statement regarding the American Nurses Association (ANA) Code of Ethics for professional nurses is accurate?

Culture and Adult: Jewish religious culture

Women are considered in a state of impurity during menstruation and after giving birth. Very religious husbands may not touch their wives during these times.

What is family organization for the African American?

Women play key roles in health-care decisions

Culture and Health Beliefs: Yang

Yang (hot foods) include meat, eggs, hot soup, cantaloupe and fried foods.

Culture and Health Beliefs: Yin

Yin (cold foods) include fruits, veggies, cold liquid and beer

osmolarity normal range

[*170-300*] mOsm/L

Serum magnesium

[1.6-2.6] mg/dL

Serum sodium

[135-145] mEq/L

Serum phosphorus

[2.5-4.5] mg/dL

Urine osmolality

[250-900] mOsm/kg

Serum osmolality

[275-295] mOsm/kg

Serum potassium

[3.5-5.0] mEq/L

Ionized calcium

[4.6-5.3] mg/dL

Total serum calcium

[8.2-10.2] mg/dL

Serum chloride

[97-107] mEq/L

The study of ethics

a branch of philosophy

osmolarity

a measure of the concentration of the solution that is expressed in terms of number of particles (osmoles) per liter of solution this concentration influences how water moves between the intracellular and extra cellular compartments of the body may be isotonic, hypotonic, or hypertonic

bolus

a specific amount of fluid over a short time period

When providing health-care information, the health-care worker should (Select all that apply): a. speak only with the patient. b. include the family member who is the decision maker of the family. c. always provide instructions clearly written in English. d. include resources to call if there are further questions.

a, d

5. In the American culture, personal space is an intimate zone that is considered to be: a. up to 1.5 feet. b. 4 feet. c. 12 feet. d. more than 12 feet.

a. up to 1.5 feet

vesicant

able to cause blisters

incompatibility

an undesirable reaction occurring between two medications or a medication and its diluent -physical: causes a visible change that may be in the form of color, cloudiness, haziness, turbidity, formation of precipitate, and even the formation of gas (precipitate formation is the most common) -chemical: correlates to the breakdown of the medication and will most likely not be visible (most common reaction is the acid-alkaline reaction) -therapeutic: causes an increased or decreased therapeutic response that may be undetected until the patient shows no clinical response to the medication

hemolytic transfusion reaction

antibodies to the ABO antigens are in the plasma, which means ABO antigens present on the transfused blood will be attacked by the antigens in the recipient's blood if they do not have the same ABO antigens

midline catheter

are inserted in a peripheral vein in the upper extremities with tips that terminate distal to the shoulder in either the basilica, cephalic, or brachial vein are appropriate for therapies expected to last between 1 and 4 weeks

The goals of the ICN

are to 1) influence global health policy by 2) advance nurses and nursing practice and 3) bring nursing together worldwide. The code of ethics of the ICN guides registered nurses worldwide to set priorities, make ethical judgments, and take action when faced with an ethical dilemma. The core values of the ICN promote ethical practice through 1) visionary leadership, 2) innovativeness, 3) solidarity, 4) accountability, and 5) social justice. The code of ethics describes what nurses are held accountable for in terms of the core values, making nurses responsible for promoting health, preventing illness, and alleviating suffering on a global level.

Hypervolemia causes

cirrhosis, HF, stress conditions causing a release of ADH and aldosterone, adrenal gland disorders, and use of corticosteroids, extra salt

central venous acess

commonly referred to as a central line

implanted ports (Mediports)

consists of a small reservoir with a septum and an attached catheter; the reservoir is placed under the skin - the catheter is inserted in the vein near the reservoir, and the tip is advanced into the central vasculature in order to use the device, it needs to be accessed with a specially designed noncoring needle that is inserted using sterile technique through the skin and into the septum of the reservoir are also used for long-term therapy and offer the added advantage of requiring minimal care when not in use

3. If a patient refuses to take his vitamin pill because he believes the gelatin base is unhealthy, the nurse should: a. challenge his belief. b. educate him concerning the need and value of that vitamin pill. c. obtain the help of a clergyman to persuade him to take the pill. d. respect his belief and find another vitamin preparation.

d

A person's culture includes a. age. b. genetic predisposition. c. handicap. d. customs

d. customs

peripheral venous access

describes when the tip of the IVAD terminates outside of the central vasculature in a peripheral vein includes the short over-the-needle catheter, steel-winged device (Butterfly), and midline

HYPOvolemia causes

excessive loss of fluids, insufficient intake of fluid, or fluid shifts: vomiting, diarrhea, nasogastric succ, increased perspiration, hemorrhage, *diabetes insipidus*, *DKA*, adrenal insufficiency

tunneled catheter

exit the skin from a site distal from the site where they enter the vein, and are tunneled through the subcutaneous tissue between the exit and insertion site the tunneled portion of the catheter contains a Dacron cuff that tissue adheres to after insertion; the cuff stabilizes the catheter and provides a barrier to organisms, minimizing infection can be permanent and appropriate for patients requiring long-term therapy are inserted in nonemergency situations in sterile environments such as an operating or procedure room

Crystalloid Isotonic

greater than 250 mOsm/L 1. *0.9%* NS (vascular expansion) 2. Lactated Ringer's (*LR*) (electrolyte replacement) considerations: may cause: -fluid overload -generalized edema -dilutes hemoglobin -*hyperchloremic acidosis* -electrolyte imbalance -proinflammatory in large doses

crystalloid hypertonic

greater than 375 mOsm/L 1. *D5 0.45% NS* 2. *D5 0.9% NS* 3. HS 3% or 5%* considerations (may cause): -irritating to veins -fluid overload -*hyperCatremia* -*hyperChloremia* (HS slows inflammation % increases capillary permeability)

hypervolemia NM

health history, performing a physical assessment, reviewing laboratory data, weight, and calculation of I&O Physical assessment: -adventitious breath sounds (crackles) -the presence of the extra heart sound S3, -abdominal distention or ascites -peripheral edema -a distended jugular vein -altered mental status.

HYPOvolemia NM

health history, physical assessment, review of diagnostic results, and calculation of I&O LABs may include elevated hemoglobin & hematocrit, serum sodium levels, & BUN

administer hypertonic saline solution (3% or 5% NaCl)

in pt's exhibiting neurological symptoms

HYPOvolemia LABs

include -electrolytes, specifically sodium and potassium (hypokalemia) -decreased hemoglobin & hematocrit -serum osmolality -elevation of BUN to creatinine ratio -urine specific gravity, & urine osmolality

Euvolemic hyponatremia

increase in total body water w/ no evidence of edema or hypovolemia. seen in SIADH & other endocrine disorders like hypothyroidism or adrenal insuff.

steel-winged device (Butterfly)

indicated only for short-term or single-dose therapy because the rigid steel needle is more likely to puncture the vein and lead to fluid or medication leaking out of the vein often referred to as a Butterfly because of the appearance of wings on each side of the needle

peripherally inserted central catheters (PICCs)

inserted into a peripheral vein and advanced into the central vasculature the veins used for PICC insertion are usually the larger veins in the upper extremities; early placement of PICCs needs to be considered in patients who require therapies of longer than a few days to maximize the availability of veins that have not previously been accessed are frequently placed by registered nurses trained and competent in their insertion at the bedside with the assistance of ultrasound guidance

HYPO-volemia medical management

intake of oral fluids is preferred -IV isotonic solutions such as 0.9% normal saline (NS) or lactated Ringer's solution expands plasma volume and corrects hypotension -IV solutions of D5 0.45% NS or 0.45% NS can replace total body water deficits and are used as maintenance fluids

intraosseous

into bone marrow requires less skill than other venous accesses and can be accomplished in under 1 minute provides rapid central venous access in emergency situations and is recommended as an alternative route in CPR especially useful in severely dehydrated patients and for prehospital IV access frequently used sites in the adult include the proximal humerus, proximal tibia, and the distal tibia

IVAD

intravenous access device either peripheral or central should be of the smallest gauge (size) and length with the fewest number of lumens and be the least invasive to provide the ordered infusion therapy

phlebitis

irritation or inflammation of the vein wall caused by medication administered via a peripheral IV characterized by pain and erythema along the vein and is graded using a standardized scale according to signs, symptoms, and severity peripheral sites that show signs should be removed and started in another location because it can progress to more serious conditions including thrombus formation, cellulitis, and sepsis causative factors are either chemical, mechanical, or bacterial

Project RED (Re-engendered discharge)

is a research group based at Boston University Medical Center that develops and tests strategies that improve hospital discharge process. Based on 12 interrelated components that promote patient safety and decrease readmissions.

Evidence-based practice

is the foundation of professional nursing practice. Based upon a problem-solving approach to health-care delivery, nursing care practices and protocols are based upon the best evidence from research studies and the expertise of clinicians.

extravasation

leakage of IV fluid into subcutaneous tissue caused when the solution or medication that has infiltrated is a vesicant the infusion needs to be stopped immediately and corrective action taken to minimize damage to the tissue

Crystalloid Hypotonic

less than 250 mOsm/L 1. *0.25%* NS 2. *0.45%* NS 3. *0.5%* D5W considerations: -may worsen HTPO-TN -can increase edema (D5Q may irritate veins)

occlusion

loss of patency a complication of central venous infusion therapy can delay delivery of lifesaving therapies and may also mean the patient is subject to the risk and discomfort of insertion of another CVC can be the result of a thrombotic process when blood or fibrin in or around the catheter interferes with flow can also be nonthrombotic and can result from medication precipitation

Multidisciplinary "rounds"

may be conducted on nursing units

peak and trough levels

measurements of medication levels after and before administration

nonthrombotic

not resulting from a clot

air embolism

occurs when air is inadvertently introduced into the venous system can be introduced if the catheter is damaged, during insertion and removal of CVCs, and if the connections in the IV delivery system (catheter hub, tubing, injection caps) are not tightly secured with Luer-locked connections

infiltration

occurs when solution or medication is inadvertently infused into the tissue surrounding the vein and is a complication that can occur with any IVAD, peripheral or central clinical manifestations include blanched skin, skin cool to the touch, edema, unexpected pain or burning at the insertion site or along the path of the vein, and leaking of fluid from the insertion site

electronic infusion device or pump

one possible solution for when there are situations when more accurate rate control is needed than can be provided by gravity administration set roller clamps these assist the nurse in providing accurate infusion therapy, but the nurse is still responsible for monitoring delivery of the therapy

Safety Alert! HYPOvolemia risk

orthostatic or postural hypotension

The nurse is providing care to a patient who is admitted to the hospital with *sudden, severe abdominal pain*. Which arterial blood gas supports the patient's current diagnosis of respiratory alkalosis?

pH is 7.47 and PaCO2 is 25. -acute pain causes hyperventilation, causes CO2 to drop and the client to have respiratory alkalosis. The pH would denote alkalosis & be higher than 7.45. HCO3 would trend downward as the kidneys begin to compensate for the alkalosis by secreting HCO3. The PaO2 is likely to be normal unless the pt has been hyperventilating fo ra long time and is beginning to tire.

primary vs. secondary administration sets

primary sets are used to deliver the main IV solution secondary sets are attached to the primary set to deliver additional solutions or intermittent medications into a y-site in the primary infusion set; these administration sets use gravity to deliver fluids into the vasculature

Safety Alert! *Postural HYPO-TN risk*

pt w/ significant *hypovolemia* -nurse must monitor pt closely when changing the pt's position, particularly when sitting or standing, as the pt is at risk for dizziness and falls

2. Culturally competent care is provided by: a. using scientific evidence. b. following all orders written by the health-care provider. c. involving the patient and family in developing the plan of care. d. researching folk laws and remedies from the cultural background of the patient.

researching folk laws and remedies from the cultural background of the patient

hypervolemia LABs

serum electrolytes, hematocrit, low BUN, serum osmolality, and albumin -Hypoalbuminemia in liver dys-f(x) -Hyponatremia and low serum osmolality may be present in chronic renal failure b/c of diluted BV

veins for peripheral access

short-term peripheral IV access is established in the superficial veins of the upper extremity. it is recommended to start therapy in the most distal available and appropriate vein and move upward with subsequent insertions. veins selected for short-term infusion therapy should be soft (nonsclerotic), nontender, and not in an area where a previous infusion has infiltrated. areas of the vein containing a valve should also be avoided. veins in areas of flexion such as the wrist and antecubital fossa should be avoided if possible because it is more difficult to stabilize the IVAD in these areas

flushing

the act of moving fluids, medications, blood, blood products, and nutrients out of a vascular access device into the bloodstream, ensuring delivery of these components and verifying device patency

locking

the instillation of a solution into a vascular access device to maintain device patency

administration of total parenteral nutrition

total parenteral nutrition (TPN) solutions provide the major macronutrients (protein, carbohydrates, and lipids) along with required micronutrients (electrolytes, vitamins, and trace minerals) and water because of the high osmolarity of TPN solutions, they should be infused into a CVAD with the tip placement confirmed in the vena cava the administration set used to infuse TPN should be changed every 24 hours to decrease the risk of contamination and infection

administration of blood products

transfusion of whole blood is rare because it is more efficient to administer only the portion of the blood, or component, required by the patient (red blood cells, fresh frozen plasma, platelets, granulocytes, clotting factors, and albumin) close observation is required to detect any reaction the patient may have to the blood product, especially during the first 15 minutes when reactions are more likely to occur, and at least every hour for the duration of the transfusion (assessments need to include vital signs, respiratory status, and any symptoms of discomfort, dyspnea, or itching)

hypervolemia CM's

weight gain, ascites, edema, and increased urinary output. cardiac CM: HTN, tachycardia, elevated CVP, development of S3 heart sounds, and jugular vein distention

central venous device (CVAD)

when the tip of the IVAD terminates in the central vasculature at the level of the superior vena cava or the inferior vena cava; are made of silicone or polyurethane and come in many sizes and can have single or multiple lumens

(Box 9.2) Causes of respiratory acidosis

• CNS depression (head trauma, oversedation, anesthesia, high cord injury) • Pneumothorax • Hypoventilation • Bronchial obstruction and atelectasis • Severe pulmonary infections • Heart failure with pulmonary edema present • Massive pulmonary embolus • Myasthenia gravis • Multiple sclerosis

(Box 9.3) Clinical Manifestations of *Respiratory Acidosis*

• Dyspnea • Restlessness • Headache • Tachycardia • Confusion • Lethargy • Dysrhythmias • Respiratory distress • Drowsiness • Decreased responsiveness

(Box 9.7) Clinical Manifestations of *Metabolic Acidosis*

• Headache • Confusion • Restlessness • Lethargy • Weakness • Stupor/Coma • Kussmaul's respiration • Nausea and vomiting • Dysrhythmias • Warm, flushed skin • Seizures • Twitching • Peripheral vasodilation


Conjuntos de estudio relacionados

Microeconomics 2302 Test 2-Chapters 8, 10, 11, and 13

View Set

Steps in Conducting Survey Research

View Set